Download ID_4649_Module 5- Differential diagnos_English_sem_10

Document related concepts

Special needs dentistry wikipedia , lookup

Prenatal testing wikipedia , lookup

Differential diagnosis wikipedia , lookup

Medical ethics wikipedia , lookup

Dental emergency wikipedia , lookup

Patient safety wikipedia , lookup

Adherence (medicine) wikipedia , lookup

Dysprosody wikipedia , lookup

Electronic prescribing wikipedia , lookup

Patient advocacy wikipedia , lookup

Transcript
1.
A.
B. *
C.
D.
E.
2.
A.
B.
C.
D. *
E.
3.
A.
B. *
C.
D.
E.
4.
A.
B.
C. *
D.
E.
5.
A.
B.
C. *
D.
E.
6.
A.
B. *
C.
D.
E.
A 39-year-old, overweight man presents with suppurative periodontitis. From review his history, he
is always hungry, drinks water almost every hour, and awakens four times each night to urinate.
What systemic disease is most likely a cofactor in his periodontal disease?
ischemic heart disease;
diabetes mellitus;
Addison-Biermer;
menopause;
hyperacidity gastritis.
During exam of 14 year old patient a doctor reaveled the following symptoms: disorder of teeth
eruption, hypodontiya, abnormalities of enamel development, increasing tongue and lips, swollen
gums are pale. What pathology does have the listed above symptoms?
adrenal pathology;
liver pathology;
hyperfunction of pituitary;
thyroid hypofunction;
cancer of the mandible.
During oral exam a doctor reaveled an alveolar osteoporosis, expansion of periodontal fissure. The
patient treated in endocrinologist. What gland pathology does have the listed above symptoms?
parathyroid hypofunction
parathyroid hyperfunction;
hyperfunction of pituitary;
thyroid hypofunction;
pancreas hyperfunction.
During oral cavity exam of 54 year old patient, a doctor reaveled desquamative gingivitis. The patient
complains about burning tongue and lips. These symptoms indicate:
hyperacidity gastritis;
coronary heart disease;
menopause;
Addison’s disease;
diabetes mellitus.
A 34-year-old patient has developed alveolar bone resorption after long-term treatment of rheumatic
disease. The cause of this pathology is:
prolonged use of estrogens;
accession of secondary infection;
prolonged use of corticosteroids;
menopause;
disbacteriosis.
During oral exam a doctor reaveled general puffiness, cyanosis of the oral mucosa and red lip fringe,
dryness and swelling of mucouse membrane, tongue size is increased, on the buccal mucosa and
tongue – teeth prints. These symptoms indicate:
chronic hyperacidity gastritis;
cardiovascular insufficiency with blood circulation;
obesity;
Addison’s disease;
diabetes.
7.
A.
B. *
C.
D.
E.
8.
A.
B.
C. *
D.
E.
9.
A. *
B.
C.
D.
E.
10.
A.
B.
C.
D.
E. *
11.
A.
B.
C.
D.
E. *
12.
A. *
B.
C.
During exam of 40-year-old engineer there were identified multiple telangiectasias, angiomatous
growths on the nose, cheeks, lips, tongue lining and on the face, especially on the wings of the nose
and earlobes. Patient complains about frequent profuse bleeding from the nose and oral mucosa.
Platelet count bleeding time and blood's ability to coagulate are normally.These symptomth indicate:
haemophilia;
hemorrhagic anhiomatosis;
hypertension;
hemorrhagic diathesis;
Addison’s disease.
A 54 year old man arrived in the emergency department without consciousness. During examining of
patient a doctor noticed cyanosis of oral mucosa, bright red tongue, erosions, ulcers, hemorrhages in
the tongue tissues. These symptoms indicate:
hypoglycemic shock;
hyperglycemic shock;
large focal myocardial infarction;
stenocardia;
hypertensive crisis.
A patient is suffering from ischemic heart disease during 10 years. What is a color of mucous
membrane on the soft palate at this pathology?
bluish;
intense red;
yellowish;
pink;
spotty pink.
48 years old patient has a dysfunction of the gastrointestinal tract, blood forming and nervous
systems. Skin and oral mucous membrane are pale, with yellow shade, sometimes there are dotted
haemorrhage. Make a diagnosis.
leukopenia;
leukemia;
trombocytopenic purpura;
hypo avitaminosis C;
B-12 (folate-) deficiency anemia.
57 years old man complains on regarding weakness, burning sensation in the tongue, taste
disturbances. Objectively: skin and oral mucosa are pale, with a yellowish tinge. On the back of the
tongue tip there are bright red spots. The back of the tongue is smooth, shiny - "polished". Blood
analysis has a significant reduction of red blood cells number, a moderate decrease of hemoglobin
and high color index, besides anizo and poikilocytosis, availability megalocytes megaloblasts. Failure
of which factor in the body caused this disease:
ascorbic acid;
iron;
niacin;
riboflavin;
cobalamin.
The patient, 43 y.o., was diagnosed with pernicious anemia. What glossitis is one of the symptoms of
that anemia?
Henter-Miller glossitis
desquvamative
fissured
D.
E.
13.
A. *
B.
C.
D.
E.
14.
A.
B.
C.
D. *
E.
15.
A.
B.
C.
D. *
E.
16.
A.
B.
C.
D.
E. *
rhomboid
candidiasis of the tongue
Patient A., 26 y.o., complains of burning, dryness and pain in area of the mucous membrane of the
mouth, lips, tongue during eating. These symptoms are combined with weakness, rapid fatigue.
OBJECTIVELY: oral mucosa is pale, slightly moist, tongue is swollen, increased in dimensions,
papillaes are atrophied, especially at the tip of the tongue. Back of the tongue is bright red. Put the
diagnosis.
pernicious anemia, Addison-Biermer;
hypochromic iron-deficiency anemia;
desquamative glossitis;
rhomboid glossitis;
glossalgia.
41 years old man complains of pain and gums bleeding (especially during food taking), weakness,
and pain in the limbs, chilliness unwarranted. He lives in the North during last 15 years.
OBJECTIVELY: skin is dry, with brownish tinge, scaly, with symptom of “goose skin”. The mucous
membrane of the gums is swollen, hyperemic with cyanotic tinge; gingival papillae are
hypertrophied, covering the crowns of the teeth and bleeds after touching. Pathological tooth mobility
of 1st degree is present. In the mucosa of the soft palate numerous petechiae was finded. Deficiency
of which vitamin may cause this clinical picture:
vitamin E;
vitamin B12;
vitamin A;
vitamin C;
vitamin B.
A 25 year old patient complains about pain in the oral cavity induced by eating. He suffers from
CHD. Objective examination revealed dyspnea, limb edema. Oral cavity isn't sanitated. On the
mucous membrane, on the right, there is an ulcer with irregular edges covered with greyish-white
necrotic deposit with low-grade inflammation around it. There is also halitosis. What is the most
probable diagnosis?
Cancerous ulcer
Traumatic ulcer
Tuberculous ulcer
Trophic ulcer
Ulcero-necrotic stomatitis
A 26 year old homeless and unemployed patient complains about body temperature rise up to 39oC,
pain during eating and deglutition, nasal haemorrhages. He has been suffering from this for 10 days.
Objectively: herpetic rash on the lips, irregular-shaped erosions covered with fibrinous deposit on the
mucous membrane of oral cavity; filmy deposits on the tonsils. Liver is enlarged and sclerotic. Blood
count: erythrocytes - 4,5*1012l; hemoglobin - 120 g/l; ESR - 25 mm/h; leukocytes – 10*109l;
eosinophils - 0; rod nuclear cells - 2; segmentonuclear leukocytes - 31, lymphocytes - 41; monocytes
- 10; atypical mononuclears - 14%, plasmatic cells - 2. What is the most probable diagnosis?
Acute leukosis
Acute herpetic stomatitis
Oropharyngal diphtheria
AIDS
Infectious mononucleosis
17.
A. *
B.
C.
D.
E.
18.
A.
B.
C. *
D.
E.
19.
A.
B.
C.
D.
E. *
20.
A. *
B.
C.
D.
E.
21.
A.
B.
C.
D.
E. *
A 32-year-old male patient consults a dentist about an ulcer on the hard palate. It appeared about a
month ago. He has treated it by rinsing with herbal water, but the ulcer is gradually "creeping".
Objectively: there is a shallow erethistic ulcer with uneven and undermined edges of soft consistency
within the mucous membrane of hard palate. Granulations of the ulcer floor are also present.
Yellowish granules are visible on the ulcer periphery. What is the most likely diagnosis?
Tuberculous ulcer
Trophic ulcer
Cancerous ulcer
Syphilitic ulcer
Actinomycosis
A 42 year old patient applied to a dental clinic for oral cavity sanation. Anamnesis data: mild case of
diabetes. Objectively: mucous membrane of cheeks is unchanged, mainly in retromolar area there are
symmetrically placed whitish papulae protruding over the mucous membrane and forming a lace-like
pattern. On the upper jaw there are two soldered bridge dentures, the 47 tooth has an amalgam filling,
the 46 tooth has a steel crown. What is the most probable diagnosis?
Leukoplakia
Secondary syphilis
Lichen ruber planus
Chronic atrophic candidosis
Lupus erythematosus
A 51-year-old patient was diagnosed with leucoplakia nicotinica Tappeiner. What pathohistological
process predominates in histologic pattern of this disease?
Papillomatosis
Acanthosis
Dyskeratosis
Parakeratosis
Hyperkeratosis
A 17-year-old male patient complains about pain in the oral cavity, ulceration, body temperature up
to 38oC, headache. Objectively: mucous membrane of the oral cavity is hyperemic and edematic.
There are a lot of confluent erosions of polycyclic shape, covered with grey and white deposit,
located on hard palate, gums, lips. What is your provisional diagnosis?
Acute herpetic stomatitis
Erythema multiforme
Pemphigus vulgaris
Aphthous fever
Allergic stomatitis
An 21-year-old patient complains about body temperature rise, weakness, pain induced by eating and
deglutition. Objectively: mucous membrane of the oral cavity is erythematic with multiple petechia.
Pharynx is hyperaemic. Regional lymph nodes are enlarged, mobile, painless. In blood: leukocytosis,
monocytosis, atypic mononuclears, ESR is 30 mm/h. What is the leading factor of disease
Immediate allergy
development?
Bacterial infection
Autoimmune disorders
Delayed allergy
Viral infection
22.
A. *
B.
C.
D.
E.
23.
A.
B.
C.
D.
E. *
24.
A.
B.
C. *
D.
E.
25.
A.
B.
C. *
D.
E.
26.
A.
B.
C.
D. *
E.
A 26-year-old patient complains about a small ulcer on the red border of her lower lip that has been
irresponsive to self-treatment for two weeks. Objectively: unchanged red border of lower lip has a
circular ulcer of 2 mm in diameter with raised regular edges, its floor is of meat-like colour, dense,
shiny, with "stearic film", with cartilaginoid infiltration, painless on palpation. Regional lymph node
is enlarged, of tight elastic consistency, painless, mobile. What is the most likely diagnosis?
Primary syphilis
Decubital ulcer
Cancerous ulcer
Lupus erythematosus
Tuberculous ulcer
A 29-year-old male patient complains of dryness and burning of tongue back that appeared for about
a week ago and get worse when he eats irritating food. The patient has a history of recent pneumonia.
He had been treated in the in-patient hospital for 2 weeks, the treatment program included antibiotics.
Now he doesn't take any drugs. Objectively: mucous membrane of the oral cavity is hyperemic, dry,
glossy. Tongue back and palate have greyish-white plicae that can be easily removed. Threads of
saliva trail behind the spatula. What is the most likely diagnosis?
Chronic atrophic candidiasis
Chronic hyperplastic candidiasis
Acute atrophic candidiasis
Medicamental stomatitis
Acute pseudomembranous candidiasis
A 30-year-old patient complains about itch, burning and edema of lips. He has been suffering from
this for a week. Objectively: reddening of red border and skin, especially in the region of mouth
corners, there are also vesicles, crusts, small cracks along with erythematous affection of red border.
What is the most likely diagnosis?
Acute herpetic cheilitis
Multiform exudative erythema
Acute eczematous cheilitis
Allergic contact cheilitis
Exudative form of exfoliative cheilitis
A 18-year-old girl complains about having crusts, lip tenderness, especially at lip joining.
Objectively: there are yellow-brown crusts on the lip red border from Klein zone to it's middle, after
their removal bright red smooth surface without erosions appears. Mucous membrane in Klein zone
is slightly hyperemic and edematic. What is the most likely diagnosis?
Exudative form of cheilitis exfoliativa
Epidermolysis bullosa
Exudative form of cheilitis actinica
Meteorological cheilitis
Eczematous cheilitis
A 61-year-old female patient was waiting for her turn at the dentist’s. Suddenly she fell down, her
respiration became hoarse, she got convulsive twitching in her upper and lower limbs, face and neck
turned cyanotic, eye pupils became mydriatic, reaction of eye to light was absent. Arterial pressure
and pulse couldn’t be measured. Heart sounds couldn’t be auscultated. Involuntary urination was
noted. What condition is characterized by such symptoms?
Coma
Epilepsy
Shock
Clinical death
Collapse
27.
A.
B.
C. *
D.
E.
28.
A.
B.
C.
D.
E. *
29.
A.
B. *
C.
D.
E.
30.
A.
B.
C.
D.
E. *
31.
A.
B.
C. *
D.
E.
A 48-year-old female patient complains about sensation of tightness of buccal mucosa and roughness
of the lateral surface of tongue. The patient undergoes regular check-up at a dispensary department
for compensated form of diabetes mellitus. Objectively: there are white and grey areas in form of
lacy pattern on the buccal mucosa on the right and on the lateral surface of tongue. The surface of
affected region cannot be scraped off. What is the most likely diagnosis?
Lupus erythematosus
Secondary syphilis
Lichen ruber planus
Pseudomembranous candidiasis
Leukoplakia
A 29-year-old patient suffers from a disease without prodromal manifestations that declares itself
through oral mucosa lesion consisting of 1-2 roundish elements 5-8 mm large which are
circumscribed by a hyperemic rim and covered with yellow-grey coating. The disaese recurrence is
observed quite regularly 3-4 times a year. These presentations are typical for the following disease:
Papular syphilis
Lichen ruber planus
Chronic herpes recidivicus
Erythema multiforme
Chronic recurrent aphthous stomatitis
A 55-year-old patient complains about dryness and burning of the lateral surface of her tongue. These
sensations disappear during eating. She noted such sensations three months ago. She has a history of
gastritis with reduced secretory function. Objectively: mucous membrane of tongue and oral cavity
has no peculiarities. The back of tongue has thin white coating. Regional lymph nodes are
unpalpable. Oral cavity is sanitized. What is the most likely diagnosis?
Lingual nerve neuritis
Glossodynia
Candidiasis
Desquamative glossitis
Hunter-Moeller glossitis
A 33-year-old woman complains about experiencing lip dryness and desquamation for a month.
Application of indifferent ointments is ineffective. Objectively: red border of lower lip is of rich red
color, it is moderately infiltrated, covered with closely adhering greyish scales, it bleeds and hurts in
the attempt to remove them. Opacification of epithelium in form of white stripes is present in the
nidus periphery, there is also an area of depression in the centre. What is the most likely diagnosis?
Cheilitis exfoliativa
Commissural cheilitis
Lichen ruber planus
Leukoplakia
Lupus erythematosus
A 44-year-old patient complains about a sensation of foreign body on his tongue, discomfort during
talking, oral cavity dryness. Objectively: there are dark filiform papillae up to 5 mm long on the back
of tongue. What is the most likely diagnosis?
Benign migratory glossitis
Median rhomboid glossitis
Glossophytia
Fissured tongue
Acute glossitis
32.
A.
B.
C. *
D.
E.
33.
A. *
B.
C.
D.
E.
34.
A.
B.
C. *
D.
E.
35.
A. *
B.
C.
D.
E.
36.
A.
B. *
C.
D.
E.
A 21-year-old woman complains about severe pain in the mouth, body temperature up to 38oC,
indisposition. The same condition occurs periodically for several years after catching a cold.
Objectively: the lips are covered with bloody crusts, there are opened bladders and erosions, covered
with fibrinogenous deposit on mucous membrane of lips and cheeks that is apparently hyperemic and
edematic. Hypersalivation is present. What is the most likely diagnosis?
Dermatitits multiformis, Duhring's disease
Pemphigus vulgaris
Erythema multiforme
Nonacantholytic pemphigus
Chronic herpes recidivicus
A 32 year old woman complains about periodical appearance of small ulcers in the oral cavity. She
has been suffering from this for 5 years, recurrences happen 4-5 times a year. The ulcer healing lasts
for 10 days. Objectively: on a mucous membrane of lower lip there is a roundish lesion element 0,5
cm large covered with white deposit and surrounded by hyperemia border, very painful when
touched. What is the most probable diagnosis?
Chronic recurrent aphthous stomatitis
Traumatic erosion
Secondary syphilis
Chronic recurrent herpes
Duhring's herpetiform dermatitis
A 62 year old patient complains of pain in the area of mucous membrane of hard palate on the left
that is getting worse during eating with use of a complete removable denture. He has been suffering
from this for 1,5 month. Objectively: left-sided hyperemia and edema of mucous membrane of hard
palate; at the border of distal denture edge there is an ulcer with dense walls and fundus, surrounding
tissues are infiltrated. The ulcer floor is tuberous,covered with fibrinous deposit; ulcer palpation is
painful. What examination method is to be applied in the first place?
Cytology
Bacterioscopy
Biopsy
Allergic contact plastic test
Serological reactions
A 29-year-old patient complains about mouth soreness, body temperature rise up to 38,5oC,
indisposition. Such condition has been occurring periodically for several years after the patient had
had a cold. Objectively: lips are covered with haemorrhagic crusts, hyperaemic mucous membrane of
lips and cheeks has erosions covered with fibrinous films. Hypersalivation is present. What is the
most likely diagnosis?
Multiform exudative erythema
Pemphigus vulgaris
Herpes recidivicus
Herpetiform Duhring's dermatitis
Stevens-Johnson syndrome
A 30-year-old male patient complains about tongue pain that is getting worse during eating and
talking. Objectively: there is a painful ulcer 0,6 cm large on the lateral surface of tongue. The floor is
covered with grey deposit. The crown of the 47 tooth is destroyed. What is the most likely diagnosis?
Trophic ulcer
Decubital ulcer
Hard chancre
Tuberculous ulcer
Cancerous ulcer
37.
A.
B.
C.
D.
E. *
38.
A.
B.
C.
D. *
E.
39.
A.
B.
C.
D. *
E.
40.
A.
B. *
C.
D.
E.
41.
A.
B.
C. *
D.
E.
A patient complains of burning, itch and lower lip enlargement. He has been suffering from this for a
long time. Objectively: the patient's face is asymmetric due to the flattening of nasolabial fold. His
lower lip is edematic, of normal colour, painless on palpation. The patient has plicated tongue. What
is your provisional diagnosis?
Granulomatous Miescher's cheilitis
Quincke's edema
Lymphangioma
Hemangioma
Melkersson-Rosenthal syndrome
An 19 year old patient complains of gingival painfulness and haemorrhage, halitosis, temperature rise
up to 38,6oC, general weakness, appetite loss. Objectively: mucous membrane of oral cavity is
hyperemic and dry; tongue is covered with white fur, gingival papillae are edematic, their apices have
areas of necrotic deposit that can be easily removed leaving bleeding surface beneath. Submaxillary
lymph nodes are enlarged, palpatory painful. What is the causative agent of this disease?
Virus of herpes simplex
Streptostaphylococci
Pale treponema
Fusospirochetal symbiosis
Candida fungi
A 42-year-old patient complains about a rapidly growing formation on his lower lip. Examination of
the red border of lips revealed a greyish-red nodule with a hollow in the centre which is filled with
corneous masses that can be easily removed. The nodule is painless, mobile. What is your provisional
Basal
cell carcinoma
diagnosis?
Papilloma
Nodulous verrucous precancer of red border
Keratoacanthoma
Localized precancerous hyperkeratosis of red border
A young woman complained about having vesicles in oral cavity, painful ulcers, especially during
talking and eating; muscle and joint pain, body temperature rise, indisposition, weakness. She fell
suddenly ill 2 days ago. Objectively: to- 38,4oC. Vermilion border is covered with bloody crusts that
stick together and impede mouth opening. Mucous membrane of lips, cheeks, mouth floor, tongue,
soft palate is hyperemic and edematic; there are single vesicles and large painful erosions covered
with fibrinous deposit on it. Regional lymph nodes are enlarged and painful. There is hypersalivation.
Nikolsky's symptom is negative. What is the most probable diagnosis?
Acantholytic pemphigus
Multiform exudative erythema
Secondary syphilis
Acute herpetic stomatitis
Non-acantholytic pemphigus
A 27 y. o. steeplejack with a long record of service consults a dentist about dryness, burning and
insignificant lip edema. The same symptoms were noted one year ago in autumn. What is the most
likely diagnosis?
Cheilitis glandularis
Contact cheilitis
Meteorogical cheilitis
Manganotti's cheilitis
Cheilitis exfoliativa
42.
A.
B.
C.
D.
E. *
43.
A.
B.
C. *
D.
E.
44.
A.
B.
C.
D.
E. *
45.
A.
B. *
C.
D.
E.
46.
A.
B.
C. *
D.
E.
A 39 year old patient applied to a dentist and complained about white caseous deposit on the dorsum
of his tongue and burning sensation. It is known from the patient's anamnesis that he underwent
treatment in an in-patient hospital on account of pneumonia. What is the most probable diagnosis?
Lupus erythematosus
Typical form of leukoplakia
Lichen ruber planus
Scarlet fever
Acute pseudomembranous candidosis
A patient complained about frequent haemorrhages from the mucous membrane of oral and nasal
cavities, he mentioned also that his father had the same problems. Objectively: there are multiple
telangiectasias and angimatous formations on face skin as well as on mucous membrane of nose,
cheeks and lips. Blood count is normal. What is the most probable diagnosis?
Vaquez disease
Werlhof's disease
Rendu-Osler-Weber disease
Cushing's basophilism
Addison-Biermer disease
A 38-year-old builder complains about a condyloma on his lower lip. It appeared 1,5 month ago. It
has been significantly growing throughout the last week. Objectively: the red border of the lower lip
is cyanotic and infiltrated, it has some isolated closly adhering squamae. There is a well-defined
hemispherical formation 8 mm in diameter and 4 mm high in the centre. The formation is of
grey-and-blue-and-red colour, it has rough surface formed by thin, closely adhering and thick-based
squamae. Regional lymph nodes are enlarged, mobile, dense and painless. What is the most likely
Viral
wart
diagnosis?
Verruciform precancer
Pyogenic granuloma
Keratoacanthoma
Lower lip cancer
A patient complains about pain in the oral cavity, burning and dryness. Examination revealed
fiery-red dry mucous membrane. The tongue is crimson, dry, glossy, filiform papillae are atrophied.
There is some deposit in tongue folds that is hard to be removed. The patient undergoes treatment for
pneumonia, she takes antibiotics. What is the most likely diagnosis?
Fastened erythema
Acute atrophic candidiasis
Pellagrous glossitis
B2 hypovitaminosis
Benign migratory glossitis
Examination of a 26 year old patient revealed chronic candidosis of oral mucous membrane,
generalized lymphadenopathy. Anamnesis data: the patient has been suffering from herpes for a year.
Body temperature persistently rises up to 37,4-37,5oC, body weight has reduced by 8 kg over the last
month. What disease can be indicated by this symptom group?
Acute leukosis
Infectious mononucleosis
AIDS
Chronic leukosis
Candidosis
47.
A.
B.
C. *
D.
E.
48.
A.
B.
C. *
D.
E.
49.
A.
B.
C.
D. *
E.
50.
A. *
B.
C.
D.
E.
51.
A.
B.
C. *
D.
E.
A 23-year-old patient complains about acute pain in the mouth, headache, articular pain, body
temperature rise up to 38,6oC. Red border of lips is covered with haemorrhagic crusts, mucous
membrane of the oral cavity has big erosions and ulcers coated with greyish incrustation. Hand skin
exhibits erythematous spots 1-1,5 cm in diameter with a vesicle in the middle. What is the most likely
diagnosis?
Lyell's syndrome
Behcet's syndrome
Stevens-Johnson syndrome
Multiform exudative erythema
Medicamentous stomatitis
A 58-year-old patient complains of a painless formation on his tongue that appeared several months
ago. Objectively: the patient has a lot of carious and completely decayed teeth, lateral surface of
tongue exhibits a painless whitish formation 10x5 mm large with irregular surface in form of
verrucae. Histological examination revealed thickening of corneal epithelial layer of intermittent
keratinization type. What is the most likely diagnosis?
Hyperplastic form of candidiasis
Verrucous precancer
Verrucous form of leukoplakia
Hyperkeratotic form of lichen ruber planus
Keratoacanthoma
An 21-year-old patient complains about ulceration in the oral cavity, spontaneous bleeding of
mucous membrane, pain during food intake and talking, nosebleeds. He has a history of: aggravation
of general condition, weakness, body temperature rise up to 39oC, headache, joint pain. What method
of diagnostics should be applied to confirm the diagnosis?
HIV test
Blood sugar test
Immunogram
Clinical blood analysis
Allergy test
A 27 year old patient applied to a clinic for the purpose of oral cavity sanitation. During preparation
of the 45 tooth on account of chronic median caries the patient turned pale, there appeared cold
clammy sweat on her forehead, nausea, ear noise. The patient lost consciousness. Objectively: pulse 50 bpm, AP - 80/60 mm Hg, shallow breath, miotic pupils. Make a diagnosis of this state:
Syncope
Anaphylactic shock
Stenocardia
Collapse
Quincke's edema
A 28-year-old patient was delivered to a traumatology centre with a dirty cut wound of her right foot.
A doctor performed initial surgical d-bridement and made an injection of antitetanus serum. Some
time later the patient’s condition got abruptly worse: she developed extreme weakness, dizziness,
palpitation. Objectively: the skin is pale, the patient has cold sweat, frequent pulse of poor volume at
a rate of 100 bpm, AP is 90/40 mm Hg. What is the cause of such aggravation?
Pain shock
Haemorrhagic shock
Anaphylactic shock
Infectious-toxic shock
Drug disease (seroreaction)
52.
A.
B.
C.
D.
E. *
53.
A.
B.
C. *
D.
E.
54.
A.
B. *
C.
D.
E.
55.
A.
B. *
C.
D.
E.
56.
A.
B.
C.
D. *
E.
57.
A.
B. *
C.
D.
E.
58.
A. *
B.
C.
A 34-year-old man attended a glasshouse in a botanic garden. After he had smelt at an orchid he
turned pale and lost consciousness. Objectively: heart rate is 115/min, arterial pressure is 50/0 mm
Hg. What drug should be injected to the patient in the first place?
Dimedrol
Cordiamin
Strophanthine
Mesaton
Prednisolone
During patient examination doctor revealed white inclusions with the size of semolina grains or
millet grain on the vestibule surface of lower lip. What can it be?:
pink acne;
Mikulych aphthae;
Fordyce’s spots;
salivary gland;
chronic recurrent herpes.
During taking patient anamnesis was revealed that patient is smoker. What is the color of the soft
palate mucosa in smokers?
bluish;
deep red;
yellow;
pink;
spotted pink.
During patient examination was found small hemorrhages. They are:
ecchymosis;
petechiae;
telangiectasia;
roseola;
erythema.
During oral exam of 28 y.o. patient was found discoloration of the oral mucosa in a limited area
(diameter 6 mm) which is not above the mucous level. What kind of element damage is in the
patient?
blisters;
node;
nodule;
spot;
aphtha.
44-year-old patient was diagnosed with herpes infection in his lower lip. During examination a few
pustules was revealed. Pustule is:
small bubble with serous content, located on the mucosa;
little bubble with purulent content placed on the skin and lips red border up to 5 mm;
without cavity formation tends to suppurate, viral etiology;
without cavity formation with hemorrhagic content larger than 5 mm;
without cavity formation of purulent hemorrhagic content located intraepithelial.
During examination of 27-year-old painter cavernous element, filled with liquid was detected. What
is that?
blister;
crack;
aphthae;
D.
E.
59.
A.
B.
C. *
D.
E.
60.
A.
B.
C.
D. *
E.
61.
A. *
B.
C.
D.
E.
62.
A.
B.
C.
D.
E. *
63.
A. *
B.
C.
D.
E.
64.
A. *
B.
C.
D.
E.
erosion;
tubercle.
During patient examination doctor decided to use additional exam methods. Find the additional
examination method in the list.
palpation;
overview of the patient;
stomatoscopy;
life history;
complaints.
During examination of 29 y.o. patient symmetrical hillocks near 6th maxillary molars were detected.
What can it be?:
sweat glands;
Mikulych aphthae;
sebaceous glands;
parotid maxillary duct;
atsynoz glands.
25 years old man applied to a dental clinic for oral cavity sanation. Anamnesis data: mild case of
diabetes. Objectively: mucous membrane of cheeks is unchanged, mainly in retromolar area there are
symmetrically placed whitish papulae protruding over the mucous membrane and forming a lace-like
pattern. On the upper jaw there are two soldered bridge dentures, the 47 tooth has an amalgam filling,
the 46 tooth has a steel crown. What is the most probable
Lichen ruber planus
Secondary syphilis
Leukoplakia
Chronic atrophic candidosis
Lupus erythematosus
Roentgenological examination of a patient revealed a cyst in the area of a premolar that contained a
tooth in its cavity. Microscopical examination reveals that the cyst wall is represented by connective
tissue and lined with multistratal squamous cell epithelium. Specify the diagnosis:
Primordial cyst
Granuloma
Epulis
Follicular cyst
Radicular cyst
A patient is diagnosed with herpetic stomatitis provoked by immunosuppression. What medicine can
provide antiviral effect?
Acyclovir
Remantadinum
Amoxicillin
Methisazonum
Levamisole
A doctor needs to anaesthetize the anterior part of mucous membrane of hard palate. What nerves
should he block?
Nasopalatine nerves
Inferior alveolar nerves
Pharyngeal nerves
Zygomatic nerves
Suborbital nerves
65.
A.
B. *
C.
D.
E.
66.
A.
B.
C. *
D.
E.
67.
A. *
B.
C.
D.
E.
68.
A. *
B.
C.
D.
E.
69.
A. *
B.
C.
D.
E.
70.
A.
B.
C. *
D.
E.
Dentists widely apply local anaesthesia adding adrenalin to an anaesthetic solution. What is the
purpose of this method?
Local reduction of vascular resistance
Local vasoconstriction
Microcirculation improvement
Local vasodilatation
Lowering of arterial pressure
Being at a dentist a patient had an attack of stenocardia. What drug from the nitrate group should be
applied in this case?
Menthol
Iodine
Nitroglycerine
Hydrogen peroxide
Paracetamol
Soft palate arches were taken for bioptic examination because of suspected tumour (macroscopical
examination revealed an ulcer with dense floor). Biopsy revealed necrosis of mucous membrane
along with infiltration of submucous layer by lymphocytes, epithelioid cells, plasmatic cells, single
neutrophils. There is also evident endo- and perivasculitis. What disease are the described changes
typical for?
Ulcerative necrotic stomatitis (Vincent's stomatitis)
Aphthous stomatitis
Pharyngeal diphtheria
Primary syphilis
Ulcerative stomatitis
A doctor used novocaine as an anaesthetic during surgical manipulations. 10 minutes after the patient
became pale, he got dyspnea and hypotension. What type of allergic reaction is it?
Anaphylactic
Cell-mediated
Cytotoxic
Stimulating
Immune complex
Mucous membrane of a patient's oral cavity has a greyish-white focus, the mass is dense and
protrudes above the mucous membran E. Histological examination revealed hyperkeratosis,
parakeratosis and acanthosis of epithelium in this are A. What pathological process was revealed in
the mucous membrane?
Leukoplakia
Hyalinosis
Inflammation
Focal ichthyosis
Leukoderm
27 year old patient complains of gum bleeding, itching in oral cavity, angular cheilitis, teeth mobility.
It’s know from the anamnesis that last two years patient spent in Siberia. Deficiency of what vitamin
can cause this changes?
A
B1
C
D3
PP
71.
A.
B.
C. *
D.
E.
72.
A.
B.
C.
D. *
E.
73.
A.
B.
C. *
D.
E.
74.
A.
B.
C.
D.
E.
75.
A.
B.
C.
D. *
E.
76.
A. *
B.
C.
D.
E.
31 y.o. patient was diagnosed with scurvy. Deficiency of what vitamin can cause this changes?
A
B12
C
PP
B1
A 23 y.o. patient has been taking antibiotics on account of bronchopneumonia for a long time. There
appeared pain and burning in the area of mucous mebrane of his lips and tongue. Objectively:
mucous membrane of lips and tongue has caseous and white plaques that can be easily removed by a
spatula leaving hyperemia foci on their spot. Microscopical examination of the plaques revealed
mycelium. What is the most probable diagnosis?
Leukoplakia
Manganotti's cheilitis
Angular cheilitis
Candidous stomatitis
Contact allergic cheilitis
A 20 y.o. patient has been taking antibiotics on account of bronchopneumonia for a long time. There
appeared pain and burning in the area of mucous mebrane of his lips and tongue. Objectively:
mucous membrane of lips and tongue has caseous and white plaques that can be easily removed by a
spatula leaving hyperemia foci on their spot. Microscopical examination of the plaques revealed
mycelium.Patient was diagnosed with candidous stomatitis. What medicine should we use for its
treatment?
Antibiotics
Antihistamines
Antifungal
Antiseptics
Antiviral
25 year old female patient was diagnosed with meteorological cheilitis. What will be the treatment?
Antibiotics
Antiseptics
Ointments
Surgical treatment
Antiviral
19 year female patient complains on lip burning and redness which she noticed after new lipstic
using. What will be the diagnosis?
Angular cheilitis
Actinic cheilitis
Rhomboid glossitis
Allergic cheilitis
Exfoliative cheilitis
17 year female patient complains on lip burning and redness which she noticed after new lipstic
using. What will be the treatment in that case?
Antihistamine ointment
Antiviral medicine
Antiseptic medicine
No treatment is needed
Antibiotics for 7 days
77.
A.
B.
C.
D. *
E.
78.
A.
B. *
C.
D.
E.
79.
A. *
B.
C.
D.
E.
80.
A.
B. *
C.
D.
E.
81.
A.
B. *
C.
D.
E.
82.
A.
B. *
C.
D.
E.
During additional examination of 44 year old male patient was found positive Wassermann test.
What will be the diagnosis at that case?
AIDS
HIV
Tuberculosis
Syphilis
Gonorrhea
During additional examination of 24 year old student with oral mucous membrane pathology
Тreponema pallidum was revealed. What will be the diagnosis?
Leprosy
Syphilis
Gonorrhea
AIDS
Flu
24 year old patient was diagnosed with syphilis. What microorganism was found during
microbiological examination?
Тreponema pallidum
Candida albicans
Streptococcus mutans
Bacillus Kochii
Retrovirus
During editional examination of 27 years old patient Nikolskyy sign was used. What will be results
of positive Nikolskyy sign?
Purulent exudate can be revealed after blisters cutting
The outer epidermis separates easily from the basal layer on exertion of firm sliding manual pressure
The outer epidermis become pale after weak pressure
Gums are bleeding after weak pressure with probe
No correct answer
During editional examination of 31 years old patient was performed Nikolskyy sign, which was
negative. What will be results of positive Nikolskyy sign?
Purulent exudate can be revealed after blisters cutting
The outer epidermis separates easily from the basal layer on exertion of firm sliding manual pressure
The outer epidermis become pale after weak pressure
Gums are bleeding after weak pressure with probe
No correct answer
Patient complains about pain during swallowing, weakness, body temperature rise up to 39,6C,
swelling of submental lymph nodes. Objectively: high body temperature, mucous membrane of oral
cavity is brightly hyperaemic and edematic with haemorrhages and ulcerations. Pharynx is brightly
hyperemic, lacunae are enlarged and have necrosis areas. Regional, cervical, occipital lymph nodes
are painful, enlarged and dense. What is the most likely diagnosis?
Acute herpetic stomatitis
Infectious mononucleosis
Lacunar tonsillitis
Herpetic angina
Necrotizing ulcerative gingivostomatitis
83.
A.
B. *
C.
D.
E.
84.
A. *
B.
C.
D.
E.
85.
A.
B. *
C.
D.
E.
86.
A.
B.
C.
D. *
E.
87.
A.
B.
C.
D.
E. *
88.
A.
B.
A 65-year-old patient complains about pain in the oral cavity induced by eating. He suffers from
CH-Objective examination revealed dyspnea, limb edem-Oral cavity isn't sanitate-On the mucous
membrane on the right, there is an ulcer with irregular edges covered with greyish-white necrotic
deposit with low-grade inflammation around it. There is also halitosis. What is the most probable
Tuberculous
diagnosis? ulcer
Trophic ulcer
Cancerous ulcer
Traumatic ulcer
Ulcero-necrotic stomatitis
A 27-year-old male complains of pain, bad breath, body temperature rise up to 38,0o-Objectively: the
patient is pale, adynamic. Regional lymph nodes are enlarged and painful. Gums are edematic,
hyperaemic, ulcerated, covered with necrotic plaque. There is an excessive buildup of calculus. What
additional study should be done in the first place?
Complete blood count
X-ray of jaws
Test for HIV infection
Blood sugar test
Microscopy of gingival plaque
A 36-year-old patient complains of discoloration of the vermilion border of the lower lip that he
noticed about 4 months ago. Objectively: in the center of the vermilion border of the lower lip there
is an irregular homogeneous grayish-white area 1x1,5 cm large that doesn't rise above the vermilion
border and has distinct outlines. Palpation of this area is painless. The film cannot be removed when
scrapes. Upper incisors are covered with dentures. What is the most likely diagnosis?
Lupus erythematosus
Leukoplakia
Premalignant circumscribed hyperkeratosis
Lichen ruber planus
Candidous cheilitis
A 37-year-old female patient complains of inability to eat hot and cold food, as well as of intense
pain caused by sour, sweet and salty food. Objectively: Probing and cold test of all teeth cause acute
pain. What is the most likely diagnosis?
Enamel necrosis
Pathological abrasion of dental hard tissues
Enamel erosion
Hyperesthesia of dental hard tissues
Enamel hypoplasia
During taking of impressions (by gyps) patient presented with the following symptoms: paroxysmal
cough, cyanosis, vomiting, clapping sound during inspiration. Make the diagnosis:
Attack of bronchial asthma
Hypersensitive gag reflex
Tracheitis
Acute bronchitis
Aspiration of a foreign body
A patient complains of burning, itch and lower lip enlargement. He has been suffering from this for a
long time. Objectively: the patient's face is asymmetric due to the flattening of nasolabial fol-His
lower lip is edematic, of normal colour, painless on palpation. The patient has plicated (fissure)
tongue. What is your provisional diagnosis?
Hemangioma
Quincke's edema
C.
D.
E. *
89.
A. *
B.
C.
D.
E.
90.
A.
B.
C.
D.
E. *
91.
A.
B.
C. *
D.
E.
92.
A.
B.
C.
D.
E. *
93.
A. *
B.
C.
D.
E.
Granulomatous Miescher's cheilitis
Lymphangioma
Melkersson-Rosenthal syndrome
A 51-year-old female patient complains of dry mouth, taste impairment, burning and pricking
sensations in the tongue that disappear during eating but intensify at the end of the day. For the first
time such problems arose 2 years ago after a psychic traum-She has a history of anacid gastritis.
Objectively: the general condition is satisfactory, the patient is restless, tearful. Oral mucosa is pale
pink, dry; filiform papillae on the dorsum of tongue are normal. Pharyngeal reflex is dramatically
reduced. There is segmetal disturbance of facial skin sensation. What is the most likely diagnosis?
Glossodynia
Desquamative glossitis
Chronic atrophic candidous glossitis
Ganglionitis of sublingual ganglions
Hairy tongue
At a dentist's appointment a patient complained of weakness, nausea, blackout, and then he lost
consciousness. Make a diagnosis:
Collapse
Shock
Insult
Coma
Syncope
Analysis of the contents of periodontal pockets revealed a significant contamination with Candida
yeast fungi. Which of the following drugs should be used for instillations?
Biseptol
Diclofenac sodium
Clotrimazole
Paracetamol
Ibuprom
During the preparation of a tooth a patient had an epileptic attach. What was doctors mistake at that
case?
Skipped psychological preparation of the patient
Did not apply one of the types of local anesthesia
Violated the rules of preparation
Didn't apply general anaesthesia
Didn't collect complete history data
A 32-year-old patient complains of a neoplasm on the tip of the tongue which he hurts with his teeth.
The neoplasm sometimes increases, and sometimes decreases in size. Objectively: on the tip of
tongue there is a roundish neoplasm 0,5 cm in diameter with distinct borders and broad base and
looks like cauliflower. The neoplasm is the same colour as the mucosa of tongue. What is the most
likely diagnosis?
Papilloma of tongue
Chancre of the tongue
Abscess of tongue
Cancer of the tongue
Hairy tongue
94.
A.
B.
C.
D. *
E.
95.
A.
B.
C.
D.
E. *
96.
A.
B.
C.
D.
E. *
97.
A.
B.
C.
D.
E. *
98.
A.
B. *
C.
D.
E.
A 28-year-old teacher consulted a doctor about thirst, massive gingival haemorrhages, dry mouth,
mobility and shifting of teeth, purulent discharges from the gums, bad breath. According to the
patient, these presentations turned up about 2 months ago. Before the diagnosis can be made, the
following tests should be done in the first place:
Immunological studies
Acute-phase reactants tests
Allergological tests
Blood sugar test
Serologic studies
A 33-year-old male patient complains of dryness and burning of tongue back that appeared for about
a week ago and get worse when he eats irritating food. The patient has a history of recent pneumonia.
He had been treated in the in-patient hospital for 2 weeks, the treatment program included antibiotics.
Now he doesn't take any drugs. Objectively: mucous membrane of the oral cavity is hyperemic, dry,
glossy. Tongue back and palate have greyish-white plicae that can be easily removed. Threads of
saliva trail behind the spatul-What is the most likely diagnosis?
Chronic hyperplastic candidiasis
Acute stomatitis
Allergic stomatitis
Medicamental stomatitis
Acute pseudomembranous candidiasis
A patient working as a bricklayer complains of itching, burning, soreness of lips that show up only in
the summer period. He has been ill for 3 years. Objectively: vermilion border of the lower lip is
hyperemic, edematic, covered with blisters and painful erosions 2 mm in diameter, crusts, cracks.
What is the most likely diagnosis?
Angular cheilitis
Exfoliative cheilitis, exudative form
Contact allergic cheilitis
Eczematous cheilitis, exudative form
Actinic cheilitis, exudative form
A 17-year-old patient presents with weakness, pain in throat when swallowing, body temperature rise
up to 38,0 C. Examination of the the oral cavity revealed massive hyperaemia of the mucous
membrane of the soft palate, palatine arches, tonsils, uvula; there were also single vesicles and
erosions extremely painful when touched. Regional lymph nodes are enlarged, painful on palpation.
What is the most likely diagnosis?
Chickenpox
Infectious mononucleosis
Mycotic angina
Diphtheria
Herpangina
A 42-year-old male patient complains of fatigue and headache, limb numbness, dry mouth, burning
and pain in the tongue. Objectively: skin and oral mucosa are pale. There are painful cracks in the
corners of mouth. Dorsum of tongue is smooth, glossy, with bright red stria-In blood: Hb- 70 g/l,
RBCs - 1,5*1012/l, color index - 1,6, leukopenia, thrombocytopenia, lymphocytosis. What is the
most likely diagnosis?
Iron deficiency anemia
Addison-Biermer anemia
Chronic posthaemorrhagic anemia
Aplastic anemia
Late chlorosis
99.
A.
B.
C.
D.
E. *
100.
A. *
B.
C.
D.
E.
101.
A.
B. *
C.
D.
E.
102.
A.
B. *
C.
D.
E.
103.
A.
B. *
C.
D.
E.
A 38-year-old patient complains of a significant enlargement of the upper lip and eyelids, which
developed within a few minutes (during cleaning the house with a detergent). Objectively: there is an
edema of the upper part of face, upper lip and eyelids. Palpation is painless. What disease are these
symptoms typical for?
Melkersson-Rosenthal syndrome
Glandular cheilitis
Lymphedema
Macrocheilitis
Angioneurotic Quincke's edema
A 24-year-old patient complains of mouth soreness, body temperature rise up to 38,5oC,
indisposition. Such condition has occurred periodically for several years after the patient had had a
col-Objectively: lips are covered with haemorrhagic crusts, hyperaemic mucous membrane of lips
and cheeks has erosions covered with fibrinous films. Hypersalivation is present. What is the most
Multiform
exudative erythema
likely diagnosis?
Herpetiform Duhring's dermatitis
Stevens-Johnson syndrome
Pemphigus vulgaris
Herpes recidivicus
A 45-year-old patient complains of subfebrile temperature and a growing ulcer on the gingival
mucosa around the molars; looseness of teeth in the affected area, cough. Objectively: gingival
mucosa in the region of the lower left molars has two superficial, extremely painful ulcers with
undermined edges. The ulcers floor is yellowish, granular, covered with yellowish pink granulations.
The ulcers are surrounded by the tubercles. Dental cervices are exposed, there is a pathological tooth
mobility. Regional lymph nodes are enlarged and make dense matted together groups. What is the
Syphilis
most likely diagnosis?
Tuberculosis
Decubital ulcer
Infectious mononucleosis
Acute aphthous stomatitis
An 26-year-old musician complains about body temperature rise, weakness, pain induced by eating
and deglutition. Objectively: mucous membrane of the oral cavity is erythematic with multiple
petechies. Pharynx is hyperaemic. Regional lymph nodes are enlarged, mobile, painless. In blood:
leukocytosis, monocytosis, atypic mononuclear cells, ESR is 30 mm/h. What is the leading factor of
disease development?
Immediate allergy
Viral infection
Delayed allergy
Autoimmune disorders
Bacterial infection
Objective examination of a 19 years old girl revealed that her lower lip was slightly hyperemic,
infiltrated, dry, covered with small scales. Architectonics of lips is changed. Patient complains of
dryness and a feeling of tense lips, especially in autumn and winter. She also had a bad habit of lip
sucking. What is the most likely diagnosis?
Allergic cheilitis
Meteorological cheilitis
Microbial cheilitis
Atopic cheilitis
Exfoliative cheilitis
104.
A.
B.
C.
D.
E.
105.
A.
B.
C. *
D.
E.
106.
A.
B. *
C.
D.
E.
107.
A.
B.
C.
D. *
E.
108.
A.
B.
C.
D. *
E.
109.
A 32 year old manager complains about itch, burning and edema of lips. She has been suffering from
this for a week. Objectively: reddening of vermilion border and skin, especially in the region of
mouth corners, there are also vesicles, crusts, small cracks along with erythematous affection of
vermilion border. What is the most likely diagnosis?
Acute herpetic cheilitis
Exudative form of exfoliative cheilitis
Acute eczematous cheilitis
Fungal stomatitis
Multiform exudative erythema
Parents of an 9-year-old child complain about a painful formation in the child's oral cavity that
obstructs food intake. The same complaints were registered two years ago. Mucous membrane of
lateral tongue surface is hyperemic and edematic. There is an oval erosion over 0,7 cm large covered
with yellow greyish deposit. Erosion edges are hyperemic and painful on palpation. The child has a
history of chronic cholecystocholangitis. What is the most likely diagnosis?
Erythema multiforme
Stevens-Johnson syndrome
Chronic recurrent aphthous stomatitis
Traumatic erosion
Behcet's syndrome
During tooth extraction a 24-year-old patient presented with sudden weakness, pale skin, cold sweat,
weak pulse, a significant AP drop (diastolic pressure - 40 mm Hg). What complication developed in
the patient?
Anaphylactic shock
Collapse
Attack of stenocardia
Syncope
Traumatic shock
During the examinations and everyday orthopaedic manipulations a dentist uses a dental mirror.
What is the way of dental mirrors sterilization?
In the 6% hydrogen peroxide solution for 6 hours
In the 0,5% ethyl spiritus solution for 20 minutes
In the dry heat sterilizer at 180oC for 10 minutes
In the triple solution for 30 minutes
In the 0,01% chloramine solution for 2 minutes
A 43-year-old gallerist complains of pain under the dental bridge. After its removal the patient has
been found to have an ulcer 0,3x0,5 cm large on the alveolar process. The ulcer is slightly painful and
soft, the surrounding mucosa is hyperaemic, submandibular lymph nodes are not enlarged. What is a
provisional diagnosis?
Tuberculous ulcer
Trophic ulcer
Cancerous ulcer
Decubital ulcer
Sutton aphtha
A 70-year-old musician complains of pain in the lower jaw region on the right. He has been using
complete removable dentures for the upper and lower jaw for 12 years. He is smoker. Objectively:
the right retromolar region exhibits a 1,51,2 cm large proliferation of mucous membrane in form of
cauliflower, here and there there are dense fissured - like ulcers. The surrounding mucous membrane
is cyanotic, infiltrated. When the teeth are closed, the mentioned formation contacts with the
posterior edges of the dentures. What is the provisional diagnosis:
A. *
B.
C.
D.
E.
110.
A.
B.
C. *
D.
E.
111.
A.
B.
C.
D. *
E.
112.
A.
B.
C. *
D.
E.
113.
A.
B.
C.
D.
E. *
114.
A.
B.
C. *
D.
Cancer of the mucous membrane
Hypertrophic gingivitis
Decubital ulcer
Leukoplakia
Syphilis ulcer
A 20 year old girl complains about burning and painfulness of her tongue, especially during eating
spicy food. Objectively: there are oval red spots on the tip and dorsum of tongue. Filiform papillae
are not present in the affected area. The girl mentions that the spots become periodically larger and
have migratory nature. What is the most probable diagnosis?
Raspberry tongue
Median rhomboid glossitis
Glossitis areata exfoliativa
Candidosis
Lingua plicata (fissured tongue)
A 25 year old patient complaines about painfulness of his lower lip. He has been suffering for two
months. Objectively: mucous membrane of lower lip is hyperemic, excretory ducts of minor salivary
glands are dilated, "dew" symptom is present. What is the most probable diagnosis?
Contact allergic cheilitis
Meteorological cheilitis
Cheilitis exfoliativa
Cheilitis glandularis
Microbal cheilitis
During tooth extraction a 53 year old patient felt dull pain behind her breastbone, got a sense of
compression. A dental surgeon diagnosed her with a stenocardia attack. What medication should be
given this patient in order to arrest this state?
Analgin
Ketanov
Nitroglycerine, validol
Baralgin
Dimedrol
A 33 year old designer applied to a dental clinic for oral cavity sanation. Anamnesis data: mild case
of diabetes. Objectively: mucous membrane of cheeks is unchanged, mainly in retromolar area there
are symmetrically placed whitish papulae protruding over the mucous membrane and forming a
lace-like pattern. On the upper jaw there are two soldered bridge dentures, the 37 tooth has an
amalgam filling, the 36 tooth has a steel crown. What is the most probable diagnosis?
Leukoplakia
Secondary syphilis
Chronic atrophic candidosis
Lupus erythematosus
Lichen ruber planus
A 46 year old patient complained about painful ulcer on the lateral surface of his tongue. Objectively:
left lateral surface of tongue has a roundish ulcer with undermined soft overhanging edges, palpatory
painful, ulcer floor is slightly bleeding and covered with yellowish nodules. What is the most
probable diagnosis?
Syphilis
Traumatic ulcer
Tuberculosis
Actinomycosis
E.
115.
A.
B.
C.
D.
E. *
116.
A.
B.
C. *
D.
E.
117.
A.
B. *
C.
D.
E.
118.
A.
B.
C.
D. *
E.
119.
A. *
B.
C.
D.
Trophic ulcer
A 63 year old patient complains of pain in the area of mucous membrane of hard palate on the left
that is getting worse during eating with use of a complete removable denture. He has been suffering
from this for 1,5 month. Objectively: left-sided hyperemia and edema of mucous membrane of hard
palate; at the border of distal denture edge there is an ulcer with dense walls and fundus, surrounding
tissues are infiltrated. The ulcer floor is tuberous, covered with fibrinous deposit; ulcer palpation is
painful. What examination method is to be applied in the first place?
Allergic contact plastic test
Cytology
Serological reactions
Bacterioscopy
Biopsy
A 40 year old homeless and unemployed patient complains about body temperature rise up to 39oC,
pain during eating and deglutition, nasal haemorrhages. He has been suffering from this for 10 days.
Objectively: herpetic rash on the lips, irregular-shaped erosions covered with fibrinous deposit on the
mucous membrane of oral cavity; filmy deposits on the tonsils. Liver is enlarged and scleroti C.
Blood count: erythrocytes - 4,5*1012/l; hemoglobin - 120 g/l; ESR - 25 mm/h; leukocytes –
10*109/l; eosinophils - 0; rod nuclear cells - 2; segmentonuclear leukocytes - 31, lymphocytes - 41;
monocytes - 10; atypical mononuclears .What is the diagnosis?
Acute leukosis
Acute herpetic stomatitis
Infectious mononucleosis
AIDS
Oropharyngal diphtheria
A 29 year old woman complains about periodical appearance of small ulcers in the oral cavity. She
has been suffering from this for 5 years, recurrences happen 4-5 times a year. The ulcer healing lasts
for 10 days. Objectively: on a mucous membrane of lower lip there is a roundish lesion element 0,5
cm large covered with white deposit and surrounded by hyperemia border, very painful when
touched. What is the most probable diagnosis?
Secondary syphilis
Chronic recurrent aphthous stomatitis
Chronic recurrent herpes
Traumatic erosion
Duhring's herpetiform dermatitis
Examination of a 40 year old patient revealed chronic candidosis of oral mucous membrane,
generalized lymphadenopathy. Anamnesis data: the patient has been suffering from herpes for a year.
Body temperature persistently rises up to 37,4-37,5oC, body weight has reduced by 8 kg over the last
month. What disease can be indicated by this symptom group?
Infectious mononucleosis
Candidosis
Acute leukosis
AIDS
Chronic leukosis
A 24 year old patient applied to a dentist and complained about white caseous deposit on the dorsum
of his tongue and burning sensation. It is known from the patient's anamnesis that he underwent
treatment in an in-patient hospital on account of pneumoni A. What is the most probable diagnosis?
Acute pseudomembranous candidosis
Typical form of leukoplakia
Scarlet fever
Lichen ruber planus
E.
120.
A.
B.
C.
D. *
E.
121.
A.
B.
C.
D. *
E.
122.
A.
B.
C. *
D.
E.
123.
A.
B. *
C.
D.
E.
124.
A.
Lupus erythematosus
An 20 year old patient complains of gingival painfulness and haemorrhage, halitosis, temperature rise
up to 38,6oC, general weakness, appetite loss. Objectively: mucous membrane of oral cavity is
hyperemic and dry; tongue is covered with white fur, gingival papillae are edematic, their apices have
areas of necrotic deposit that can be easily removed leaving bleeding surface beneath. Submaxillary
lymph nodes are enlarged, palpatory painful. What is the causative agent of this disease?
Pale treponema
Candida fungi
Virus of herpes simplex
Fusospirochetal symbiosis
Streptostaphylococci
A young woman complained about having vesicles in oral cavity, painful ulcers, especially during
talking and eating; muscle and joint pain, body temperature rise, indisposition, weakness. She fell
suddenly ill 2 days ago. Objectively: to- 38,4oC. Vermilion border is covered with bloody crusts that
stick together and impede mouth opening. Mucous membrane of lips, cheeks, mouth floor, tongue,
soft palate is hyperemic and edematic; there are single vesicles and large painful erosions covered
with fibrinous deposit on it. Regional lymph nodes are enlarged and painful. There is hypersalivation.
Nikolsky's symptom is negative. What is the most probable diagnosis?
Acantholytic pemphigus
Non-acantholytic pemphigus
Acute herpetic stomatitis
Multiform exudative erythema
Secondary syphilis
A 34 year old patient applied to an oral surgeon for the purpose of oral cavity sanitation. After
anesthetization the patient felt sudden attack of nausea, weakness, he got the sense of compression
behind his breastbone, heart pain; he began vomiting. The patient lost consciousness, there appeared
convulsions. Objectively: the patient's skin is pale, covered with cold clammy sweat, pupils don't
react to the light. The pulse is thready, arterial pressure cannot be detected. What is the most probable
diagnosis?
Syncope
Anaphylactic shock
Collapse
Traumatic shock
Epileptic attack
A 26-years-old patient complains about a small ulcer on the red border of her lower lip that has been
irresponsive to self-treatment for two weeks. Objectively: unchanged red border of lower lip has a
circular ulcer of 2 mm in diameter with raised regular edges, its floor is of meat-like colour, dense,
shiny, with "stearic film", with cartilaginoid infiltration, painless on palpation. Regional lymph node
is enlarged, of tight elastic consistency, painless, mobil e. What is the most likely diagnosis?
Cancerous ulcer
Primary syphilis
Tuberculous ulcer
Decubital ulcer
Lupus erythematosus
An 17-year-old student complains of roughness and dryness of mucous membrane of the oral cavity.
He always bites uncontrolledly off mucous membrane along the line of teeth joining. The patient
suffers from chronic cholecystitis, he is smoker. Hygiene index is 2,3. Objectively: mucous
membrane along the line of teeth joining is quaggy, edematic, whitish. What is the leading factor of
this disease development?
Unsatisfactory level of oral cavity hygiene
B. *
C.
D.
E.
125.
A.
B. *
C.
D.
E.
126.
A. *
B.
C.
D.
E.
127.
A.
B.
C.
D. *
E.
128.
A. *
B.
C.
D.
E.
129.
A. *
B.
C.
Habitual biting of mucous membrane of the oral cavity
Xerostomia
Smoking
Somatic pathology
A 54-year-old patient complains about a rapidly growing formation on his lower lip. Examination of
the red border of lips revealed a greyish-red nodule with a hollow in the centre which is filled with
corneous masses that can be easily removed. The nodule is painless, mobil e. What is your
Nodulous
precancer of red border
provisionalverrucous
diagnosis?
Keratoacanthoma
Basal cell carcinoma
Localized precancerous hyperkeratosis of red border
Papilloma
Preventive examination of a 19-year-old schoolboy revealed unremovable grey-and-white layerings
on the mucous membrane of cheek along the line of teeth joining. Mucous membrane is slightly
hyperaemic, painless on palpation. The boy is emotionally unbalanced, bites his cheeks. What is the
most likely diagnosis?
Mild leukoplakia
Lichen ruber planus
Multiform exudative erythema
Chronic candidous stomatitis
Chronic recurrernt aphthous stomatitis
A 53-year-old patient complains about a nonhealing ulcer on his lower lip. The patient is smoker. He
hasn’t ever consulted a doctor about it. In the region of red border of the lower lip a roundish ulcer is
present. It is up to 2,0 cm in diameter. The ulcer edges are thickened and a little bit raised in form of
a whitish swelling. In the left submandibular region palpation revealed enlarged, painless, dense
lymph nodes with limited mobility. What is the most likely diagnosis?
Fibroma of the lower lip
Erosive verrucous leukoplakia
Syphilitic ulcer
Cancer of the lower lip
Keratoacanthoma
A 65-year-old female patient was waiting for her turn at the dentist’s. Suddenly she fell down, her
respiration became hoarse, she got convulsive twitching in her upper and lower limbs, face and neck
turned cyanotic, eye pupils became mydriatic, reaction of eye to light was absent. Arterial pressure
and pulse couldn’t be measured. Heart sounds couldn’t be auscultate d. Involuntary urination was
noted. What condition is characterized by such symptoms?
Clinical death
Collapse
Coma
Epilepsy
Shock
A 28-year-old patient complains about acute pain in the mouth, headache, articular pain, body
temperature rise up to 38,6oC. Red border of lips is covered with haemorrhagic crusts, mucous
membrane of the oral cavity has big erosions and ulcers coated with greyish incrustation. Hand skin
exhibits erythematous spots 1-1,5 cm in diameter with a vesicle in the middl e. What is the most
likely diagnosis?
Multiform exudative erythema
Lyell's syndrome
Medicamentous stomatitis
D.
E.
130.
A. *
B.
C.
D.
E.
131.
A.
B.
C. *
D.
E.
132.
A.
B.
C.
D. *
E.
133.
A.
B.
C.
D.
E. *
134.
A. *
B.
C.
D.
E.
Stevens-Johnson syndrome
Behcet's syndrome
An 19-year-old patient complains about body temperature rise, weakness, pain induced by eating and
deglutition. Objectively: mucous membrane of the oral cavity is erythematic with multiple petechi a.
Pharynx is hyperaemi c. Regional lymph nodes are enlarged, mobile, painless. In blood: leukocytosis,
monocytosis, atypic mononuclears, ESR is 30 mm/h. What is the leading factor of disease
Viral
infection
development?
Immediate allergy
Delayed allergy
Bacterial infection
Autoimmune disorders
A 26-year-old patient complains of heavy gingival haemorrhages, pain in the oral cavity, weakness,
fatigue, fever up to 38°C. These presentations appeared a week ago. Objectively: the patient is pale,
adynamic. Examination of the oral mucosa reveals multiple haemorrhages, friable gums, necrotic
areas on the tops of gingival papillae, as well as enlarged, soft, painless lymph nodes. The oral
mucosal lesion can be a symptom of the following disease:
Vincent stomatitis
Chronic leukemia
Acute leukemia
Hypovitaminosis C
Intoxication with heavy metal salts
A 25-year-old male complains of pain, bad breath, body temperature rise up to 38°C. Objectively: the
patient is pale, adynamic. Regional lymph nodes are enlarged and painful. Gums are edematic,
hyperaemic, ulcerated, covered with necrotic plaque. There is an excessive buildup of calculus. What
additional study should be done in the first place?
Blood sugar test
Test for HIV infection
X-ray of jaws
Complete blood count
Microscopy of gingival plaque
A 68-year-old patient complains of burning pain in the corners of mouth. Objectively: the folds of
mouth corners have erosions, covered with white coating that can be easily removed, mucous
membrane of mouth corners is macerated, white. There is pathological tooth wear and decreased
vertical dimension of occlusion. What is your provisional diagnosis?
Streptococcal angular cheilitis
Syphilitic angular cheilitis
Atopic cheilitis
Chronic recurrent labial fissure
Angular cheilitis candidiasis
26-year-old patient consulted a doctor about massive gingival haemorrhages, dry mouth, mobility
and shifting of teeth, purulent discharges from the gums, bad breath. According to the patient, these
presentations turned up about 2 months ago. Before the diagnosis can be made, the following tests
should be done in the first place:
Blood sugar test
Allergological tests
Serologic studies
Acute-phase reactants tests
Immunological studies
135.
A.
B.
C. *
D.
E.
136.
A.
B. *
C.
D.
E.
137.
A.
B.
C.
D.
E. *
138.
A.
B.
C. *
D.
E.
139.
A.
B.
C. *
D.
E.
A 34-year-old patient complains of gingival haemorrhages, fatigue, indisposition. The symptoms
have appeared quite recently. Objectively: the skin and oral mucosa are pale. Gums bleed when
touched. There are petechiae on the mucous membrane of cheeks, lips and palate. What laboratory
test will allow to make a diagnosis?
Blood test for Vitamin C
Blood test for sugar
Complete blood count
Bleeding time test
Immunogram
21-year-old patient complains of general weakness, fever, headache. These presentations appeared
three days ago. Objectively: the regional lymph nodes are enlarged, painful on palpationl, body
temperature is 37,8°C, oral mucosa is hyperemic and edematous. Mucous mebrane of lips, palate,
gums, cheeks presents single erosions of polycyclic irregular shape, covered with grayish-white film,
painful. Which drugs should be administered for the local treatment of the early disease?
Diet
Antiviral
Corticosteroid ointments
Keratoplastic agents
Antiseptics
A 30-year-old female patient complains of peeling, dryness and burning of the lower lip. Objectively:
on the vermilion border of lip there are multiple gray scales with tightly attached center and peeling
off edges. The scales are found all across the mouth and from the wet-dry line to the middle of the
lip. The skin is not affected. Scales removal doesnt lead to erosions. The patient has the lability of
psycho-emotional sphere. What is the most likely diagnosis?
Eczematous cheilitis
Actinic cheilitis
Exfoliative cheilitis, exudative form
Meteorological cheilitis
Exfoliative cheilitis, dry form
A 46-year-old patient complains about itch, burning and edema of lips. These presentations occured a
week ago. Objectively: there is reddening of vermilion border and skin, especially in the region of
mouth corners, there are also vesicles, crusts, small cracks along with erythematous affection of
vermilion border. What is the most likely diagnosis?
Acute herpetic cheilitis
Multiform exudative erythema
Acute eczematous cheilitis
Allergic contact cheilitis
Exudative form of exfoliative cheilitis
A 45-year-old male patient complains of fatigue and headache, limb numbness, dry mouth, burning
and pain in the tongue. Objectively: skin and oral mucosa are pale. There are painful cracks in the
corners of mouth. Dorsum of tongue is smooth, glossy, with bright red striae. Blood count: Hb - 70
g/l, RBCs - 1,5x1012/l, colour index - 1,6, leukopenia, thrombocytopenia, lymphocytosis. What is the
most likely diagnosis?
Late chlorosis
Chronic posthaemorrhagic anemia
Addison-Biermer anemia
Iron deficiency anemia
Aplastic anemia
140.
A.
B.
C. *
D.
E.
141.
A.
B.
C. *
D.
E.
142.
A.
B. *
C.
D.
E.
143.
A. *
B.
C.
D.
E.
144.
A. *
B.
C.
D.
E.
25 female patient complains about having crusts, lip tenderness, especially at lip joining. Objectively:
there are yellow-brown crusts on the lip red border from Klein zone to its middle, after their removal
bright red smooth surface without erosions appears. Mucous membrane in Klein zone is slightly
hyperemic and edematic. What is the most likely diagnosis?
Epidermolysis bullosa
Exudative form of cheilitis actinica
Exudative form of cheilitis exfoliativa
Meteorological cheilitis
Eczematous cheilitis
A 29 y.o. male patient complains of a painless nonhealing mouth ulcer. Objectively: regional lymph
nodes are enlarged, painless, of cartilaginous consistency. On the buccal mucosa there is a round
ulcer 1 cm in diameter with regular raised edges and a dense elastic infiltrate at the base. The ulcer
surface is of red colour, painless on palpation. What is the most likely diagnosis?
Secondary tuberculosis
Primary tuberculosis
Primary syphilis
Cancer
Secondary syphilis
A 53-year-old patient complains of a painful ulcer in the mouth that is getting bigger and does not
heal over 1,5 months. Objectively: on the buccal mucosa there is a shallow soft ulcer 2 cm in
diameter with irregular undermined edges. The ulcer floor is uneven and covered with yellow-gray
coating. The ulcer is surrounded by many small yellowish tubercles. Regional lymph nodes are
elastic, painful, matted together. Which disease is characterized by such symptoms?
Ulcerative necrotizing stomatitis
Tuberculosis
Lichen planus
Syphilis
Cancer
A 31 y.o. patient complains of general weakness, pain in the gums, halitosis. Objectively: the patient
is pale, adynamic, body temperature is 38,5°C, submandibular lymph nodes are enlarged, painful on
palpation. Interdental gingival papillae are inflamed, their tops are "detruncated", covered with
gray-yellow necrotic incrustation. Radiography of alveolar process and blood count reveal no
apparent changes. What is the most likely diagnosis?
Acute ulcerous gingivitis
Acute leukosis
Localized periodontitis
Agranulocytosis
Generalized periodontitis
A 62-year-old patient complains of burning tongue, a metallic taste in mouth. Three months ago she
got a dental bridge made of gold and supported by the 16, 14 teeth. Oral cavity exasmination reveals
no objective changes. The 36, 37, 46 teeth are sealed with amalgam fillings. What is the most likely
cause of this condition?
Galvanic currents
Neurologic disorder
Mechanic trauma
Chemical factors
Allergy
145.
A.
B.
C.
D. *
E.
146.
A.
B.
C.
D. *
E.
147.
A.
B.
C. *
D.
E.
148.
A. *
B.
C.
D.
E.
149.
A.
B. *
C.
D.
E.
150.
A.
A 31-year-old patient complains of general weakness, spasmodic neuralgic pain in the right side of
face, rash in the mouth and on the skin. Objectively: lip and chin skin is markedly hyperemic, there
are numerous vesicles with clear exudate on the right. The right cheek mucosa is hyperemic, there is
a string of erosions covered with fibrinous pellicle. What is the provisional diagnosis?
Acute herpetic stomatitis
Allergic stomatitis
Aphthous fever
Herpes zoster
Pemphigus vulgaris
A 23-year-old patient complains of roughness and dryness of mucous membrane of the oral cavity.
He always bites uncontrolledly off mucous membrane along the line of teeth joining. The patient
suffers from chronic cholecystitis, he is smoker. Hygiene index is 2,3. Objectively: mucous
membrane along the line of teeth joining is quaggy, edematic, whitish. What is the leading factor of
this disease development?
Unsatisfactory level of oral cavity hygiene
Xerostomia
Smoking
Habitual biting of mucous membrane of the oral cavity
Somatic pathology
A chemical plant worker consulted a dentist about an oral mucosa burn caused by caustic soda.
Which of the following medications should be chosen for emergency care?
0,1% liquid ammonia
50% ethyl alcohol
0,5% acetic acid solution
3% sodium chloride solution
2% citric acid solution
A 19-year-old student suffers from a disease without prodromal manifestations that declares itself
through oral mucosa lesion consisting of 1-2 roundish elements 5-8 mm large which are
circumscribed by a hyperemic rim and covered with yellow-grey coating. The disaese recurrence is
observed quite regularly 3-4 times a year. These presentations are typical for the following disease:
Chronic recurrent aphthous stomatitis
Chronic herpes recidivicus
Papular syphilis
Erythema multiforme
Lichen ruber planus
A 57-year-old female patient complains about dryness and burning of the lateral surface of her
tongue. These sensations disappear during eating. She noted such sensations three months ago. She
has a history of gastritis with reduced secretory function. Objectively: mucous membrane of tongue
and oral cavity has no peculiarities. The back of tongue has thin white coating. Regional lymph nodes
are unpalpable. Oral cavity is sanitized. What is the most likely diagnosis?
Hunter-Moeller glossitis
Glossodynia
Candidiasis
Lingual nerve neuritis
Desquamative glossitis
A 46-year-old patient complains about a sensation of foreign body on his tongue, discomfort during
talking, oral cavity dryness. Objectively: there are dark filiform papillae up to 5 mm long on the back
of tongue. What is the most likely diagnosis?
Acute glossitis
B. *
C.
D.
E.
151.
A.
B.
C.
D.
E. *
152.
A.
B.
C. *
D.
E.
153.
A.
B. *
C.
D.
E.
154.
A.
B. *
C.
D.
E.
155.
Glossophytia (hairy tongue)
Benign migratory glossitis
Median rhomboid glossitis
Fissured tongue
33-year-old woman complains about severe pain in the mouth, body temperature up to 38°C,
indisposition. The same condition occurs periodically for several years after catching a cold.
Objectively: the lips are covered with bloody crusts, there are opened bladders and erosions, covered
with fibrinogenous deposit on mucous membrane of lips and cheeks that is apparently hyperemic and
edematic. Hypersalivation is present. What is the most likely diagnosis?
Nonacantholytic pemphigus
Chronic herpes recidivicus
Flu
Dermatitits multiformis, Duhrings disease
Erythema multiforme
28 year old driver complains of acute pain in his mouth and sore throat, difficult ingestion,
indisposition, temperature rise up to 38,6°C. These symptoms appeared after exposure to cold. He
has been ill for a day, in the evening he has to go to work. Objectively: gums in the area of inferior
frontal teeth as well as pharynx mucous membrane are hyperemic, edematic; gingival edge is
necrotic, tonsills are enlarged. Results of bacterioscopy: fusospirochetal symbiosis. In blood:
erythrocytes - 4,5x1012/l; leukocytes - 7,2x109/l; ESR - 18 mm/h. What actions should a
To
give him antake?
order for hospitalization in oral department
stomatologist
To prescribe a medication and allow to go to work
To prescribe a medication and give a sick-list
To refer him to an infectious disease specialist
To refer him to an otolaryngologist
71 year old patient complains of pain in the area of mucous membrane of hard palate on the left that
is getting worse during eating with use of a complete removable denture. He has been suffering from
this for 1,5 month. Objectively: left-sided hyperemia and edema of mucous membrane of hard palate;
at the border of distal denture edge there is an ulcer with dense walls and fundus, surrounding tissues
are infiltrated. The ulcer floor is tuberous, covered with fibrinous deposit; ulcer palpation is painful.
What examination method is to be applied in the first place?
Serological reactions
Biopsy
Blood test
Bacterioscopy
Allergic contact plastic test
A 35-year-old writer patient complains of mouth soreness, body temperature rise up to 38,5°C,
indisposition. Such condition has occurred periodically for several years after the patient had had a
cold. Objectively: lips are covered with haemorrhagic crusts, hyperaemic mucous membrane of lips
and cheeks has erosions covered with fibrinous films. Hypersalivation is present. What is the most
likely diagnosis?
Stevens-Johnson syndrome
Multiform exudative erythema
Herpes recidivicus
Pemphigus vulgaris
Herpetiform Duhrings dermatitis
Preventive examination of a 55-year-old man revealed a well-defined area of opaque mucosal
epithelium of the left cheek that didnt protrude above the surrounding tissues and could not be
removed on scraping. Crowns of the 34, 35, 36 teeth were strongly decayed and had sharp edges.
What is the most likely diagnosis?
A.
B.
C.
D. *
E.
156.
A.
B.
C. *
D.
E.
157.
A.
B.
C. *
D.
E.
158.
A.
B. *
C.
D.
E.
159.
A.
B.
C. *
D.
E.
160.
A. *
B.
C.
D.
Lichen ruber planus
Candidiasis
Soft cancer
Leukoplakia
Lupus erythematosus
A 53-year-old male patient complains about tongue pain that is getting worse during eating and
talking. Objectively: there is a painful ulcer 0,6 cm large on the lateral surface of tongue. The floor is
covered with grey deposit. The crown of the 47 tooth is destroyed. What is the most likely diagnosis?
Hard chancre
Trophic ulcer
Decubital ulcer
Tuberculous ulcer
Cancerous ulcer
A 51-year-old male patient complains of dry mouth, taste impairment, burning and pricking
sensations in the tongue that disappear during eating but intensify at the end of the day. For the first
time such problems arose 2 years ago after a psychic trauma. He also has a history of anacid gastritis.
There is segmetal disturbance of facial skin sensation. What is the most likely diagnosis?
Ganglionitis of sublingual ganglions
Moller-Hunter glossitis
Glossodynia
Chronic atrophic candidous glossitis
Desquamative glossitis
A young woman complained about having vesicles in oral cavity, painful ulcers, especially during
talking and eating; muscle and joint pain, body temperature rise, indisposition, weakness. She fell
suddenly ill 2 days ago. Objectively: to- 38,4°C. Vermilion border is covered with bloody crusts that
stick together and impede mouth opening. Mucous membrane of lips, cheeks, mouth floor, tongue,
soft palate is hyperemic and edematic; there are single vesicles and large painful erosions covered
with fibrinous deposit on it. Regional lymph nodes are enlarged and painful. There is hypersalivation.
Nikolskys symptom is negative. What is the most probable diagnosis?
Non-acantholytic pemphigus
Multiform exudative erythema
Secondary syphilis
Acantholytic pemphigus
Acute herpetic stomatitis
A 33 year old policeman applied to a dentist and complained about white caseous deposit on the
dorsum of his tongue and burning sensation. It is known from the patients anamnesis that he
underwent treatment in an in-patient hospital on account of pneumonia. What is the most probable
Lichen
ruber planus
diagnosis?
Typical form of leukoplakia
Acute pseudomembranous candidosis
Scarlet fever
Lupus erythematosus
Unused stomatological instruments were left on a sterile table at the end of the working day. What
measures should be taken in order to provide sterility of these instruments?
Sterilization without preliminary processing
Disinfection, presterilization treatment, sterilization
Disinfection only
Presterilization treatment, sterilization
E.
161.
A.
B.
C. *
D.
E.
162.
A.
B. *
C.
D.
E.
163.
A.
B. *
C.
D.
E.
164.
A.
B.
C.
D.
E. *
165.
A. *
B.
C.
D.
E.
Disinfection, sterilization
A 51-year-old patient complains about a condyloma on his lower lip. It appeared 1,5 month ago. It
has been significantly growing throughout the last week. Objectively: the red border of the lower lip
is cyanotic and infiltrated, it has some isolated closly adhering squamae. There is a well-defined
hemispherical formation 8 mm in diameter and 4 mm high in the centre. The formation is of
grey-and-blue-and-red colour, it has rough surface formed by thin, closely adhering and thick-based
squamae. Regional lymph nodes are enlarged, mobile, dense and painless. What is the most likely
Pyogenic
diagnosis?granuloma
Verruciform precancer
Lower lip cancer
Keratoacanthoma
Viral wart
44-year-old patient complains of a painless formation on his tongue that appeared several months
ago. Objectively: the patient has a lot of carious and completely decayed teeth, lateral surface of
tongue exhibits a painless whitish formation 10x5 mm large with irregular surface in form of
verrucae. Histological examination revealed thickening of corneal epithelial layer of intermittent
keratinization type. What is the most likely diagnosis?
Keratoacanthoma
Verrucous form of leukoplakia
Hyperplastic form of candidiasis
Verrucous precancer
Hyperkeratotic form of lichen ruber planus
A patient being at a dentist presented suddenly with tachycardia, sweating, trembling, nausea,
hunger. The patient is conscious. He suffers from diabetes mellitus. What aid should be rendered by
the dentist?
The
dentist should make intravenous injection of corticosteroids
The dentist should break the procedure off and give the patient sugar in any form
The dentist should give the patient nitroglycerine in form of spray or sublingually
The dentist should lay the patient down on a flat surface, lower cephalic pole, open the windows
The dentist should make intramuscular injection of adrenaline (0,5-1,0 mg)
An 19-year-old patient complains about ulceration in the oral cavity, spontaneous bleeding of
mucous membrane, pain during food intake and talking, nosebleeds. He has a history of: aggravation
of general condition, weakness, body temperature rise up to 39°C, headache, joint pain. What method
of diagnostics should be applied to confirm the diagnosis?
HIV test
Allergy test
Blood sugar test
Immunogram
Clinical blood analysis
A 24 year old female patient applied to a clinic for the purpose of oral cavity sanitation. During
preparation of the 45 tooth on account of chronic median caries the patient turned pale, there
appeared cold clammy sweat on her forehead, nausea, ear noise. The patient lost consciousness.
Objectively: pulse - 50 bpm, AP - 80/60 mm Hg, shallow breath, miotic pupils. Make a diagnosis of
Syncope
this state:
Stenocardia
Quinckes edema
Collapse
Anaphylactic shock
166.
A.
B. *
C.
D.
E.
167.
A. *
B.
C.
D.
E.
168.
A.
B.
C. *
D.
E.
169.
A.
B. *
C.
D.
E.
170.
A.
B.
C. *
D.
E.
Rising from a chair after a dental procedure, a 67-year-old patient felt retrosternal pain radiating to
the left arm, nausea, weakness. Objectively: the patient is pale, his face is sweaty. AP is 90/60 mm
Hg, pulse is arrhythmic, of poor volume, 100/min. Heart sounds are muffled, vesicular breathing is
present. What is your provisional diagnosis?
Pulmonary artery thrombosis
An attack of coronary artery disease
Hypertensic crisis
Myocardial infarction
Stenocardia
A man was found in a street in unconscious state and delivered to the sanitary inspection room. At
small intervals the man presents with recurrent attacks of tonic and clonic convulsions of the upper
and lower limbs. Objectively: the patient is unconscious, his pupils are mydriatic, don’t react to light.
The tongue has teeth indentations. Involuntary urination was noted. Examination revealed no
symptoms of focal cerebral lesion. AP is 140/90 mm Hg. The patients head can be freely brought to
the chest. Specify the most probable pathology:
Epilepsy, status epilepticus
Cerebral infraction
Acute meningoencephalitis
Tetanus
Parenchymatous hemorrhage
During transportation of a 63-year-old patient an emergency doctor noted aggravation of his general
condition, pulselessness, mydriatic pupils, absence of respiratory movements. It will take 5 minutes
to get to the admission ward. What actions should the doctor take in the first place?
Make an intracardiac injection of adrenaline
Deliver the patient to the hospital as soon as possible
Start cardiopulmonary resuscitation
Facilitate oxygen inhalation
Make an intravenous injection of polyglucinum
26 year old patient complains about pain during swallowing, weakness, body temperature rise up to
39,5oC, swelling of submental lymph nodes. Objectively: high body temperature, mucous membrane
of oral cavity is brightly hyperaemic and edematic with haemorrhages and ulcerations. Pharynx is
brightly hyperemic, lacunae are enlarged and have necrosis areas. Regional, cervical, occipital lymph
nodes are painful, enlarged and dense. What is the most likely diagnosis?
Acute herpetic stomatitis
Infectious mononucleosis
Lacunar tonsillitis
Herpetic angina
Necrotizing ulcerative gingivostomatitis
A 25-year-old patient complains about sensation of tightness of buccal mucosa and roughness of the
lateral surface of tongue. The patient undergoes regular check-up at a dispensary department for
compensated form of diabetes mellitus. Objectively: there are white and grey areas in form of lacy
pattern on the buccal mucosa on the right and on the lateral surface of tongue. The surface of affected
region cannot be scraped off. What is the most likely diagnosis?
Stomatitis
Secondary syphilis
Lichen ruber planus
Pseudomembranous candidiasis
Leukoplakia
171.
A.
B.
C. *
D.
E.
172.
A.
B. *
C.
D.
E.
173.
A.
B. *
C.
D.
E.
174.
A.
B.
C.
D.
E. *
175.
A.
B.
C.
D.
E. *
176.
A.
B.
C. *
A 64 year old patient arrived to the dentist for caries treatment. After anesthetization the patient felt
sudden attack of nausea, weakness, he got the sense of compression behind his breastbone, heart
pain; he began vomiting. The patient lost consciousness, there appeared convulsions. Objectively: the
patient's skin is pale, covered with cold clammy sweat, pupils don't react to the light. The pulse is
thready, arterial pressure cannot be detected. What is the most probable diagnosis?
Syncope
Anaphylactic shock
Collapse
Traumatic shock
Epileptic attack
A 54-year-old male patient complains about pain in the oral cavity induced by eating. He suffers
from chronic heart disease. Oral cavity isn't sanitated. On the mucous membrane on the right, there is
an ulcer with irregular edges covered with greyish-white necrotic deposit with low-grade
inflammation around it. There is also halitosis. What is the most probable diagnosis?
Tuberculous ulcer
Trophic ulcer
Cancerous ulcer
Traumatic ulcer
Ulcero-necrotic stomatitis
A 37-year-old patient complains of discoloration of the vermilion border of the lower lip that he
noticed about 3 months ago. Objectively: in the center of the vermilion border of the lower lip there
is an irregular homogeneous grayish-white area 1x1,5 cm large that doesn't rise above the vermilion
border and has distinct outlines. Palpation of this area is painless. The film cannot be removed when
scrapes. Upper incisors are covered with braces (2 year). What is the most likely diagnosis?
Lupus erythematosus
Leukoplakia
Premalignant circumscribed hyperkeratosis
Lichen ruber planus
Candidous cheilitis
Before anestetic injection, in dental clinic, patient complained of weakness, nausea, blackout, and
then he lost consciousness. Make a diagnosis:
Collapse
Shock
Insult
Coma
Syncope
Before anestetic injection, in dental clinic, patient complained of weakness, nausea, blackout, and
then he lost consciousness. Patient was diagnosed with syncope. With what other condition should
we differentiate it?
Collapse
Shock
Insult
No correct answer
Answers A and B are correct
Before anestetic injection, in dental clinic, patient complained of weakness, nausea, blackout, and
then he lost consciousness. Patient was diagnosed with syncope. What should doctor do at that case?
Inject insuline
Inject adrenaline 0,5ml IV
Give patient some air and to smell medicine with strong odour
D.
E.
177.
A.
B.
C. *
D.
E.
178.
A.
B.
C.
D.
E. *
179.
A.
B.
C.
D.
E. *
180.
A.
B.
C. *
D.
E.
181.
A.
B.
C.
D.
E. *
To pour glass of water in patient’s face
To give patient a nitroglicerine
Analysis of the contents of periodontal pockets of 21 year old patient revealed a significant
contamination with Candida yeast fungi. Which of the following drugs should be used for
instillations?
Biseptol
Diclofenac sodium
Clotrimazole
Paracetamol
Ibuprom
During the preparation of a tooth becouse of caries treatment, 23 y.o. patient had an epileptic attach.
What was doctor mistake at that case?
Skipped psychological preparation of the patient
Did not apply one of the types of local anesthesia
Violated the rules of preparation
Didn't apply general anaesthesia
Didn't collect complete history data
A 31-year-old male patient complains of dryness and burning of tongue back that appeared for about
a week ago. The patient has a history of recent pneumonia. Objectively: mucous membrane of the
oral cavity is hyperemic, dry, glossy. Tongue back and palate have white plicae that can be easily
removed. Threads of saliva trail behind the spatule.What is the most likely diagnosis?
Chronic hyperplastic candidiasis
Acute stomatitis
Allergic stomatitis
Medicamental stomatitis
Acute pseudomembranous candidiasis
A 31-year-old male patient complains of dryness and burning of tongue back that appeared for about
a week ago. The patient has a history of recent pneumonia. Objectively: mucous membrane of the
oral cavity is hyperemic, dry, glossy. Tongue back and palate have white plicae that can be easily
removed. Threads of saliva trail behind the spatule.Patient was diagnosed with acute
pseudomembranous candidiasis. What will be the key medicine in the treatment?
Preccribe antibiotics again.
Oral washing with antiseptics
Antifungal medicine
Corticosteroids
Antihistamines
A 26-year-old male patient presents with weakness, pain in throat when swallowing, body
temperature rise up to 38,0 C. Examination of the the oral cavity revealed massive hyperaemia of the
mucous membrane of the soft palate, palatine arches, tonsils, uvula; there were also single vesicles
and erosions extremely painful when touched. Regional lymph nodes are enlarged, painful on
palpation. What is the most likely diagnosis?
Chickenpox
Infectious mononucleosis
Mycotic angina
Diphtheria
Herpangina
182.
A.
B.
C.
D.
E.
183.
A.
B.
C.
D.
E. *
184.
A. *
B.
C.
D.
E.
185.
A. *
B.
C.
D.
E.
186.
A.
B. *
C.
D.
E.
A 31-year-old male patient complains of dryness and burning of tongue back that appeared for about
a week ago. The patient has a history of recent pneumonia. Objectively: mucous membrane of the
oral cavity is hyperemic, dry, glossy. Tongue back and palate have white plicae that can be easily
removed. Threads of saliva trail behind the spatule. Patient was diagnosed with acute
pseudomembranous candidiasis. What diseases should we differentiate candidiasis with?
Leukoplakia
Necrotic gingivitis
Lichen planus
No correct answer
Answers A, B, C are correct
A 25 y.o. patient, complains of a significant enlargement of the upper lip and eyelids, which
developed within a few minutes after bee sticking. Objectively: there is an edema of the upper part of
face, upper lip and eyelids. Palpation is painless. What disease are these symptoms typical for?
Melkersson-Rosenthal syndrome
Glandular cheilitis
Lymphedema
Macrocheilitis
Angioneurotic Quincke's edema
A 26 y.o. patient, complains of a significant enlargement of the upper lip and eyelids, which
developed within a few minutes after bee sticking. Objectively: there is an edema of the upper part of
face, upper lip and eyelids. Palpation is painless. Patient was diagnosed with Quincke's edema. What
will be the treatment at that case?
Inject adrenaline, corticosteroids and antihistamines
Prescribe antihistamines in form of pills
Prescribe antibiotics
Prescribe antiinflammatory drugs and analgetigs
Put a cold on the lip
A 45-year-old patient complains of mouth soreness, body temperature rise up to 38,5oC,
indisposition. Such condition has occurred periodically for several years after the patient had had a
col-Objectively: lips are covered with haemorrhagic crusts, hyperaemic mucous membrane of lips
and cheeks has erosions covered with fibrinous films. Hypersalivation is present. What is the most
Multiform
exudative erythema
likely diagnosis?
Herpetiform Duhring's dermatitis
Stevens-Johnson syndrome
Pemphigus vulgaris
Herpes recidivicus
A 47-year-old patient complains of subfebrile temperature and a growing ulcer on the gingival
mucosa around the molars; looseness of teeth in the affected area, cough for 2 month. Objectively:
gingival mucosa in the region of the lower left molars has two superficial, extremely painful ulcers
with undermined edges. The ulcers floor is yellowish, granular, covered with yellowish granulations.
What is the most likely diagnosis?
Syphilis
Tuberculosis
Decubital ulcer
Infectious mononucleosis
Acute aphthous stomatitis
187.
A.
B.
C.
D.
E.
188.
A.
B. *
C.
D.
E.
189.
A.
B. *
C.
D.
E.
190.
A. *
B.
C.
D.
E.
191.
A.
B. *
C.
D.
E.
192.
A.
B. *
C.
D.
E.
A 46-year-old patient complains of subfebrile temperature and a growing ulcer on the gingival
mucosa around the molars; looseness of teeth in the affected area, cough for 2 month. Objectively:
gingival mucosa in the region of the lower left molars has two superficial, extremely painful ulcers
with undermined edges. The ulcers floor is yellowish, granular, covered with yellowish granulations.
Patient was diagnosed with tuberculosis. What should be the dentist tactics at that case?
Oral cavity sanitation, ulcer treatment
Treatment of the ulcrers, direction to the tuberculosis dispensary
Recommendation for ulcrers treatment, direction to the tuberculosis dispensary
Just direction to the tuberculosis dispensary
Oral cavity sanitation
67 year old patient was diagnosed with decubital ulcer. What is the main causative factor for this
lesion?trauma
Acute
Chronic trauma
Infection
Alcohol
Low immunity
Patient was diagnosed with decubital ulcer. What will be the treatment?
Surgical treatment
Antiseptics for 7 days
Antibiotics for 7 days
Antiviral for 7 days
Laser treatment
After 56 year old patient examination he was diagnosed with decubital ulcer. What should doctor do
at that case?
Remove causative factor, prescribe oral washing for 7 days, if it’s not helpful – take byopsy
Remove causative factor, prescribe oral washing for 7 days, if it’s not helpful – for 1 month
Prescribe biopsy immediately
Prescribe antibiotic treatment
No treatment is needed
A 28-year-old patient complains about body temperature rise, weakness, pain induced by eating and
deglutition. Objectively: mucous membrane of the oral cavity is erythematic with multiple petechies.
Pharynx is hyperaemic. Regional lymph nodes are enlarged, mobile, painless. In blood: leukocytosis,
monocytosis, atypic mononuclear cells, ESR is 30 mm/h. What is the leading factor of disease
development?
Immediate allergy
Viral infection
Delayed allergy
Autoimmune disorders
Bacterial infection
Objective examination of 18 years old female patient revealed that her lower lip was slightly
hyperemic, dry, covered with small scales. Patient complains of dryness and a feeling of tense lips,
especially in autumn and winter, at the wind. What is the most likely diagnosis?
Allergic cheilitis
Meteorological cheilitis
Microbial cheilitis
Atopic cheilitis
Exfoliative cheilitis
193.
A.
B. *
C.
D.
E.
194.
A.
B.
C. *
D.
E.
195.
A.
B.
C. *
D.
E.
196.
A.
B. *
C.
D.
E.
197.
A.
B.
C.
D. *
E.
198.
A.
B.
Objective examination of 16 years old female patient revealed that her lower lip was slightly
hyperemic, dry, covered with small scales. Patient complains of dryness and a feeling of tense lips,
especially in autumn and winter, at the wind. Patient was diagnosed with meteorological cheilitis.
What will be the treatment?
Antibiotics taking
Using some lip oinment at that period
Antiseptic using at that periods
Surgical removing of scales with antiinflammatory drugs after
Antiinflammatory therapy
A 23 year old patient complains about itch, burning and edema of lips. She has been suffering from
this for a week. Objectively: reddening of vermilion border and skin, especially in the region of
mouth corners, there are also crusts, small cracks along with erythematous affection of vermilion
border. What is the most likely diagnosis?
Acute herpetic cheilitis
Exudative form of exfoliative cheilitis
Acute eczematous cheilitis
Fungal stomatitis
Multiform exudative erythema
Patient complains about a painful formation in the oral cavity that obstructs food intake. The same
complaints were noticed two years ago. Mucous membrane of lateral tongue surface is hyperemic
and edematic. There is an oval erosion over 0,7 cm large covered with yellow greyish deposit.
Erosion edges are hyperemic and painful on palpation. Patient has a history of chronic
cholecystocholangitis. What is the most likely diagnosis?
Erythema multiforme
Stevens-Johnson syndrome
Chronic recurrent aphthous stomatitis
Traumatic erosion
Behcet's syndrome
During tooth extraction a 31 y.o. patient presented with sudden weakness, pale skin, cold sweat,
weak pulse, a significant AP drop (diastolic pressure - 40 mm Hg). What complication developed in
the patient?
Anaphylactic shock
Collapse
Attack of stenocardia
Syncope
Traumatic shock
During everyday therapeutic manipulations dentist uses a dental mirror. What is the way of dental
mirrors sterilization?
In the 6% hydrogen peroxide solution for 6 hours
In the 0,5% ethyl spiritus solution for 20 minutes
In the dry heat sterilizer at 180oC for 10 minutes
In the triple solution for 30 minutes
In the 0,01% chloramine solution for 2 minutes
A 48 y. o. male patient complains of pain under the dental bridge. After its removal the patient has
been found to have an ulcer 0,3x0,5 cm large on the alveolar process. The ulcer is slightly painful and
soft, the surrounding mucosa is hyperaemic, submandibular lymph nodes are not enlarged. What is a
provisional diagnosis?
Tuberculous ulcer
Trophic ulcer
C.
D. *
E.
199.
A.
B.
C.
D.
E. *
200.
A.
B.
C. *
D.
E.
201.
A.
B.
C.
D. *
E.
202.
A.
B.
C. *
D.
E.
203.
A.
B.
C.
D.
E. *
Cancerous ulcer
Decubital ulcer
Sutton aphtha
A 47 y. o. male patient complains of pain under the dental bridge. After its removal the patient has
been found to have an ulcer 0,3x0,5 cm large on the alveolar process. The ulcer is slightly painful and
soft, the surrounding mucosa is hyperaemic, submandibular lymph nodes are not enlarged. Patient
was diagnosed with decubital ulcer. What should these lesion be differentiated with?
Tuberculous ulcer
Trophic ulcer
Cancerous ulcer
Syphilis ulcer
All answers are correct
A 26 year old male patient complains about burning and painfulness of her tongue, especially during
eating spicy food. Objectively: there are oval red spots on the tip and dorsum of tongue. Filiform
papillae are not present in the affected area. Patient mentions that the spots become periodically
larger and have migratory nature. What is the most probable diagnosis?
Raspberry tongue
Median rhomboid glossitis
Geographic tongue
Candidosis
Fissured tongue
A 30 year old patient complaines about painfulness of his lower lip. He has been suffering for two
months. Objectively: mucous membrane of lower lip is hyperemic, excretory ducts of minor salivary
glands are dilated, "dew" symptom is present. What is the most probable diagnosis?
Contact allergic cheilitis
Meteorological cheilitis
Cheilitis exfoliativa
Cheilitis glandularis
Microbal cheilitis
During pulpitis deep caries treatment a 63 year old patient felt dull pain behind her breastbone, got a
sense of compression. A dentist diagnosed her with a stenocardia attack. What medication should be
given this patient in order to arrest this state?
Analgin
Ketanov
Nitroglycerine, validol
Baralgin
Dimedrol
A 36 year old patient applied to a dental clinic for oral cavity sanation. Anamnesis data: mild case of
diabetes. Objectively: mucous membrane of cheeks is unchanged, mainly in retromolar area there are
symmetrically placed whitish papulae protruding over the mucous membrane and forming a lace-like
pattern. What is the most probable diagnosis?
Leukoplakia
Secondary syphilis
Chronic atrophic candidosis
Lupus erythematosus
Lichen ruber planus
204.
A.
B.
C. *
D.
E.
205.
A.
B.
C.
D.
E. *
206.
A.
B. *
C.
D.
E.
207.
A.
B.
C. *
D.
E.
208.
A.
B.
C.
D. *
E.
A 46 year old patient businessman complained about painful ulcer on the lateral surface of his
tongue. Objectively: left lateral surface of tongue has a roundish ulcer with undermined soft
overhanging edges, palpatory painful, ulcer floor is slightly bleeding and covered with yellowish
nodules. What is the most probable diagnosis?
Syphilis
Traumatic ulcer
Tuberculosis
Decubital ulcer
Trophic ulcer
A 75 year old patient complains of pain in the area of mucous membrane of hard palate on the left
that is getting worse during eating with use of a complete removable denture. He has been suffering
from this for 1,5 month. Objectively: left-sided hyperemia and edema of mucous membrane of hard
palate; at the border of distal denture edge there is an ulcer with dense walls and fundus, surrounding
tissues are infiltrated. The ulcer floor is tuberous, covered with fibrinous deposit; ulcer palpation is
painful. What examination method is to be applied in the first place?
Allergic contact plastic test
Blood test
Serological reactions
Bacterioscopy
Biopsy
A 25 year old woman complains about periodical appearance of small ulcers in the oral cavity. She
has been suffering from this for 3 years, recurrences happen 4-5 times a year. The ulcer healing lasts
for 7 days. Objectively: on a mucous membrane of lower lip there is a roundish lesion element 0,5
cm large covered with white deposit and surrounded by hyperemia border, very painful when
touched. What is the most probable diagnosis?
Secondary syphilis
Chronic recurrent aphthous stomatitis
Chronic recurrent herpes
Traumatic erosion
Duhring's herpetiform dermatitis
A 32 year old woman complains about periodical appearance of small ulcers in the oral cavity. She
has been suffering from this for 3 years, recurrences happen 4-5 times a year. The ulcer healing lasts
for 7 days. Objectively: on a mucous membrane of lower lip there is a roundish lesion element 0,5
cm large covered with white deposit and surrounded by hyperemia border, very painful when
touched.She was diagnosed with cronic reccurent aphthous stomatitis. What will be the treatment at
Antibiotics
that case? for 7 days
Antiviral for 10 days
Antiseptics and analgetics symptomaticly
Surgical treatment of the ulcer
Corticosteroid therapy
Examination of a 53 year old patient revealed chronic candidosis of oral mucous membrane,
generalized lymphadenopathy. Anamnesis data: the patient has been suffering from herpes for a year.
Body temperature persistently rises up to 37,4-37,5oC, body weight has reduced by 8 kg over the last
month. What disease can be indicated by this symptom group?
Infectious mononucleosis
Candidosis
Acute leukosis
AIDS
Chronic leukosis
209.
A. *
B.
C.
D.
E.
210.
A.
B.
C.
D. *
E.
211.
A.
B.
C. *
D.
E.
212.
A.
B.
C. *
D.
E.
213.
A.
B.
C. *
D.
E.
A 51 year old patient applied to a dentist and complained about white caseous deposit on the dorsum
of his tongue and burning sensation. It is known from the patient's anamnesis that he underwent
treatment in an in-patient hospital on account of pneumonia. What is the most probable diagnosis?
Acute pseudomembranous candidosis
Typical form of leukoplakia
AIDS
Lichen ruber planus
Lupus erythematosus
An 41 year old patient complains of gingival painfulness and haemorrhage, halitosis, temperature rise
up to 38,6oC, general weakness, appetite loss. Objectively: mucous membrane of oral cavity is
hyperemic and dry; tongue is covered with white fur, gingival papillae are edematic, their apices have
areas of necrotic deposit that can be easily removed leaving bleeding surface beneath. Submaxillary
lymph nodes are enlarged, palpatory painful. What is the causative agent of this disease?
Treponema
Candida albicans
Virus of herpes simplex
Fusospirochetal symbiosis
Streptostaphylococci
A 28 year old female patient complains about itch, burning and edema of lips. These presentations
occured a week ago. Objectively: there is reddening of vermilion border and skin, especially in the
region of mouth corners, there are also crusts, small cracks along with erythematous affection of
vermilion border. What is the most likely diagnosis?
Acute herpetic cheilitis
Multiform exudative erythema
Acute eczematous cheilitis
Allergic contact cheilitis
Exudative form of exfoliative cheilitis
A 39 y.o. patient complains of a painless nonhealing mouth ulcer. Objectively: regional lymph nodes
are enlarged, painless, of cartilaginous consistency. On the buccal mucosa there is a round ulcer 1 cm
in diameter with regular raised edges and a dense elastic infiltrate at the base. The ulcer surface is of
red colour, painless on palpation. What is the most likely diagnosis?
Secondary tuberculosis
Primary tuberculosis
Primary syphilis
Cancer
Secondary syphilis
A 37 y.o. patient complains of a painless nonhealing mouth ulcer. Objectively: regional lymph nodes
are enlarged, painless, of cartilaginous consistency. On the buccal mucosa there is a round ulcer 1 cm
in diameter with regular raised edges and a dense elastic infiltrate at the base. The ulcer surface is of
red colour, painless on palpation. Patient was diagnosed with prymary syphilis. What should be the
doctor tactics at that case?
Professional oral hygiene, antiseptics for ulcer treatment
Antiseptics for ulcer treatment, antibiotics for syphilis treatment
Direction to venereologist, symptomatic treatment precription
Direction to venereologist
No treatment is needed
214.
A.
B. *
C.
D.
E.
215.
A.
B.
C. *
D.
E.
216.
A.
B.
C.
D. *
E.
217.
A. *
B.
C.
D.
E.
218.
A. *
B.
C.
D.
E.
A 55 year old male patient complains of a painful ulcer in the mouth that is getting bigger and does
not heal over 1,5 months. Objectively: on the buccal mucosa there is a shallow soft ulcer 2 cm in
diameter with irregular undermined edges. The ulcer is surrounded by many small yellowish
tubercles. Regional lymph nodes are elastic, painful, matted together. Which disease is characterized
Ulcerative
necrotizing stomatitis
by such symptoms?
Tuberculosis
Lichen planus
Syphilis
Cancer
A 56 year old male patient complains of a painful ulcer in the mouth that is getting bigger and does
not heal over 1,5 months. Objectively: on the buccal mucosa there is a shallow soft ulcer 2 cm in
diameter with irregular undermined edges. The ulcer is surrounded by many small yellowish
tubercles. Regional lymph nodes are elastic, painful, matted together. Patient was diagnosed with
tuberculosis. What should be a doctor tactic at that case?
Professional oral hygiene, antiseptics for ulcer treatment
Antiseptics for ulcer treatment, antibiotics for tuberculosis treatment
Direction to T.B. prophylactic centre (dispensary) , symptomatic treatment precription
Direction to T.B. prophylactic centre (dispensary)
No treatment is needed
A 56 year old male patient complains of a painful ulcer in the mouth that is getting bigger and does
not heal over 1,5 months. Objectively: on the buccal mucosa there is a shallow soft ulcer 2 cm in
diameter with irregular undermined edges. The ulcer is surrounded by many small yellowish
tubercles. Regional lymph nodes are elastic, painful, matted together. Patient was diagnosed with
tuberculosis. What additional method of examination is needed to clarify the diagnosis?
Biopsy
X-ray
Histological
Bacteriological
Nikolskyy sign
A 38 y.o. patient complains of general weakness, pain in the gums, halitosis. Objectively: the patient
is pale, adynamic, body temperature is 38,5°C, submandibular lymph nodes are enlarged, painful on
palpation. Interdental gingival papillae are inflamed, their tops are "detruncated", covered with
gray-yellow necrotic incrustation. Radiography of alveolar process and blood count reveal no
apparent changes. What is the most likely diagnosis?
Acute ulcerous gingivitis
Acute leukosis
Localized periodontitis
Agranulocytosis
Generalized periodontitis
A 66 year old patient complains of burning tongue, a metallic taste in mouth. Three months ago he
got a dental bridge made of gold and supported by the 16, 14 teeth. Oral cavity exasmination reveals
no objective changes. The 36, 37, 46 teeth are sealed with amalgam fillings. What is the most likely
cause of this condition
Galvanic currents
Neurologic disorder
Mechanic trauma
Chemical factors
Allergy
219.
A.
B.
C.
D. *
E.
220.
A.
B.
C.
D. *
E.
221.
A.
B.
C. *
D.
E.
222.
A. *
B.
C.
D.
E.
223.
A.
B.
C.
D. *
E.
224.
A.
B.
C. *
Patient complains of general weakness, spasmodic neuralgic pain in the right side of face, rash in the
mouth and on the skin. Objectively: lip and chin skin is markedly hyperemic, there are numerous
vesicles with clear exudate on the right. The right cheek mucosa is hyperemic, there is a string of
erosions covered with fibrinous pellicle. What is the provisional diagnosis?
Acute herpetic stomatitis
Allergic stomatitis
Aphthous fever
Herpes zoster
Pemphigus vulgaris
33 y.o. patient complains of general weakness, spasmodic neuralgic pain in the left side of face, rash
in the mouth and on the skin. Objectively: lip and chin skin is markedly hyperemic, there are
numerous vesicles with clear exudate on the left. The left cheek mucosa is hyperemic, there can be
visible vesicles that looks like bunch of grapes. What will be the diagnosis?
Acute herpetic stomatitis
Allergic stomatitis
Aphthous fever
Herpes zoster
Pemphigus vulgaris
26 year old chemical plant worker consulted a dentist about an oral mucosa burn caused by caustic
soda. Which of the following medications should be chosen for emergency care?
0,1% liquid ammonia
50% ethyl alcohol
0,5% acetic acid solution
3% sodium chloride solution
2% chlorhexidine solution
A 28 year old DJ suffers from a disease without prodromal manifestations that declares itself through
oral mucosa lesion consisting of 1-2 roundish elements 5-8 mm large which are circumscribed by a
hyperemic rim and covered with yellow-grey coating. The disaese recurrence is observed quite
regularly 3-4 times a year. These presentations are typical for the following disease:
Chronic recurrent aphthous stomatitis
Chronic herpes recidivicus
Papular syphilis
Erythema multiforme
Lichen ruber planus
Preventive examination of a 58 year old man revealed a well-defined area of opaque mucosal
epithelium of the left cheek that didnt protrude above the surrounding tissues and could not be
removed on scraping. Crowns of the 34, 35, 36 teeth were strongly decayed and had sharp edges.
What is the most likely diagnosis?
Lichen ruber planus
Candidiasis
Soft cancer
Leukoplakia
Lupus erythematosus
A 53 year old male patient complains about tongue pain that is getting worse during eating and
talking. Objectively: there is a painful ulcer 0,6 cm large on the lateral surface of tongue. The floor is
covered with grey deposit. The crown of the 47 tooth is destroyed. What is the most likely diagnosis?
Hard chancre
Trophic ulcer
Decubital ulcer
D.
E.
225.
A.
B.
C.
D.
E.
226.
A.
B.
C.
D.
E. *
227.
A. *
B.
C.
D.
E.
228.
A.
B. *
C.
D.
E.
229.
A.
B.
C.
D. *
E.
230.
Tuberculous ulcer
Cancerous ulcer
A 63 year old male patient complains about tongue pain that is getting worse during eating and
talking. Objectively: there is a painful ulcer 0,6 cm large on the lateral surface of tongue. The floor is
covered with grey deposit. The crown of the 47 tooth is destroyed. What will be the key in decubital
ulcer treatment?
47 tooth restoration.
Antibiotics prescribing
Antiseptics prescribing
Analgetics prescribing
Improving the oral hygiene
During the anamnesis taking of 65 year old patient diabetes was revealed. Deficiency of which
hormone, described below, leads to diabetes:
Cortisone
Calcitonin
Parathyroid hormone
Thyroxine
Insulin
During 29 y.o. patient examination doctor revealed hyperplastic candidiasis and mild form of
Kaposi's sarcoma. From anamnesis is known, that patient has 37,5-37,6 C temperature for a month
and weight loss. What test should patient do to identify the diagnosis?
ELISA
Blood test
Insulin test
FTR test
Bacteriological finding
During 26 y.o. patient examination doctor revealed hyperplastic candidiasis and mild form of
Kaposi's sarcoma. From anamnesis is known, that patient has 37,5-37,6 C temperature for a month
and weight loss. What will be the provisional diagnosis in that case?
Tuberculosis
AIDS
Chronic candidiasis
Chronic leukosis
Chronic stomatitis
A 56-year-old female patient was waiting for her turn at the dentist’s. Suddenly she fell down, her
respiration became hoarse, she got convulsive twitching in her upper and lower limbs, face and neck
turned cyanotic, eye pupils became mydriatic, reaction of eye to light was absent. Arterial pressure
and pulse couldn’t be measured. Heart sounds couldn’t be auscultated. Involuntary urination was
noted. What condition is characterized by such symptoms?
Coma
Epilepsy
Shock
Clinical death
Collapse
A 40-year-old patient complains about mouth soreness, body temperature rise up to 38,5oC,
indisposition. Such condition has been occurring periodically for several years after the patient had
had a cold. Objectively: lips are covered with haemorrhagic crusts, hyperaemic mucous membrane of
lips and cheeks has erosions covered with fibrinous films. Hypersalivation is present. What is the
most likely diagnosis?
A. *
B.
C.
D.
E.
231.
A.
B.
C.
D.
E. *
232.
A. *
B.
C.
D.
E.
233.
A.
B.
C. *
D.
E.
234.
A.
B.
C.
D.
E. *
235.
A.
B.
Multiform exudative erythema
Pemphigus vulgaris
Herpes recidivicus
Herpetiform Duhring's dermatitis
Stevens-Johnson syndrome
A 23-years-old woman complained about having vesicles in oral cavity, painful ulcers, especially
during talking and eating; muscle and joint pain, body temperature rise, indisposition, weakness. She
fell suddenly ill 2 days ago. Objectively: to- 38,4oC. Vermilion border is covered with bloody crusts
that stick together and impede mouth opening. Mucous membrane of lips, cheeks, mouth floor,
tongue, soft palate is hyperemic and edematic; there are single vesicles and large painful erosions
covered with fibrinous deposit on it. Regional lymph nodes are enlarged and painful. There is
hypersalivation. Nikolsky's symptom is negative. What is the most probable diagnosis?
Acantholytic pemphigus
Multiform exudative erythema
Secondary syphilis
Acute herpetic stomatitis
Non-acantholytic pemphigus
A 28 year old patient applied to a clinic for the purpose of oral cavity sanitation. During preparation
of the 45 tooth on account of chronic median caries the patient turned pale, there appeared cold
clammy sweat on her forehead, nausea, ear noise. The patient lost consciousness. Objectively: pulse 50 bpm, AP - 80/60 mm Hg, shallow breath, miotic pupils. Make a diagnosis of this state:
Syncope
Anaphylactic shock
Stenocardia
Collapse
Quincke's edema
A 23-year-old patient was delivered to a traumatology centre with a dirty cut wound of her right foot.
A doctor performed initial surgical d-bridement and made an injection of antitetanus serum. Some
time later the patient’s condition got abruptly worse: she developed extreme weakness, dizziness,
palpitation. Objectively: the skin is pale, the patient has cold sweat, frequent pulse of poor volume at
a rate of 100 bpm, AP is 90/40 mm Hg. What is the cause of such aggravation?
Pain shock
Haemorrhagic shock
Anaphylactic shock
Infectious-toxic shock
Drug disease (seroreaction)
A 28-year-old man attended a glasshouse in a botanic garden. After he had smelt at an orchid he
turned pale and lost consciousness. Objectively: heart rate is 115/min, arterial pressure is 50/0 mm
Hg. What drug should be injected to the patient in the first place?
Dimedrol
Cordiamin
Strophanthine
Mesaton
Prednisolone
After injection of anaesthetic solution to 27-years-old man, doctor noticed the swelling of patient
face, which started from upper lip and rapidly progressed, wheezing and difficulty breathing. What is
the most likely diagnosis?
Idiopathic face edema
Anaphylactic shock
C. *
D.
E.
236.
A. *
B.
C.
D.
E.
237.
A.
B.
C. *
D.
E.
238.
A.
B.
C. *
D.
E.
239.
A.
B.
C.
D. *
E.
240.
A.
B.
C. *
D.
E.
241.
A.
B.
Quincke’s edema (angioedema)
Syncope
Asthma
After injection of anaesthetic solution to 39 year old patient before caries treatment, doctor noticed
the swelling of patient face, which started from upper lip and rapidly progressed, wheezing and
difficulty breathing. What doctors mistake can cause this complication?
Incorrect anamnesis taking.
Incorrect caries treatment.
Incorrect patient choosing
Incorrect anaesthetic injection
All answers are wrong
After injection of anaesthetic solution to 28 year old patient before pulpitis treatment, doctor noticed
the swelling of patient face, which started from upper lip and rapidly progressed, wheezing and
difficulty breathing. What will be the key medication in the treatment of this disease?
Antibiotics
Anti-inflammatory
Antihistamines
Corticosteroids
Antiviral
46-year-old woman complaines about swelling of her upper lip, wheezing and voice change which
started half an hour ago after bee bites. What is the most likely diagnosis?
Idiopathic face edema
Anaphylactic shock
Quincke’s edema (angioedema)
Syncope
Melkersson-Rosenthal syndrome
During the examination of 29-year-old patient the subcutaneous allergy test was made. 5 minutes
later after injection small red spots on the foream were revealed. Patient feel itch at those areas. What
does this reaction mean?
Erythema multiforme
Anaphylactic shock – allergic reaction after injection
Quincke’s edema – complication after injection
Nettle rash – allergy reaction after injection
Red idiopathic erythema
During the examination of 25-year-old woman the subcutaneous allergy test was made. 5 minutes
later after injection small red spots on the foream were revealed. Patient feel itch at those areas. What
will be your tactics at that case?
To try another syringe
To do additional allergic test
To choose another anaeathetic
To use analgetic spray in that area
To do nothing and to continue treatment
5 year old boy complains about painful mouth sores, which he noticed few days ago. From the
parents words this sores comes periodically, trice a year. Objectively: on the tongue and lower lip
there are two sores (0,7 mm in diameter) with hyperemic membrane, covered with grey-whitish
deposits. Food allergy in anamnesis. What is the most likely diagnosis?
Acute herpetic stomatitis
Erythema multiforme
C.
D. *
E.
242.
A. *
B.
C.
D.
E.
243.
A.
B.
C.
D. *
E.
244.
A. *
B.
C.
D.
E.
245.
A.
B.
C.
D.
E. *
246.
A. *
B.
Chronic herpetic stomatitis
Chronic aphthous stomatitis
Pre-cancer
18 year old yougster complains about painful mouth sores, which he noticed few days ago. From the
parents words this sores comes periodically, thrice a year. Objectively: on the tongue and lower lip
there are two sores (0,7 mm in diameter) with hyperemic membrane, covered with grey-whitish
deposits. Food allergy in anamnesis. What will be the treatment in that case?
Oral rinsing with antiseptics
Antibiotics for 5 days
Antibiotics for 7 days
Surgical treatment
Oral rinsing with corticosteroids
60-year-old male patient was waiting for her turn at the dentist’s. Suddenly he fell down, his
respiration became hoarse, he got convulsive twitching in his upper and lower limbs, face and neck
turned cyanotic, eye pupils became mydriatic, reaction of eye to light was absent. Arterial pressure
and pulse couldn’t be measured. Heart sounds couldn’t be auscultated. Involuntary urination was
noted. What condition is characterized by such symptoms?
Coma
Epilepsy
Shock
Clinical death
Collapse
38-year-old patient complains about mouth soreness, body temperature rise up to 38,5 C,
indisposition. Such condition has been occurring periodically for several years after the patient had
had a cold. Objectively: lips are covered with haemorrhagic crusts, hyperaemic mucous membrane of
lips and cheeks has erosions covered with fibrinous films. Hypersalivation is present. What is the
most likely diagnosis?
Multiform exudative erythema
Pemphigus vulgaris
Herpes recidivicus
Herpetiform Duhring's dermatitis
Stevens-Johnson syndrome
Patient complains about having vesicles in oral cavity, painful ulcers, especially during talking and
eating; muscle and joint pain, body temperature rise, indisposition, weakness. He fell suddenly ill 2
days ago. Objectively: temperature - 38,4 C. Vermilion border is covered with bloody crusts that
stick together and impede mouth opening. Mucous membrane of lips, cheeks, mouth floor, tongue,
soft palate is hyperemic and edematic; there are single vesicles and large painful erosions covered
with fibrinous deposit on it. Regional lymph nodes are enlarged and painful. There is hypersalivation.
Nikolsky's symptom is negative. What is the most probable diagnosis?
Acantholytic pemphigus
Multiform exudative erythema
Secondary syphilis
Acute herpetic stomatitis
Non-acantholytic pemphigus
A 37 year old patient applied to a clinic for the purpose of oral cavity sanitation. During preparation
of the 45 tooth on account of chronic median caries the patient turned pale, there appeared cold
clammy sweat on her forehead, nausea, ear noise. The patient lost consciousness. Objectively: pulse 50 bpm, AP - 80/60 mm Hg, shallow breath, miotic pupils. Make a diagnosis of this state:
Syncope
Anaphylactic shock
C.
D.
E.
247.
A.
B.
C. *
D.
E.
248.
A.
B.
C.
D.
E. *
249.
A.
B.
C. *
D.
E.
250.
A. *
B.
C.
D.
E.
251.
A.
B.
C.
D. *
E.
252.
A.
Stenocardia
Collapse
Quincke's edema
Patient R., 25 y.o. was delivered to the traumatology center with a dirty cut wound of her right foot.
A doctor performed initial surgical d-bridement and made an injection of antitetanus serum. After
some time later the patient’s condition got abruptly worse: she developed extreme weakness,
dizziness, palpitation. Objectively: the skin is pale, the patient has cold sweat, frequent pulse of poor
volume at a rate of 100 bpm, AP is 90/40 mm Hg. What is the cause of such aggravation?
Pain shock
Haemorrhagic shock
Anaphylactic shock
Infectious-toxic shock
Drug disease (seroreaction)
A 24 y.o. man attended a glasshouse in a botanic garden. After he had smelt at an orchid he turned
pale and lost consciousness. Objectively: heart rate is 115/min, arterial pressure is 50/0 mm Hg. What
drug should be injected to the patient in the first place?
Dimedrol
Cordiamin
Strophanthine
Mesaton
Prednisolone
After injection of anaesthetic solution to 31 year old patient before caries treatment, doctor noticed
the swelling of patient face, which started from upper lip and rapidly progressed, wheezing and
difficulty breathing. What is the most likely diagnosis?
Idiopathic face edema
Anaphylactic shock
Quincke’s edema (angioedema)
Syncope
Asthma
After injection of anaesthetic solution to 36 year old man, doctor noticed the swelling of patient face,
which started from upper lip and rapidly progressed, wheezing and difficulty breathing. What doctors
mistake can cause this complication?
Incorrect anamnesis taking.
Incorrect caries treatment.
Incorrect patient choosing
Incorrect anaesthetic injection
All answers are wrong
After injection of anaesthetic solution to 28 year old patient before pulpitis treatment, doctor noticed
the swelling of patient face, which started from upper lip and rapidly progressed, wheezing and
difficulty breathing. What will be the key medication in the treatment of this disease?
Antibiotics
Anti-inflammatory
Antihistamines
Corticosteroids
Antiviral
Patient complains about swelling of her upper lip, wheezing and voice changing which started half an
hour ago after bee bites. What is the most likely diagnosis?
Idiopathic face edema
B.
C. *
D.
E.
253.
A.
B.
C.
D. *
E.
254.
A. *
B.
C.
D.
E.
255.
A.
B.
C.
D.
E. *
256.
A. *
B.
C.
D.
E.
257.
A.
Anaphylactic shock
Quincke’s edema (angioedema)
Syncope
Melkersson-Rosenthal syndrome
A 57-year-old female patient was waiting for her turn at the dentist’s. Suddenly she fell down, her
respiration became hoarse, she got convulsive twitching in her upper and lower limbs, face and neck
turned cyanotic, eye pupils became mydriatic, reaction of eye to light was absent. Arterial pressure
and pulse couldn’t be measured. Heart sounds couldn’t be auscultated. Involuntary urination was
noted. What condition is characterized by such symptoms?
Coma
Epilepsy
Shock
Clinical death
Collapse
Patient complains about mouth soreness, body temperature rise up to 38,5oC, indisposition. Such
condition has been occurring periodically for several years after the patient had had a cold.
Objectively: lips are covered with haemorrhagic crusts, hyperaemic mucous membrane of lips and
cheeks has erosions covered with fibrinous films. Hypersalivation is present. What is the most likely
Multiform
diagnosis? exudative erythema
Pemphigus vulgaris
Herpes recidivicus
Herpetiform Duhring's dermatitis
Stevens-Johnson syndrome
Woman complains about having vesicles in oral cavity, painful ulcers, especially during talking and
eating; muscle and joint pain, body temperature rise, indisposition, weakness. She fell suddenly ill 2
days ago. Objectively: to- 38,4oC. Vermilion border is covered with bloody crusts that stick together
and impede mouth opening. Mucous membrane of lips, cheeks, mouth floor, tongue, soft palate is
hyperemic and edematic; there are single vesicles and large painful erosions covered with fibrinous
deposit on it. Regional lymph nodes are enlarged and painful. There is hypersalivation. Nikolsky's
symptom is negative. What is the most probable diagnosis?
Acantholytic pemphigus
Multiform exudative erythema
Secondary syphilis
Acute herpetic stomatitis
Non-acantholytic pemphigus
A 43 year old patient applied to a clinic for the purpose of oral cavity sanitation. During preparation
of the 45 tooth on account of chronic median caries the patient turned pale, there appeared cold
clammy sweat on her forehead, nausea, ear noise. The patient lost consciousness. Objectively: pulse 50 bpm, AP - 80/60 mm Hg, shallow breath, miotic pupils. Make a diagnosis of this state:
Syncope
Anaphylactic shock
Stenocardia
Collapse
Quincke's edema
A 34-year-old man was delivered to a traumatology centre with a dirty cut wound of her right foot. A
doctor performed initial surgical d-bridement and made an injection of antitetanus serum. Some time
later the patient’s condition got abruptly worse: she developed extreme weakness, dizziness,
palpitation. Objectively: the skin is pale, the patient has cold sweat, frequent pulse of poor volume at
a rate of 100 bpm, AP is 90/40 mm Hg. What is the cause of such aggravation?
Pain shock
B.
C. *
D.
E.
258.
A.
B.
C.
D.
E. *
259.
A. *
B.
C.
D.
E.
260.
A.
B.
C. *
D.
E.
261.
A. *
B.
C.
D.
E.
262.
A.
B.
C. *
D.
E.
263.
A.
B.
C. *
Haemorrhagic shock
Anaphylactic shock
Infectious-toxic shock
Drug disease (seroreaction)
A 26-year-old woman attended a glasshouse in a botanic garden. After she had smelt at an orchid he
turned pale and lost consciousness. Objectively: heart rate is 115/min, arterial pressure is 50/0 mm
Hg. What drug should be injected to the patient in the first place?
Dimedrol
Cordiamin
Strophanthine
Mesaton
Prednisolone
During caries treatment of 27 y.o. patient doctor decided to use an anaesthesia. What is the technique
of scratch test which can be used for allergy detecting?
Test is done by placing a drop of a solution containing a possible allergen on the scratched skin.
Test is done by injecting the allergen under the skin
Test is done by a drop of a solution containing a possible allergen on the normal skin.
Test is done by scratching the normal skin by infected tool.
No correct answer.
After injection of anaesthetic solution to 18 year old student before pulpitis treatment, doctor noticed
the swelling of patient face, which started from upper lip and rapidly progressed, wheezing and
difficulty breathing. What is the most likely diagnosis?
Idiopathic face edema
Anaphylactic shock
Quincke’s edema (angioedema)
Syncope
Asthma
After injection of anaesthetic solution to 28 year old engineer before caries treatment, doctor noticed
the swelling of patient face, which started from upper lip and rapidly progressed, wheezing and
difficulty breathing. What doctors mistake can cause this complication?
Incorrect anamnesis taking.
Incorrect caries treatment.
Incorrect patient choosing
Incorrect anaesthetic injection
All answers are wrong
After injection of anaesthetic solution to 30 year old patient, doctor noticed the swelling of patient
face, which started from upper lip and rapidly progressed, wheezing and difficulty breathing. What
will be the key medication in the treatment of this disease?
Antibiotics
Anti-inflammatory
Antihistamines
Corticosteroids
Antiviral
43 year old patient complains about swelling of her upper lip, wheezing and voice change which
started half an hour ago after bee bites. What is the most likely diagnosis?
Idiopathic face edema
Anaphylactic shock
Quincke’s edema (angioedema)
D.
E.
264.
A.
B.
C.
D. *
E.
265.
A.
B.
C. *
D.
E.
266.
A.
B.
C.
D. *
E.
267.
A. *
B.
C.
D.
E.
268.
A.
B.
C.
D. *
E.
Syncope
Melkersson-Rosenthal syndrome
During the examination of 27 year old woman the subcutaneous allergy test was made. 5 minutes
later after injection small red spots on the foream was revealed. Patient feel itch at those areas. What
does this reaction mean?
Erythema multiforme
Anaphylactic shock – allergic reaction after injection
Quincke’s edema – complication after injection
Nettle rash – allergy reaction after injection
Red idiopathic erythema
During the examination of 22 year old patient the subcutaneous allergy test was made. 5 minutes later
after injection small red spots on the foream was revealed. Patient feel itch at those areas. What will
be your tactics at that case?
To try another syringe
To do additional allergic test
To choose another anaeathetic
To use analgetic spray in that area
To do nothing and to continue treatment
40 year old male complains about painful mouth sores, which he noticed few days ago. From the
parents words this sores comes periodically, thrice a year. Objectively: on the tongue and lower lip
there are two sores (0,7 mm in diameter) with hyperemic membrane, covered with grey-whitish
deposits. Food allergy in anamnesis. What is the most likely diagnosis?
Acute herpetic stomatitis
Erythema multiforme
Chronic herpetic stomatitis
Chronic aphthous stomatitis
Pre-cancer
54 year old patient complains about painful mouth sores, which he noticed few days ago. From the
parents words this sores comes periodically, thrice a year. Objectively: on the tongue and lower lip
there are two sores (0,7 mm in diameter) with hyperemic membrane, covered with grey-whitish
deposits. Food allergy in anamnesis. What will be the treatment in that case?
Oral rinsing with antiseptics
Antibiotics for 5 days
Antibiotics for 7 days
Surgical treatment
Oral rinsing with corticosteroids
39 year old patient was waiting for her turn at the dentist’s. Suddenly she fell down, her respiration
became hoarse, she got convulsive twitching in her upper and lower limbs, face and neck turned
cyanotic, eye pupils became mydriatic, reaction of eye to light was absent. Arterial pressure and
pulse couldn’t be measured. Heart sounds couldn’t be auscultated. Involuntary urination was noted.
What condition is characterized by such symptoms?
Coma
Epilepsy
Shock
Clinical death
Collapse
269.
A. *
B.
C.
D.
E.
270.
A.
B.
C.
D.
E. *
271.
A. *
B.
C.
D.
E.
272.
A.
B.
C. *
D.
E.
273.
A.
B.
C.
D.
E. *
Patient complains about mouth soreness, body temperature rise up to 38,5 C, indisposition. Such
condition has been occurring periodically for several years after the patient had had a cold.
Objectively: lips are covered with haemorrhagic crusts, hyperaemic mucous membrane of lips and
cheeks has erosions covered with fibrinous films. Hypersalivation is present. What is the most likely
Multiform
diagnosis? exudative erythema
Pemphigus vulgaris
Herpes recidivicus
Herpetiform Duhring's dermatitis
Stevens-Johnson syndrome
33 year old patient complains about having vesicles in oral cavity, painful ulcers, especially during
talking and eating; muscle and joint pain, body temperature rise, indisposition, weakness. He fell
suddenly ill 2 days ago. Objectively: temperature - 38,4 C. Vermilion border is covered with bloody
crusts that stick together and impede mouth opening. Mucous membrane of lips, cheeks, mouth floor,
tongue, soft palate is hyperemic and edematic; there are single vesicles and large painful erosions
covered with fibrinous deposit on it. Regional lymph nodes are enlarged and painful. There is
hypersalivation. Nikolsky's symptom is negative. What is the most probable diagnosis?
Acantholytic pemphigus
Multiform exudative erythema
Secondary syphilis
Acute herpetic stomatitis
Non-acantholytic pemphigus
Patient G., applied to a clinic for the purpose of oral cavity sanitation. During preparation of the 45
tooth on account of chronic median caries the patient turned pale, there appeared cold clammy sweat
on her forehead, nausea, ear noise. The patient lost consciousness. Objectively: pulse - 50 bpm, AP 80/60 mm Hg, shallow breath, miotic pupils. Make a diagnosis of this state:
Syncope
Anaphylactic shock
Stenocardia
Collapse
Quincke's edema
37 y.o. patient was delivered to the traumatology centre with a dirty cut wound of her right foot. A
doctor performed initial surgical d-bridement and made an injection of
antitetanus serum. Some
time later the patient’s condition got abruptly worse: she developed extreme weakness, dizziness,
palpitation. Objectively: the skin is pale, the patient has cold sweat, frequent pulse of poor volume at
a rate of 100 bpm, AP is 90/40 mm Hg. What is the cause of such aggravation?
Pain shock
Haemorrhagic shock
Anaphylactic shock
Infectious-toxic shock
Drug disease (seroreaction)
A 30 y.o. male attended a glasshouse in a botanic garden. After he had smelt at an orchid he turned
pale and lost consciousness. Objectively: heart rate is 115/min, arterial pressure is 50/0 mm Hg. What
drug should be injected to the patient in the first place?
Dimedrol
Cordiamin
Strophanthine
Mesaton
Prednisolone
274.
A.
B.
C. *
D.
E.
275.
A. *
B.
C.
D.
E.
276.
A.
B.
C. *
D.
E.
277.
A.
B.
C. *
D.
E.
278.
A.
B.
C.
D. *
E.
279.
A. *
B.
C.
D.
E.
After injection of anaesthetic solution to 40 year old patient before caries treatment, doctor noticed
the swelling of patient face, which started from upper lip and rapidly progressed, wheezing and
difficulty breathing. What is the most likely diagnosis?
Idiopathic face edema
Anaphylactic shock
Quincke’s edema (angioedema)
Syncope
Asthma
After injection of anaesthetic solution to 65 year old patient, doctor noticed the swelling of patient
face, which started from upper lip and rapidly progressed, wheezing and difficulty breathing. What
was doctors mistake can cause this complication?
Incorrect anamnesis taking.
Incorrect caries treatment.
Incorrect patient choosing
Incorrect anaesthetic injection
All answers are wrong
After injection of anaesthetic solution to 38 year old patient, doctor noticed the swelling of patient
face, which started from upper lip and rapidly progressed, wheezing and difficulty breathing. What
will be the key medication in the treatment of this disease?
Antibiotics
Anti-inflammatory
Antihistamines
Corticosteroids
Antiviral
Patient A., 34 y.o., complaines about swelling of her upper lip, wheezing and voice changing which
started half an hour ago after bee bites. What is the most likely diagnosis?
Idiopathic face edema
Anaphylactic shock
Quincke’s edema (angioedema)
Syncope
Melkersson-Rosenthal syndrome
A 34 years old patient came to the dentist for sanation. The doctor found signs of catarrhal gingivitis
and the dark fringe on the gum free edge beside of front teeth' neck. From the anamnesis — the
patient works in printing office for years. What disease we suspect?
Addison's disease
Mercury stomatitis
Necrotizing stomatitis
Lead stomatitis
Galvanosis
A 27 years old patient complains on gray-black spots in the red lip rim area and acompanying
weakness and headache. Is treated at an endocrinologist about the violation of adrenal hormones.
What disease we suspect?
Addison's disease
Lead stomatitis
Mercury stomatitis
Bismuth stomatitis
Cadmium stomatitis
280.
A.
B. *
C.
D.
E.
281.
A.
B.
C. *
D.
E.
282.
A.
B. *
C.
D.
E.
283.
A.
B.
C.
D. *
E.
284.
A.
B.
C. *
D.
E.
285.
A.
B.
C.
D.
E. *
A 46 years old patient who works at gas station complains on unpleasant metallic taste in a mouth,
difficulty chewing, indigestion and feeling unwell. Objectively: there is a dark, grey-blue strip around
bottom teeth necks and yellow spots in the sky. The skin is earthy-grey. What disease we suspect?
Addison's disease
Lead stomatitis
Zinc stomatitis
Mercury stomatitis
No correct answer
A patient (35 y.o.) who works at mirror manufacturing complains on headache, malaise, feeling the
heat, salivation violation, severe throbbing pain in mouth. There is grey-black border around teeth
necks, hyperemic swollen gums. What metal will be detected after the urine analysis?
Lead
Bismuth
Mercury
Zinc
No correct answer
A patient complains on increased salivation, headache, unpleasant metallic taste in a mouth,
throbbing pain in gums. Man works at mirror manufacturing. What disease we suspect?
Galvanosis
Mercury stomatitis
Vegetative vascular dystonia
Endemic parotitis
Purulent necrotic angina of oral cavity bottom
The patient complains on metallic taste in the mouth, feeling the heat, acute throbbing pain in the
mouth and headache. Gums are hyperemic and swollen. The patient works on mirror manufacturing.
What is the diagnosis?
Bismuth stomatitis
Ulcer-necrotizing gingivitis
Catarrhal gingivitis
Mercury stomatitis
Galvanism
A patient complains on unpleasant taste in the mouth, specific breath, dark spots on lips and tongue.
Objectively: the skin is grey-ground. In the mouth - catarrhal gingivitis and there is a blue-black strip
around front teeth necks from the vestibular side. Poisoning by which metal makes such changes?
Tin
Zinc
Lead
Mercury
Argentum
Mercury stomatitis was diagnosed on 24 y.o. patient. What should we prescribe for treating chronic
intoxication?
D-penicillamine
Vitamin C
Potassium iodide
Polyvitamins
All answers are correct
286.
A. *
B.
C.
D.
E.
287.
A.
B.
C. *
D.
E.
288.
A.
B.
C.
D.
E. *
289.
A.
B.
C. *
D.
E.
290.
A.
B.
C.
D. *
E.
291.
A. *
B.
C.
D.
E.
The patient (28 y.o.) complains on metallic taste, unpleasant specific taste, dark spots on lips and
tongue, difficulty chewing. In the mouth - catarrhal gingivitis and there is a blue-black strip around
front teeth necks from the vestibular side. Local treatment of this disease is carried by the scheme as:
Treatment of ulcerative necrotic stomatitis
Treatment of catarrhal gingivitis
Treatment of chronic generalized periodontitis
Treatment of parotitis
Treatment of acute pseudomembranous candidiasis
33 y.o. woman complains on unpleasant specific taste, dark spots on lips and tongue, difficulty
chewing. In the mouth - catarrhal gingivitis and there is a blue-black strip around front teeth necks
from the vestibular side. What is one of the first symptoms of this desease?
Wrinkled kidney symptom
Difficulty chewing andtrismus
Metallic taste and specific lead breath
Optic atrophy
Nervous system damage (paresthesia, anesthesia, paralysis)
The patient (40 y.o., a painter) complains on metallic taste, difficulty chewing, weakness, indigestion.
Objectively: the skin is fround-grey, dark-blue strips around teeth necks, yellow spots in the sky.
What medication is used for this desease?
Dimercaprol
EDTA
Succimer
Penicillamine
All answers are correct
42 y.o. patient is diagnosed with mercury stomatitis. What medication is used for removing mercury
from the body?
Mineral water
Smecta
Unithiolum
Atropine
All these drugs complex
A 32 years old patient came to the dentist for sanation. The doctor found signs of catarrhal gingivitis
and the dark fringe on the gum free edge beside of front teeth' neck. From the anamnesis — the
patient works in printing office for years. What disease does we suspect?
Addison's disease
Mercury stomatitis
Necrotizing stomatitis
Lead stomatitis
Galvanosis
A 26 years old patient complains on gray-black spots in the red lip rim area and acompanying
weakness and headache. Is treated at an endocrinologist about the violation of adrenal hormones.
What disease we suspect?
Addison's disease
Lead stomatitis
Mercury stomatitis
Bismuth stomatitis
Cadmium stomatitis
292.
A.
B. *
C.
D.
E.
293.
A.
B.
C. *
D.
E.
294.
A.
B. *
C.
D.
E.
295.
A.
B.
C.
D. *
E.
296.
A.
B.
C. *
D.
E.
297.
A.
B.
C.
D.
E. *
A 43 years old patient who works at gas station complains on unpleasant metallic taste in a mouth,
difficulty chewing, indigestion and feeling unwell. Objectively: there is a dark, grey-blue strip around
bottom teeth necks and yellow spots in the sky. The skin is earthy-grey. What disease we suspect?
Addison's disease
Lead stomatitis
Zinc stomatitis
Mercury stomatitis
No correct answer
A patient (36 y.o.) who works at mirror manufacturing complains on headache, malaise, feeling the
heat, salivation violation, severe throbbing pain in mouth. There is grey-black border around teeth
necks, hyperemic swollen gums. What metal will be detected after the urine analysis?
Lead
Bismuth
Mercury
Zinc
No correct answer
A 30-year-old patient complains on increased salivation, headache, unpleasant metallic taste in a
mouth, throbbing pain in gums. Man works at mirror manufacturing. What disease we suspect?
Galvanosis
Mercury stomatitis
Vegetative vascular dystonia
Endemic parotitis
Purulent necrotic angina of oral cavity bottom
The patient complains on metallic taste in the mouth, feeling the heat, acute throbbing pain in the
mouth and headache. Gums are hyperemic and swollen. The patient works on mirror manufacturing.
What is the diagnosis?
Bismuth stomatitis
Ulcer-necrotizing gingivitis
Catarrhal gingivitis
Mercury stomatitis
Galvanism
A 46 year old manager patient complains on unpleasant taste in the mouth, specific breath, dark spots
on lips and tongue. Objectively: the skin is grey-ground. In the mouth - catarrhal gingivitis and there
is a blue-black strip around front teeth necks from the vestibular side. Poisoning by which metal
makes such changes?
Tin
Zinc
Lead
Mercury
Argentum
Mercury stomatitis was diagnosed in 26 y.o. accounter. What should we prescribe for treating chronic
intoxication?
D-penicillamine
Vitamin C
Potassium iodide
Polyvitamins
All answers are correct
298.
A. *
B.
C.
D.
E.
299.
A.
B.
C. *
D.
E.
300.
A.
B.
C.
D.
E. *
301.
A.
B.
C. *
D.
E.
302.
A.
B.
C. *
D.
E.
303.
A.
B. *
C.
D.
E.
The patient (26 y.o.) complains on metallic taste, unpleasant specific taste, dark spots on lips and
tongue, difficulty chewing. In the mouth - catarrhal gingivitis and there is a blue-black strip around
front teeth necks from the vestibular side. Local treatment of this disease is carried by the scheme as:
Treatment of ulcerative necrotic stomatitis
Treatment of catarrhal gingivitis
Treatment of chronic generalized periodontitis
Treatment of parotitis
Treatment of acute pseudomembranous candidiasis
25 year old nurse complains on unpleasant specific taste, dark spots on lips and tongue, difficulty
chewing. In the mouth - catarrhal gingivitis and there is a blue-black strip around front teeth necks
from the vestibular side. What is one of the first symptoms of this desease?
Wrinkled kidney symptom
Difficulty chewing andtrismus
Metallic taste and specific lead breath
Optic atrophy
Nervous system damage (paresthesia, anesthesia, paralysis)
The patient (42 y.o., a painter) complains on metallic taste, difficulty chewing, weakness, indigestion.
Objectively: the skin is fround-grey, dark-blue strips around teeth necks, yellow spots in the sky.
What medication is used for this desease?
Dimercaprol
EDTA
Succimer
Penicillamine
All answers are correct
43 y.o. patient is diagnosed with mercury stomatitis. What medication is used for removing mercury
from the body?
Mineral water
Smecta
Unithiolum
Atropine
All these drugs complex
45-years old waiter is diagnosed with erosive form of leukoplakia. What kind of therapy should we
apply?
Phototherapy
Hydrotherapy
Cryotherapy
Radiotherapy
Vacuum therapy
62-years old patient suffers from high sunlight sensitivity. There are red spots on the face, which
have developed with atrophic changes. In these areas skin became thin. The shiny spots, that look
like leaves-like, and the warts have appeared. Diagnosis?
Senile keratosis
Xeroderma pigmentosum
Bowen's disease
Erythroplasia of Queyrat
Erysipelas
304.
A.
B.
C. *
D.
E.
305.
A.
B. *
C.
D.
E.
306.
A.
B.
C.
D.
E. *
307.
A.
B.
C. *
D.
E.
308.
A.
B.
C.
D. *
E.
309.
A. *
B.
C.
Semispherical dense formation was found above the lower lip in the 60-years professor. In the central
part of it there is the crater filled with keratinized gray mass. The formation is grey-brown.
Kerarinized mass is easy to remove, and we can see dry bottom with papillary excrescence and tight
shaft on the edge of the crater. No bleeding after mass removal is observed. Formation is not soldered
to the surrounding tissues. What is the diagnosis?
Syphilitic ulcer
Squamous cell carcinoma
Keratoakantoma
limited hyperkeratosis
red lip rims warty precancer
The presumptive diagnosis of 49-years old patient is limited inhyperkeratosis of lower lip red border.
What method of diagnosis should we use in this case?
X-ray
Biopsy
Puncture
Smear-mark
Scraping of the surface formation
There is keratinized area on the cheek mucosa along the interdigitation line. It rises above the
surrounding tissues. Also it's grayish-white, dense to the touch, filled with erosion and cracks. What
is the preliminary diagnosis?
simple leukoplakia
verrucous leukoplakia
Lichen planus
lupus erythematosus
erosive and ulcerative form of leukoplakia
Patient L. (69 years old) two years ago found keratinized ledge on the chin. This ledge is conical,
tapering top. Put diagnosis.
warty leukoplakia
Keratoakantoma
Skin corner
Nodular skin precancer
Erythroplasia of Queyrat
41-year old patient complains on burning lower lip mucosa, presence of granular formation for
several months. Objectively: there is painless formation on the lower lip mucosa, rightside in Klein
area. This formation (6x14 mm) rises on surrounding tissues with grayish tight warts. What is the
most likely diagnosis?
Chronic hyperplastic candidiasis
Viral wart
Papilloma with keratinization
verrucous leukoplakia
warty precancer
The patient (59 years old) complains on color change of the lower lip red rim. Changes have started 4
months ago. Objectively: missing 31, 32, 41, 42. On the center of lower lip red rim there is irregular
shaped area (grey-white, 8x14 mm) which is on the same level as red rim. Palpation is painless, the
surrounding tissue isn't changed. When scraping pellicle is not removing. What is the most likely
diagnosis?
leukoplakia
Limited precancerous hyperkeratosis
Red flat lichen
D.
E.
310.
A.
B. *
C.
D.
E.
311.
A. *
B.
C.
D.
E.
312.
A. *
B.
C.
D.
E.
313.
A.
B.
C.
D. *
E.
314.
A.
B.
C. *
D.
E.
315.
A.
B.
C.
D.
Candida cheilitis
lupus erythematosus
82-years old patient complains on ulcer on the lower lip red rim, which have appeared three month
ago. Objectively: there is irregular shaped erosion (8x12 mm) on the lower lip red rim. It's covered
with bloody crust and lightly bleeding after removing it. Palpation is painless, no signs of
inflammation around, regional lymph nodes are not enlarged. What additional research should be
Serological
held?
Histological
Immunological
fluorescent
Bacterioscopic
53-years old patient complains on pathological changes of the lower lip. Objectively: the lip is
purplish-red; there are erythematous spots and erosions on the red rim. Wasting areas in the center of
affection, on the periphery - epithelium dimmed areas in the form of irregular white stripes. Oral
mucosa are intact. What additional research should be appeal?
Histological
Serological
blood
Immunological
Cytological
47-years old patient, smokes excessively. The mouth cavity is not treated. Verrucous leukoplakia was
diognosed. How will look like the mucous membrane of this patient?
presence of warty excrescence of the epithelium
presence of epithelium atrophy
presence of crack epithelium
presence of whitish spot
presence of limited swelling
Mr G. (53 y.o.) complains on heartburn in the mouth. He is smoker for 30 years. Objectively: there is
triangle-like hyperceratozic plaque on the cheek mucosa, on the hard palate there is greyish
keratinized area. Non-keratinized areas are just round ducts of minor salivary glands. Diagnosis?
Red flat lichen
lupus erythematosus
Hyperplasic candidiasis
leukoplakia smokers
Atrophic candidiasis
The patient is 62 years old, smokes over 20 years. There is black hyperceratozic plaque on the
mucosa palate. On this background we can see dilated ducts of minor salivary glands. Diagnosis?
Red flat lichen
lupus erythematosus
smokers leukoplakia
Hyperplasic candidiasis
There is no right answer
There are plaques on the cheek mucosa of the 55 years old male. Recently, cracks and sores appeared
on the affected areas, painful while eating. What is the most likely diagnosis?
Leukoplakia, a typical form
Red flat lichen
Chronic aphthous stomatitis
There is no right answer
E. *
316.
A.
B.
C.
D. *
E.
317.
A.
B.
C.
D. *
E.
318.
A.
B.
C.
D. *
E.
319.
A.
B.
C.
D. *
E.
320.
A.
B.
C.
D.
E. *
Leukoplakia, erosive and ulcerative form
The patient (60 y.o.) has a rare warty globular neoplasm (diameter 8 mm) on the left cheek. It's
clearly separated from the healthy skin. Surface is granular, grayish-brown, no hair. At the touch is
soft, easily movable relative to the skin. It has appeared 6 years ago. What is the most likely
diagnosis?
skin
corner
neurofibromas
rodent ulcer
skin papilloma
There is no right answer
52 y.o. patient complains on formation appearance on the lower lip. After the survey there was
established diagnosis of precancerous hyperkeratosis. What treatment shold we apply?
Keratoplastic means
Antibiotic
Anti-inflammatory therapy
Surgical removal
Medical observation without treatment
The patient (45 y.o.) complains on feeling burning, tightness of buccal mucosa, roughness,
uncomfortable feeling. Objectively: there is a dense grayish-white area on the lower lip red rim and
hyperemic cheek mucosa. This area rises over the mucous membrane in the form of white warty
growths. Dental plaque. Put the most likely diagnosis:
Verrucous leukoplakia
Red flat lichen
Leukoplakia, flat shape
Chronic hyperplastic candidiasis
Galvanic stomatitis.
A 49 year old patient complains about pain in the oral cavity induced by eating. He suffers from
CHD. Objective examination revealed dyspnea, limb edema. Oral cavity isn't sanitated. On the
mucous membrane, on the right, there is an ulcer with irregular edges covered with greyish-white
necrotic deposit with low-grade inflammation around it. There is also halitosis. What is the most
probable diagnosis?
Cancerous ulcer
Traumatic ulcer
Tuberculous ulcer
Trophic ulcer
Ulcero-necrotic stomatitis
A 45 year old homeless and unemployed patient complains about body temperature rise up to 39oC,
pain during eating and deglutition, nasal haemorrhages. He has been suffering from this for 10 days.
Objectively: herpetic rash on the lips, irregular-shaped erosions covered with fibrinous deposit on the
mucous membrane of oral cavity; filmy deposits on the tonsils. Liver is enlarged and sclerotic. Blood
count: erythrocytes - 4,5*1012l; hemoglobin - 120 g/l; ESR - 25 mm/h; leukocytes – 10*109l;
eosinophils - 0; rod nuclear cells - 2; segmentonuclear leukocytes - 31, lymphocytes - 41; monocytes
- 10; atypical mononuclears - 14%, plasmatic cells - 2. What is the most probable diagnosis?
Acute leukosis
Acute herpetic stomatitis
Oropharyngal diphtheria
AIDS
Infectious mononucleosis
321.
A. *
B.
C.
D.
E.
322.
A.
B.
C. *
D.
E.
323.
A.
B.
C.
D.
E. *
324.
A. *
B.
C.
D.
E.
325.
A.
B.
C.
D.
E. *
A 34-year-old male patient consults a dentist about an ulcer on the hard palate. It appeared about a
month ago. He has treated it by rinsing with herbal water, but the ulcer is gradually "creeping".
Objectively: there is a shallow erethistic ulcer with uneven and undermined edges of soft consistency
within the mucous membrane of hard palate. Granulations of the ulcer floor are also present.
Yellowish granules are visible on the ulcer periphery. What is the most likely diagnosis?
Tuberculous ulcer
Trophic ulcer
Cancerous ulcer
Syphilitic ulcer
Actinomycosis
A 39 year old patient applied to a dental clinic for oral cavity sanation. Anamnesis data: mild case of
diabetes. Objectively: mucous membrane of cheeks is unchanged, mainly in retromolar area there are
symmetrically placed whitish papulae protruding over the mucous membrane and forming a lace-like
pattern. On the upper jaw there are two soldered bridge dentures, the 47 tooth has an amalgam filling,
the 46 tooth has a steel crown. What is the most probable diagnosis?
Leukoplakia
Secondary syphilis
Lichen ruber planus
Chronic atrophic candidosis
Lupus erythematosus
A 49-year-old patient was diagnosed with leucoplakia nicotinica Tappeiner. What pathohistological
process predominates in histologic pattern of this disease?
Papillomatosis
Acanthosis
Dyskeratosis
Parakeratosis
Hyperkeratosis
A 17-year-old male patient complains about pain in the oral cavity, ulceration, body temperature up
to 38oC, headache. Objectively: mucous membrane of the oral cavity is hyperemic and edematic.
There are a lot of confluent erosions of polycyclic shape, covered with grey and white deposit,
located on hard palate, gums, lips. What is your provisional diagnosis?
Acute herpetic stomatitis
Erythema multiforme
Pemphigus vulgaris
Aphthous fever
Allergic stomatitis
A 16-year-old patient complains about body temperature rise, weakness, pain induced by
eating and deglutition. Objectively: mucous membrane of the oral cavity is erythematic with
multiple petechia. Pharynx is hyperaemic. Regional lymph nodes are enlarged, mobile, painless. In
blood: leukocytosis, monocytosis, atypic mononuclears, ESR is 30 mm/h. What is the leading factor
Immediate
allergy
of disease development?
Bacterial infection
Autoimmune disorders
Delayed allergy
Viral infection
326.
A. *
B.
C.
D.
E.
327.
A.
B.
C.
D.
E. *
328.
A.
B.
C. *
D.
E.
329.
A.
B.
C. *
D.
E.
330.
A.
B.
C.
D. *
E.
A 23-year-old patient complains about a small ulcer on the red border of her lower lip that has been
irresponsive to self-treatment for two weeks. Objectively: unchanged red border of lower lip has a
circular ulcer of 2 mm in diameter with raised regular edges, its floor is of meat-like colour, dense,
shiny, with "stearic film", with cartilaginoid infiltration, painless on palpation. Regional lymph node
is enlarged, of tight elastic consistency, painless, mobile. What is the most likely diagnosis?
Primary syphilis
Decubital ulcer
Cancerous ulcer
Lupus erythematosus
Tuberculous ulcer
A 31-year-old male patient complains of dryness and burning of tongue back that appeared for about
a week ago and get worse when he eats irritating food. The patient has a history of recent pneumonia.
He had been treated in the in-patient hospital for 2 weeks, the treatment program included antibiotics.
Now he doesn't take any drugs. Objectively: mucous membrane of the oral cavity is hyperemic, dry,
glossy. Tongue back and palate have greyish-white plicae that can be easily removed. Threads of
saliva trail behind the spatula. What is the most likely diagnosis?
Chronic atrophic candidiasis
Chronic hyperplastic candidiasis
Acute atrophic candidiasis
Medicamental stomatitis
Acute pseudomembranous candidiasis
A 35-year-old patient complains about itch, burning and edema of lips. He has been suffering from
this for a week. Objectively: reddening of red border and skin, especially in the region of mouth
corners, there are also vesicles, crusts, small cracks along with erythematous affection of red border.
What is the most likely diagnosis?
Acute herpetic cheilitis
Multiform exudative erythema
Acute eczematous cheilitis
Allergic contact cheilitis
Exudative form of exfoliative cheilitis
A 19-year-old girl complains about having crusts, lip tenderness, especially at lip joining.
Objectively: there are yellow-brown crusts on the lip red border from Klein zone to it's middle, after
their removal bright red smooth surface without erosions appears. Mucous membrane in Klein zone
is slightly hyperemic and edematic. What is the most likely diagnosis?
Exudative form of cheilitis exfoliativa
Epidermolysis bullosa
Exudative form of cheilitis actinica
Meteorological cheilitis
Eczematous cheilitis
A 64-year-old female patient was waiting for her turn at the dentist’s. Suddenly she fell down, her
respiration became hoarse, she got convulsive twitching in her upper and lower limbs, face and neck
turned cyanotic, eye pupils became mydriatic, reaction of eye to light was absent. Arterial pressure
and pulse couldn’t be measured. Heart sounds couldn’t be auscultated. Involuntary urination was
noted. What condition is characterized by such symptoms?
Coma
Epilepsy
Shock
Clinical death
Collapse
331.
A.
B.
C. *
D.
E.
332.
A.
B.
C.
D.
E. *
333.
A.
B. *
C.
D.
E.
334.
A.
B.
C.
D.
E. *
335.
A.
B.
C. *
D.
E.
A 50-year-old female patient complains about sensation of tightness of buccal mucosa and roughness
of the lateral surface of tongue. The patient undergoes regular check-up at a dispensary department
for compensated form of diabetes mellitus. Objectively: there are white and grey areas in form of
lacy pattern on the buccal mucosa on the right and on the lateral surface of tongue. The surface of
affected region cannot be scraped off. What is the most likely diagnosis?
Lupus erythematosus
Secondary syphilis
Lichen ruber planus
Pseudomembranous candidiasis
Leukoplakia
A 27-year-old patient suffers from a disease without prodromal manifestations that declares itself
through oral mucosa lesion consisting of 1-2 roundish elements 5-8 mm large which are
circumscribed by a hyperemic rim and covered with yellow-grey coating. The disaese recurrence is
observed quite regularly 3-4 times a year. Which presentations are typical for the following disease?
Papular syphilis
Lichen ruber planus
Chronic herpes recidivicus
Erythema multiforme
Chronic recurrent aphthous stomatitis
A 56-year-old patient complains about dryness and burning of the lateral surface of her tongue. These
sensations disappear during eating. She noted such sensations three months ago. She has a history of
gastritis with reduced secretory function. Objectively: mucous membrane of tongue and oral cavity
has no peculiarities. The back of tongue has thin white coating. Regional lymph nodes are
unpalpable. Oral cavity is sanitized. What is the most likely diagnosis?
Lingual nerve neuritis
Glossodynia
Candidiasis
Desquamative glossitis
Hunter-Moeller glossitis
A 35-year-old woman complains about experiencing lip dryness and desquamation for a month.
Application of indifferent ointments is ineffective. Objectively: red border of lower lip is of rich red
color, it is moderately infiltrated, covered with closely adhering greyish scales, it bleeds and hurts in
the attempt to remove them. Opacification of epithelium in form of white stripes is present in the
nidus periphery, there is also an area of depression in the centre. What is the most likely diagnosis?
Cheilitis exfoliativa
Commissural cheilitis
Lichen ruber planus
Leukoplakia
Lupus erythematosus
A 46-year-old broker complains about a sensation of foreign body on his tongue, discomfort during
talking, oral cavity dryness. Objectively: there are dark filiform papillae up to 5 mm long on the back
of tongue. What is the most likely diagnosis?
Benign migratory glossitis
Median rhomboid glossitis
Glossophytia
Fissured tongue
Acute glossitis
336.
A.
B.
C. *
D.
E.
337.
A.
B.
C.
D.
E.
338.
A.
B.
C. *
D.
E.
339.
A. *
B.
C.
D.
E.
340.
A.
B. *
C.
D.
E.
A 23-year-old woman complains about severe pain in the mouth, body temperature up to 38oC,
indisposition. The same condition occurs periodically for several years after catching a cold.
Objectively: the lips are covered with bloody crusts, there are opened bladders and erosions, covered
with fibrinogenous deposit on mucous membrane of lips and cheeks that is apparently hyperemic and
edematic. Hypersalivation is present. What is the most likely diagnosis?
Dermatitits multiformis, Duhring's disease
Pemphigus vulgaris
Erythema multiforme
Nonacantholytic pemphigus
Chronic herpes recidivicus
A 35 year old teacher complains about periodical appearance of small ulcers in the oral cavity. She
has been suffering from this for 5 years, recurrences happen 4-5 times a year. The ulcer healing lasts
for 10 days. Objectively: on a mucous membrane of lower lip there is a roundish lesion element 0,5
cm large covered with white deposit and surrounded by hyperemia border, very painful when
touched. What is the most probable diagnosis?
Chronic recurrent aphthous stomatitis
Traumatic erosion
Secondary syphilis
Chronic recurrent herpes
Duhring's herpetiform dermatitis
A 64 year old patient complains of pain in the area of mucous membrane of hard palate on the left
that is getting worse during eating with use of a complete removable denture. He has been suffering
from this for 1,5 month. Objectively: left-sided hyperemia and edema of mucous membrane of hard
palate; at the border of distal denture edge there is an ulcer with dense walls and fundus, surrounding
tissues are infiltrated. The ulcer floor is tuberous, covered with fibrinous deposit; ulcer palpation is
painful. What examination method is to be applied in the first place?
Cytology
Bacterioscopy
Biopsy
Allergic contact plastic test
Serological reactions
A 31-year-old patient complains about mouth soreness, body temperature rise up to 38,5oC,
indisposition. Such condition has been occurring periodically for several years after the patient had
had a cold. Objectively: lips are covered with haemorrhagic crusts, hyperaemic mucous membrane of
lips and cheeks has erosions covered with fibrinous films. Hypersalivation is present. What is the
most likely diagnosis?
Multiform exudative erythema
Pemphigus vulgaris
Herpes recidivicus
Herpetiform Duhring's dermatitis
Stevens-Johnson syndrome
A 32-year-old male patient complains about tongue pain that is getting worse during eating and
talking. Objectively: there is a painful ulcer 0,6 cm large on the lateral surface of tongue. The floor is
covered with grey deposit. The crown of the 47 tooth is destroyed. What is the most likely diagnosis?
Trophic ulcer
Decubital ulcer
Hard chancre
Tuberculous ulcer
Cancerous ulcer
341.
A.
B.
C.
D.
E. *
342.
A.
B.
C.
D. *
E.
343.
A.
B.
C.
D. *
E.
344.
A.
B. *
C.
D.
E.
345.
A.
B.
C. *
D.
E.
346.
A patient complains of burning, itch and lower lip enlargement. He has been suffering from this for a
long time. Objectively: the patient's face is asymmetric due to the flattening of nasolabial fold. His
lower lip is edematic, of normal colour, painless on palpation. The patient has plicated tongue. What
is your provisional diagnosis?
Granulomatous Miescher's cheilitis
Quincke's edema
Lymphangioma
Hemangioma
Melkersson-Rosenthal syndrome
An 17 year old patient complains of gingival painfulness and haemorrhage, halitosis, temperature rise
up to 38,6oC, general weakness, appetite loss. Objectively: mucous membrane of oral cavity is
hyperemic and dry; tongue is covered with white fur, gingival papillae are edematic, their apices have
areas of necrotic deposit that can be easily removed leaving bleeding surface beneath. Submaxillary
lymph nodes are enlarged, palpatory painful. What is the causative agent of this disease?
Virus of herpes simplex
Streptostaphylococci
Pale treponema
Fusospirochetal symbiosis
Candida fungi
A 44-year-old patient complains about a rapidly growing formation on his lower lip. Examination of
the red border of lips revealed a greyish-red nodule with a hollow in the centre which is filled with
corneous masses that can be easily removed. The nodule is painless, mobile. What is your provisional
Basal
cell carcinoma
diagnosis?
Papilloma
Nodulous verrucous precancer of red border
Keratoacanthoma
Localized precancerous hyperkeratosis of red border
A young woman complains about having vesicles in oral cavity, painful ulcers, especially during
talking and eating; muscle and joint pain, body temperature rise, indisposition, weakness. She fell
suddenly ill 2 days ago. Objectively: to- 38,4oC. Vermilion border is covered with bloody crusts that
stick together and impede mouth opening. Mucous membrane of lips, cheeks, mouth floor, tongue,
soft palate is hyperemic and edematic; there are single vesicles and large painful erosions covered
with fibrinous deposit on it. Regional lymph nodes are enlarged and painful. There is hypersalivation.
Nikolsky's symptom is negative. What is the most probable diagnosis?
Acantholytic pemphigus
Multiform exudative erythema
Secondary syphilis
Acute herpetic stomatitis
Non-acantholytic pemphigus
A steeplejack with a long record of service consults a dentist about dryness, burning and insignificant
lip edema. The same symptoms were noted one year ago in autumn. What is the most likely
diagnosis?
Cheilitis
glandularis
Contact cheilitis
Meteorogical cheilitis
Manganotti's cheilitis
Cheilitis exfoliativa
A 41 year old patient applied to a dentist and complained about white caseous deposit on the dorsum
of his tongue and burning sensation. It is known from the patient's anamnesis that he underwent
treatment in an in-patient hospital on account of pneumonia. What is the most probable diagnosis?
A.
B.
C.
D.
E. *
347.
A.
B.
C. *
D.
E.
348.
A.
B.
C.
D.
E. *
349.
A.
B. *
C.
D.
E.
350.
A.
B.
C. *
D.
E.
351.
A.
Lupus erythematosus
Typical form of leukoplakia
Lichen ruber planus
Scarlet fever
Acute pseudomembranous candidosis
A patient complained about frequent haemorhages from the mucous membrane of oral and nasal
cavities, he mentioned also that his father had the same problems. Objectively: there are multiple
telangiectasias and angimatous formations on face skin as well as on mucous membrane of nose,
cheeks and lips. Blood count is normal. What is the most probable diagnosis?
Vaquez disease
Werlhof's disease
Rendu-Osler-Weber disease
Cushing's basophilism
Addison-Biermer disease
A 41-year-old builder complains about a condyloma on his lower lip. It appeared 1,5 month ago. It
has been significantly growing throughout the last week. Objectively: the red border of the lower lip
is cyanotic and infiltrated, it has some isolated closly adhering squamae. There is a well-defined
hemispherical formation 8 mm in diameter and 4 mm high in the centre. The formation is of
grey-and-blue-and-red colour, it has rough surface formed by thin, closely adhering and thick-based
squamae. Regional lymph nodes are enlarged, mobile, dense and painless. What is the most likely
Viral
wart
diagnosis?
Verruciform precancer
Pyogenic granuloma
Keratoacanthoma
Lower lip cancer
A patient complains about pain in the oral cavity, burning and dryness. Examination revealed
fiery-red dry mucous membrane. The tongue is crimson, dry, glossy, filiform papillae are atrophied.
There is some deposit in tongue folds that is hard to be removed. The patient undergoes treatment for
pneumonia, she takes antibiotics. What is the most likely diagnosis?
Fastened erythema
Acute atrophic candidiasis
Pellagrous glossitis
B2 hypovitaminosis
Benign migratory glossitis
Examination of a 22 year old patient revealed chronic candidosis of oral mucous membrane,
generalized lymphadenopathy. Anamnesis data: the patient has been suffering from herpes for a year.
Body temperature persistently rises up to 37,4-37,5oC, body weight has reduced by 8 kg over the last
month. What disease can be indicated by this symptom group?
Acute leukosis
Infectious mononucleosis
AIDS
Chronic leukosis
Candidosis
A 24-year-old patient complains about acute pain in the mouth, headache, articular pain, body
temperature rise up to 38,6oC. Red border of lips is covered with haemorrhagic crusts, mucous
membrane of the oral cavity has big erosions and ulcers coated with greyish incrustation. Hand skin
exhibits erythematous spots 1-1,5 cm in diameter with a vesicle in the middle. What is the most likely
diagnosis?
Lyell's syndrome
B.
C. *
D.
E.
352.
A.
B.
C. *
D.
E.
353.
A.
B.
C.
D. *
E.
354.
A. *
B.
C.
D.
E.
355.
A.
B.
C. *
D.
E.
356.
A.
B.
C.
D.
Behcet's syndrome
Stevens-Johnson syndrome
Multiform exudative erythema
Medicamentous stomatitis
A 61-year-old patient complains of a painless formation on his tongue that appeared several months
ago. Objectively: the patient has a lot of carious and completely decayed teeth, lateral surface of
tongue exhibits a painless whitish formation 10x5 mm large with irregular surface in form of
verrucae. Histological examination revealed thickening of corneal epithelial layer of intermittent
keratinization type. What is the most likely diagnosis?
Hyperplastic form of candidiasis
Verrucous precancer
Verrucous form of leukoplakia
Hyperkeratotic form of lichen ruber planus
Keratoacanthoma
An 17-year-old student complains about ulceration in the oral cavity, spontaneous bleeding of
mucous membrane, pain during food intake and talking, nosebleeds. He has a history of: aggravation
of general condition, weakness, body temperature rise up to 39oC, headache, joint pain. What method
of diagnostics should be applied to confirm the diagnosis?
HIV test
Blood sugar test
Immunogram
Clinical blood analysis
Allergy test
A 24 year old patient applied to a clinic for the purpose of oral cavity sanitation. During preparation
of the 45 tooth on account of chronic median caries the patient turned pale, there appeared cold
clammy sweat on her forehead, nausea, ear noise. The patient lost consciousness. Objectively: pulse 50 bpm, AP - 80/60 mm Hg, shallow breath, miotic pupils. Make a diagnosis of this state:
Syncope
Anaphylactic shock
Stenocardia
Collapse
Quincke's edema
A 22-year-old patient was delivered to a traumatology centre with a dirty cut wound of her right foot.
A doctor performed initial surgical d-bridement and made an injection of antitetanus serum. Some
time later the patient’s condition got abruptly worse: she developed extreme weakness, dizziness,
palpitation. Objectively: the skin is pale, the patient has cold sweat, frequent pulse of poor volume at
a rate of 100 bpm, AP is 90/40 mm Hg. What is the cause of such aggravation?
Pain shock
Haemorrhagic shock
Anaphylactic shock
Infectious-toxic shock
Drug disease (seroreaction)
A 27-year-old man attended a glasshouse in a botanic garden. After he had smelt at an orchid he
turned pale and lost consciousness. Objectively: heart rate is 115/min, arterial pressure is 50/0 mm
Hg. What drug should be injected to the patient in the first place?
Dimedrol
Cordiamin
Strophanthine
Mesaton
E. *
357.
A. *
B.
C.
D.
E.
358.
A. *
B.
C.
D.
E.
359.
A.
B.
C.
D.
E. *
360.
A.
B.
C.
D.
E. *
361.
A. *
B.
C.
D.
E.
362.
Prednisolone
To the dentist clinic appealed patient (20 years old), with the aim of oral rehabilitation. There are
main and auxiliary methods of examination of patients. What methods of examination in therapeutic
dentistry are main?
examination, palpation, percussion, probing
percussion, probing
X-ray
Termo-test
All answers are correct
The patient, 34 year old, complains of a constant aching pain in the 25tooth, which enhanced while
biting. Objectively: the chewing surface of the cavity of 25-th communicates with the cavity of the
tooth.The folder in the transition area above 25-th tooth are edematous, hiperemic, painful at
palpation, percussion of 25-th is sharply painful. What method of research is necessary to conduct for
the diagnosis?
X - ray
Kulazhenko’ test
blister test
reoparodontographia
Provocative tests.
Patient, 18 years old, complains of heartburn in the mouth, pain when eating. In anamnesis treatment
of pneumonia with large doses of antibiotics. On hiperemic mucosa of tongue and cheeks there are
white patches, which can be easily removed. What research method is used for diagnosis?
X- ray
Kulazhenko’ test
blister test
reoparodontographia
microbiological investigation.
Patient K., 37 years old, complains of dryness and edema of oral mucosa. She took antibiotics
becouse of the treatment of somatic pathology. On the back of the tongue there are white-yellowish
plaques that are difficult to remove. What examination is necessary to conduct for clarifing the
Xray
diagnosis?
Kulazhenko’ test
blister test
reoparodontographia
microbiological investigation.
The patient O., 62 years old appealed to the dentist with complaints of pain and the presence of ulcers
under the tongue. The patient uses removable prostheses(dentures), 3 months ago he noticed a tumor
under the tongue. On examination - at the bottom of the oral mucosa is infiltrated ulcer. Regional
lymph nodes are not changed. What auxiliary investigation should be conducted?
A biopsy of the tumor
Clinical analysis of blood
X-ray of the mandible
Puncture of the tumor
Ultrasonography of the neck
The patient V., 33 years old appealed to the dentist with complaints of pain in 37 tooth, which
amplifies while biting and irradiates over the course of the trigeminal nerve. The tooth was treated
previously. Examination: the crown of 37 is destroyed at 1/3, percussion is positive, oral mucosa
above 37 tooth is hyperemic, edematous, painful on palpation. What kind of investigation is needed
for the setting of the final diagnosis?
A.
B.
C. *
D.
E.
363.
A.
B.
C. *
D.
E.
364.
A.
B. *
C.
D.
E.
365.
A. *
B.
C.
D.
E.
366.
A. *
B.
C.
D.
E.
Probing of the 37-th tooth cavity
Determination of the degree of 37-th tooth mobility
X-ray of 37-th tooth
Determine the depth of gum pockets in 37-th tooth
Electroodontodiagnosis of 37-th tooth
The patient F., 18 years old, complains on increased sensitivity to sweet and sour. Objectively: on the
cervical parts of 14, 13, 12, 23, 24 are chalky-liked, dull spots with indistinct edges. What additional
method of examination should be undertaken to clarify the diagnosis?
Elektroodontodiagnosis
Thermometry
methylene blue coloring
Probing
X-ray
51 year old patient appealed to the clinic with complains of the pain in the lower right lip, which
amplifies while taking food and during conversation. The first symptoms appeared about a month
ago. On examination: on the lower lip in the region of 42.43 is a deep mucosal defect rounded 0.8 cm
in diameter, on the basis it is infiltrated, with elevated edges, flat bottom, covered with fibrinous
plaque, coarsen on the edge, painful. Crowns of 41,42,43 are destroyed, have sharp edges. What
additional examination is necessary to conduct to determine the tactics of the treatment?
allergological
Cytological
Biochemical
Microbial
Immunological
Patient A., 35 year old, complains of burn of the tongue tip, which increases during meals and
dryness of oral mucosa. In the anamnesis - pneumonia 2 months ago. OBJECTIVE: dryness,
hyperemia and edema of the mucous membrane of mouth, on the back of the tongue yellowish
caseous layers are revealed. Sharp edges of the cavities of 26, 34,35 and 46 teeth are visible. Hygiene
index by Fedorov - Volodkina - 2.6 points. What investigation should be conducted to establish the
Microbial
final diagnosis?
General blood test
Blood test for glucose
Biomicroscopic
Analysis of immunological parameters of mixed saliva
During examination the 45 year old patient there were noticed mild edema and hyperemia of the
marginal gum throughout. Periodontal pocket with the depth of 3-6 mm with a purulent exudate.
GI-2,1 points. Traumatic occlusion in the area of lateral teeth. Radiologically: resorption of
interdental membranes from 1/3 to 1/2 lengths of the tooth root. What should be considered, when
causal therapy is assigned?
Microbiological investigation of contents of periodontal pockets
Age of the patients
Concomitant somatic disease
Data Kulazhenko test
General blood test
367.
A. *
B.
C.
D.
E.
368.
A. *
B.
C.
D.
E.
369.
A.
B. *
C.
D.
E.
370.
A.
B.
C. *
D.
E.
371.
A.
B.
C.
D. *
E.
372.
A.
B.
Patient K., 22 years old complains of pain in the gums that amplifies when eating, foul odor from
mouth, bad state of health. Temperature - 38.8 C. Consider himself ill for 2 days, after the removal of
28 teeth. OBJECTIVE: mucosa of gums is hiperemic, swollen, bleeds easily when probing. Gum
region in 23, 24, 25, 26 and 27 teeth covered with gray necrotic bloom. Significant dental deposits.
Regional lymph nodes are enlarged, painful on palpation. At what additional investigation it is
necessary to direct the patient?
X-ray of 28 tooth hole
Fluorescent
Microscopy
Transillumination
Serological
Patient Z., 44 year old, complaints of bleeding gums. OBJECTIVE: gums edge is swollen, hiperemic,
bleeding of the mild touch. It is defined periodontal pockets of 3-3.5 mm. On the neck of the teeth –
hard dental deposits. At what additional investigation it is necessary to direct the patient?
X-ray of jaws
Schiller-Pisarev test
Assessment of tooth mobility
Determination of capillary resistance
Determination of periodontal index
Patient K., 18, applied with household face trauma. On examination, the timer of 21-st tooth on 1/3
of the crown, probing through fracture is painful, reaction to cold causes intermittent pain, percussion
painless. What research is primarily to hold for the diagnosis and selection of treatment?
EDI
X-ray
Palpation of the apex of the root projection
Metulen- blue coloring
Termotest
Patient S., 23 years old, complains of sharp pain in the gums, inability to eat, bad smell from the
mouth, raising the temperature to 38oC, general weakness. On examination: the patient is pale, with
enlarged lymph nodes. Edema, hyperemia of gum papillae, some ulcers are covered with necrotic
bloom. What additional research is necessary to clarify the diagnosis?
General blood test
urine test
Panoramic X-ray
allergological tests
serological tests
Patient A., appealed to the dentistry clinic, where she was diagnosed with folded tongue. What is a
module according to ICD-10 diseases?
K14.1;
K14.3;
K14.4;
D. K14.5;
K14.6.
A girl, 17 year old, complains about deterioration and appearance of rashes on the lips. From history:
the same rashes occurred one year ago in the same place. Objectively: body temperature is 37.1 C, on
the edge of the red border of the upper lip and the skin are grouped vesicles with a diameter of 1-3
mm with clear contents. Indicate the most likely diagnosis.
herpes zoster;
acute herpetic stomatitis;
C.
D.
E. *
373.
A.
B. *
C.
D.
E.
374.
A. *
B.
C.
D.
E.
375.
A.
B.
C.
D. *
E.
376.
A.
B. *
C.
D.
E.
377.
A.
B.
C.
D.
E. *
stomatitis at measles;
stomatitis at chicken pox;
chronic recurrent aphthous stomatitis.
A patient, 57 year old, has symptoms of galvanosis. Objectively: there are gold, metal crowns on the
teeth and couple amalgam fillings in the mouth; the oral mucosa is hyperemic and swollen. Which
way can prevent the occurrence of galvanosis?
replacement of amalgam fillings;
using of homogeneous metal for prosthesis;
taking of vitamins A and E;
rinsing with antiseptics;
desynsebilizition drugs.
An 19 year old boy was under treatment in the infectious disease department. He was taking
ampicillin. On the 6th day of treatment the white deposits similar of caseous films were revealed on
the hyperemic mucous membrane in the area of gingival torus, cheeks and on the tongue. The films
can be removed leaving hyperemic surface underneath. General condition is satisfactory. Body
temperature is 36,7oC. What is the provisional diagnosis?
acute candidiasis stomatitis;
drug-induced stomatitis;
chronic candidiasis stomatitis;
acute herpetic stomatitis;
mild form of leukoplakia.
A 64 year old patient has a complete removable denture and complains of pain in the area of mucous
membrane of hard palate on the left that is getting worse during eating. He has been suffering from
this for 1,5 month. Objectively: left-sided hyperemia and edema of mucous membrane of hard palate;
at the border of distal denture edge there is an ulcer with dense walls, surrounding tissues are
infiltrated. The ulcer floor is tuberous, covered with fibrinous deposit; ulcer palpation is painful.
What kind of examination methods will be applied in the first place?
allergic contact plastic test;
cytology;
serological reactions;
bacterioscopy;
biopsy.
A 23 year old patient complains about high body temperature, halitosis. Objectively: gingiva is
swollen, easy bleeds by touching, apexes of interdental papillae affected by necrosis and remind
beheaded cones. At cytology detected fuzo-spirochetes symbiosis. The diagnosis is:
leukemia;
Vincent’s ulcer-necrotic stomatitis;
allergic stomatitis;
secondary syphilis;
avitaminosis.
A 62 year old patient complains of fever to 38C, burning pain, presence of vesicles on the tongue to
the left. Objectively: there are bubbles on the hyperemic edematous mucosa of the tongue on the left
as a chain filled with serous fluid, regional lymphadenitis. What is a diagnosis?
blister-vascular syndrome;
primary herpetic infection;
recurrent herpes;
multiform exudative erythema;
shingles.
378.
A.
B.
C.
D.
E. *
379.
A.
B.
C.
D. *
E.
380.
A.
B.
C. *
D.
E.
381.
A.
B.
C.
D.
E. *
382.
A.
B.
C.
D. *
E.
383.
A. *
B.
In 19 year old young man on buccal mucosa and tongue was detected a cheesy white film that is
easily removed. It began appeared 4 days ago. The young man held a long course of antibiotic
therapy. What is a probable diagnosis?
chronic aphthous stomatitis;
chronic candidiasis;
exudative erythema multiforme;
acute herpetic stomatitis
acute candidiasis.
A 30 year old patient complains of headache, fever to 39C, general weakness and the appearance of
the oral mucosa cruel painful erosions. Objectively: on the buccal, lips mucosa--- multiple group
erosions 2-3mm in diameter with scalloped edge covered with grayish-white bloom. The surrounding
mucosa is hyperemic, regional lymph nodes are enlarged, painful on palpation. At cytology identified
giant multinuclear cells, macrophages. What is the final diagnosis?
erosive stomatitis;
vulgar pemphigus;
exudative erythema multiforme;
herpetic stomatitis;
shingles.
A 21 year old man complains about presence of painful ulcers in the mouth. Similar symptoms were
noted six months ago. Objectively: on the lateral surface of the tongue and at the bottom of the oral
cavity detected two erosions circular form, covered with grayish-white bloom. Erosions have
hyperemic border. Its edges slightly raised above the mucosa. What is the most likely diagnosis?
syndrome Behcheta;
exudative erythema multiforme
chronic recurrent aphthous stomatitis;
Stevens-Johnson syndrome;
recurrent herpetic stomatitis
A 51 year old patient complains of burning and dryness of the tongue that appeared about 10 days
ago and intensified when taking spicy food. Two weeks ago was in the hospital and received large
dose antibiotics. On the hyperemic and edematous cheeks and tongue mucosa gray-white color
crumbs like plaque that are easily removed by erasure. What are found on smears from lesions during
giant
multinuclear cells;
microscopy?
single cell fungi form;
threads of mycelium;
giant cells Lanhansa;
numerous cells of fungi.
A 23 year old man notes available on the palate wound that appeared after the session intrachannel
electrophoresis of 26 tooth by 5% solution of iodine. Objectively: on the hard palate mucosa to the
left is the lesion area dashed form with whitish-gray surface. What is the probable cause of lesions?
chemical injury;
mechanical trauma;
thermal damage;
electrical injury;
galvanism.
A 75 year old patient complains about appearance of rash on the face and oral mucosa, severe
neuralgic pains. After examination was determined diagnosis - shingles. What is the typical clinical
picture for this disease?
sharply painful blisters and erosions in a chain over the course of the branches of the trigeminal
nerve;
rash blisters and vesicles in different parts of the skin, lips red border and the mouth;
C.
D.
E.
384.
A.
B.
C.
D.
E. *
385.
A.
B.
C.
D. *
E.
386.
A.
B.
C.
D.
E. *
387.
A.
B.
C.
D. *
E.
388.
A.
B. *
C.
D.
E.
389.
rash with blisters and bubbles with serous and serous-hemorrhagic exudates;
polymorphism of elements lesions;
single painful round erosions with red border and fibrous plaque.
A 44 year old patient complains regarding low-grade fever, malaise, bleeding gums, difficult meal.
Objectively: gum edge and top of interdental papillae are covered with dirty-gray plaque, edge of
ulcers rough under dig, bad breath, increased regional nodes. The diagnosis is:
scarlet fever;
diphtheria;
measles;
herpangina;
Vincent’s ulcer-necrotic stomatitis.
A 77 year old patient complains about appearance of rash on the face and oral mucosa, severe
neuralgic pains. After examination was determined diagnosis-shingles. In the absence of
complications the disease continues average:
1-3 days;
8-12 days;
2-3 months;
2-3 weeks;
1-2 weeks.
A 35 year old patient complains about appearance of ulcer under the tongue. From history: pathology
of the gastrointestinal tract. Objectively: under his tongue is a round aphthae 5-7 mm surrounded by a
rim of inflammatory hyperemia covered with fibrinous plaque, sharply painful to the touch. What is a
diagnosis?
syndrome Behcheta;
recurrent herpes;
stomatitis Settona;
secondary syphilis;
chronic recurrent aphthous stomatitis.
A 62 year old patient complains of fever to 38C, burning pain, the presence of vesicles on the tongue
to the left. Objectively: on the hyperemic edematous mucosa of the tongue on the left are bubbles as a
chain filled with serous fluid, regional lymphadenitis. What is a diagnosis?
blister-vascular syndrome;
primary herpetic infection;
recurrent herpes;
shingles;
exudative erythema multiforme.
A 62 year old patient complains about pain, appearance of ulcer in the mouth. During 12 years the
patient used a mandible partial denture and suffering from ischemic heart disease, PI-3.0. After
examination was determined diagnosis: decubital ulcer of mouth floor. Which of the following will
help to eliminate the cause of the disease?
treatment in cardiology;
production of a new partial denture;
cutting sores;
application of kerato plastic products;
improve an oral hygiene.
A 31 year old patient complains about appearance of pain ulcer on lower lip mucosa that appeared
before. The patient suffers from chronic enterocolitis. On the lower lip mucosa (right side) is a
surface rounded defect tissue surrounded by the whisk of hyperemia covered with fibrous plaque,
palpation painful. The diagnosis is:
A. *
B.
C.
D.
E.
390.
A.
B. *
C.
D.
E.
391.
A.
B.
C.
D.
E. *
392.
A.
B.
C.
D. *
E.
393.
A.
B. *
C.
D.
E.
394.
A.
B.
C.
D.
E. *
395.
chronic recurrent aphthous stomatitis;
traumatic erosion;
recurrent herpes;
stomatitis Settona;
secondary syphilis.
A 54 year old patient complains of mouth burning, pain during meal, whitish patches on the cheeks
and tongue. From history the patient for a long time treated with antibiotics regarding pneumonia. On
hyperemic buccal and tongue mucosa is a white film that is difficult to be removed. What kind of the
additional methods can clarify the diagnosis?
histological;
bacterioscopic;
serological;
allergological tests;
stomatoskopy.
A girl is sick couple days, her body temperature is normal. Objectively: on the hyperemic edematous
mucosa of cheeks and tongue dorsum is a white film like a cottege cheese. After its removal remains
bright hyperemic surface with hemorrhages. The patient was treated with antibiotics. What is a
previous diagnosis?
mild form of leuckoplakiy;
acute herpetic stomatitis;
chronic candidiasis stomatitis;
recurrent aphthous stomatitis;
acute candidiasis stomatitis.
A 68 year old patient appealed to the dentist with complaints of rash on the face and oral mucosa,
severe neuralgic pains. After examination was diagnosed—shingles. For local treatment of the
disease
are used:
metronidazole;
ointments: nistatynova, levorinov;
Solcoseryl-jelly, aerosol “Livian”;
bonafton, acyclovir, Zovirax-ointments;
nysoral, suprastin, multivitamins.
A 35 year old patient has diagnosis erosive stomatitis. Which group of drugs should be used at the
final stage of treatment?
antibiotics;
keratoplastics;
glucocorticosteroids;
antioxidants;
proteolytic.
The patient in 55 year old with diabetes mellitus uses removable dentures and complains of
heartburn, dry mouth, pain during eating. At survey on buccal and tongue mucosa, hard palate is lean
hyperemic plaque that taking off hardly. After its removal appears a visible erosive surface. What is a
probable diagnosis?
acute aphthous stomatitis;
acute pseudomembranous candidiasis;
erosive form of leuckoplakiy;
red lichen planus, erosive form;
chronic atrophic candidiasis.
A patient is diagnosed with Vincent ulcerative-necrotic stomatitis based on clinical and laboratory
findings. What treatment does need to prescribe to the patient?
A.
B.
C.
D. *
E.
396.
A.
B.
C.
D. *
E.
397.
A. *
B.
C.
D.
E.
398.
A.
B.
C.
D.
E. *
399.
A.
B.
C. *
D.
E.
400.
A.
B.
C.
D.
E. *
multivitamins;
antibiotics,
corticosteroids, antiseptics;
proteolytic enzymes, antiseptics;
antiviral, immunomodulators.
A 24 patient complains of a painful rash on the mucosa of the upper and lower lip, raising body
temperature to 39C. Easy bleeding, hyperemic swollen gums are painful. The mucous has rash
blisters and erosions. What is a diagnosis?
measles;
ulcer-necrotic stomatitis;
chronic recurrent herpetic stomatitis;
acute herpetic stomatitis, mild form;
scarlet fever.
A 67 year old patient appealed to the dentist with complaints of rash on the face and oral mucosa,
severe neuralgic pains. After examination was diagnosed—shingles. What kind of additional methods
can clarify the diagnosis?
histological;
bacterioscopic;
virus isolation;
determination of immunity;
biopsy.
An 17 year old girl complains about the pain in the mouth, increasing the body temperature until
38,5C. After the exam was diagnosed with acute herpetic stomatitis. What will to be found in the
cytological study of erosion surface?
elements of the reticuloendothelial system;
akantolityc cells;
atypical cells;
cells of Pirogov-Lanhansa;
giant multinuclear cells.
A 24 year old patient applied on the second day from the disease beginning and was diagnosed with
acute herpetic stomatitis. Which ointment is necessary to apply the first for treatment of elements
lesions?
ointment with corticosteroids;
ointment with antibiotics;
antiviral;
ointment wityh sulfonamides;
ointment with proteolytic enzymes.
A 51 year old patient complains about mouth dryness and white plaque on the tongue, cheecks that
appeared after taking of antibiotics dose. The dry oral mucosa has bright red color and covered with
white color crumbs like plaque that removed by cotton roll. What is a probable diagnosis?
lupus erythematosus;
leukoplakia of the mucous;
red plane lichen;
syphilitic papules;
chronic candidiasis stomatitis.
401.
A.
B.
C. *
D.
E.
402.
A. *
B.
C.
D.
E.
403.
A.
B.
C.
D. *
E.
404.
A. *
B.
C.
D.
E.
405.
A.
B.
C.
D. *
E.
406.
A.
A 29 year old patient complains of headache, fever to 39C and appearance on the lips mucosa cruel
painful erosions. Objectively: on the edge of a red border and skin of the upper lip there are multiple
group erosions 1-3mm in diameter with serous fluid, covers of some bubbles have destroyed. The
surrounding mucosa is hyperemic, regional lymph nodes are enlarged, painful on palpation. At
cytology identified giant multinuclear cells. Which drug of local action should be used for etiotropic
phenylbutazone;
treatment?
klotrimazol;
Zovirax;
lorinden A;
solcoseryl.
A 52 year old patient complains about dryness, burning of the mouth that increase during meal.
Objectively: partial denture in upper jaw, tongue is swollen and smooth, there are prints of teeth on
the lateral tongue surfaces, atrophied papillae. The tongue folds have baldly plaque that removes with
efforts. What is the final diagnosis?
chronic atrophic candidiasis;
acute atrophic candidosis;
diamond-shaped glossitis;
desqumative glossitis;
leukoplakia.
After the flu a 31 year old patient complains about general weakness, gums pain, their bleeding and
halitosis. Objectively: regional lymph nodes are enlarged, painful on palpation, the hyperemic,
swollen gingiva has numerous necrotic ulcers covered with grayish-green plaque. What will be found
on smears from lesions during microscopy?
neuthrophils, epithelial cells, cocci;
erythrocytes, leukocytes, fungi Candida;
neyseriyi, veylonely bacteria collie;
significant number of fuzobacterias, spirochetes;
lactobacilli, gonococci, erythrocytes.
A patient complains about pain in the tongue duiring meal, talk. Sick 2 weeks ago. The lateral surface
of the tongue mucosa has irregular ulcer with hyperemic edges covered with necrotic plaque. Its
palpation is painful. Crowns of 46 and 47 teeth are destroyed. What are the priority actions of
dentist? of traumatic factors;
removal
analgesia lesions;
application of anti-inflammatory drugs;
application of kerato plastic drugs;
surgery.
A 25 year old patient complains of fever to 38,5C, pain duiring meal. There are small vesicles with
clear fluid and bright red erosions with scalloped outlines covered with plaque on the lips and
cheecks mucosa. Regional nodes are painful. The diagnosis is:
erosive stomatitis;
exudative erythema multiforme;
shingles;
herpetic stomatitis;
malignant pemphigus.
A patient complains about an ulcer that appears periodically on the mucous membrane of the mouth.
He is sick on pancreatitis, enterocolitis. On the left surface of the tongue is defined oval ulcer covered
with whitish-yellow plaque surrounded by edge of hyperemia, painful on palpation.What is most
likely diagnosis?
aphtha Settona;
B.
C. *
D.
E.
407.
A.
B.
C.
D.
E. *
408.
A.
B. *
C.
D.
E.
409.
A. *
B.
C.
D.
E.
410.
A.
B.
C.
D. *
E.
411.
A.
B. *
C.
D.
chronic recurrent herpetic stomatitis;
chronic recurrent aphthous stomatitis;
secondary syphilis;
traumatic errosion.
A girl, 19 year old, complains about deterioration and appearance of rashes on the lips. From history:
the same rashes occurred one year ago in the same place. Objectively: body temperature is 37.1C, on
the edge of the red border of the upper lip and the skin are grouped vesicles with a diameter of 1-3
mm with clear contents. Indicate the likely diagnosis.
herpes zoster;
acute herpetic stomatitis;
stomatitis at measles;
stomatitis at chicken pox;
chronic recurrent aphthous stomatitis.
A patient, 56 year old, has symptoms of galvanosis. Objectively: there are gold, metal crowns on the
teeth and couple amalgam fillings in the mouth; the oral mucosa is hyperemic and swollen. Which
way can prevent the occurrence of galvanosis?
replacement of amalgam fillings;
using of homogeneous metal for prosthesis;
taking of vitamins A and E;
rinsing with antiseptics;
desynsebilizition drugs.
An 19 year old boy was under treatment in the infectious disease department. He was taking
ampicillin. On the 6th day of treatment the white deposits similar of caseous films were revealed on
the hyperemic mucous membrane in the area of gingival torus, cheeks and on the tongue. The films
can be removed leaving hyperemic surface underneath. General condition is satisfactory. Body
temperature is 36,7oC. What is the provisional diagnosis?
acute candidiasis stomatitis;
drug-induced stomatitis;
chronic candidiasis stomatitis;
acute herpetic stomatitis;
mild form of leukoplakia.
A 63 year old patient has a complete removable denture and complains of pain in the area of mucous
membrane of hard palate on the left that is getting worse during eating. He has been suffering from
this for 1,5 month. Objectively: left-sided hyperemia and edema of mucous membrane of hard palate;
at the border of distal denture edge there is an ulcer with dense walls, surrounding tissues are
infiltrated. The ulcer floor is tuberous, covered with fibrinous deposit; ulcer palpation is painful.
What kind of examination methods will be applied in the first place?
allergic contact plastic test;
cytology;
serological reactions;
bacterioscopy;
biopsy.
A 26 year old patient complains about high body temperature, halitosis. Objectively: gingiva is
swollen, easy bleeds by touching, apexes of interdental papillae affected by necrosis and remind
beheaded cones. At cytology detected fuzo-spirochetes symbiosis. The diagnosis is:
leukemia;
Vincent’s ulcer-necrotic stomatitis;
allergic stomatitis;
secondary syphilis;
E.
412.
A.
B.
C.
D.
E. *
413.
A.
B.
C.
D. *
E.
414.
A.
B.
C.
D.
E.
415.
A.
B.
C.
D.
E.
416.
A.
B.
C.
D.
E.
417.
avitaminosis.
A 61 year old patient complains of fever to 38C, burning pain, presence of vesicles on the tongue to
the left. Objectively: there are bubbles on the hyperemic edematous mucosa of the tongue on the left
as a chain filled with serous fluid, regional lymphadenitis. What is a diagnosis?
blister-vascular syndrome;
primary herpetic infection;
recurrent herpes;
multiform exudative erythema;
shingles.
A 47 year old patient complains about pain in the oral cavity induced by eating. He suffers from
CHD. Objective examination revealed dyspnea, limb edema. Oral cavity isn't sanitated. On the
mucous membrane, on the right, there is an ulcer with irregular edges covered with greyish-white
necrotic deposit with low-grade inflammation around it. There is also halitosis. What is the most
probable diagnosis?
Cancerous ulcer
Traumatic ulcer
Tuberculous ulcer
Trophic ulcer
Ulcero-necrotic stomatitis
Patient , 43 year old, complains of cracks in the corners of his mouth , also of photophobia,
lacrimation, weakness, loss of appetite, burning sensation in the tongue. OBJECTIVE: lips are red,
swollen, cracked in the corners of the mouth (which are covered with crusts). Red border of lips are
dry, scaly, similar to peeling of the skin around the lips. Tongue is red, shiny, filiform papillae are
atrophied. What can be the cause of this disease:
Hypovitaminosis PP
Hypovitaminosis B6
Hypovitaminosis A
Hypovitaminosis B12
Hypovitaminosis B1
42 y.o. woman complains of pain, burning the tongue tip, taste disturbances. Also, fatigue, headache,
tinnitus, dyspnea. Skin and mucous membranes are pale, with a lemon-yellow colour. Henter-Miller
tongue. What will be the diagnosis?
Hypovitaminosis of vitamin B1
desquamative glossitis
diamond tongue
There is no right answer
Pernicious anemia
During the examination of 31 y.o. patient, doctor found symmetrical erythema on exposed areas of
the body, dark pigmentation and keratinization. Oral mucous is hyperemic, with hyperemic swollen
tongue, filiform papillae are atrophied. The patient complains of headache, insomnia, loss of appetite
and loose stools. Deficiency of what vitamin can cause these changes?
Riboflavin.
Ascorbic acid.
Retinol.
Nicotinic acid.
Thiamine.
Patient complains on dry mouth, cracks in the corners of the mouth, pain during food taking. There is
gastritis in anamnesis. During examination - lips are swollen, with erosion and cracks in the corners
of the mouth. Tongue is shiny, smooth, covered with furrows. What will be the diagnosis?
A.
B.
C.
D.
E.
418.
A.
B.
C.
D.
E.
419.
A.
B.
C.
D.
E.
420.
A.
B.
C.
D.
E.
421.
A.
B.
C.
D.
E.
422.
A.
B.
C.
Hypovitaminosis B2
Hypovitaminosis C
Hypovitaminosis PP
Hypovitaminosis A
Hypovitaminosis B1
Patient, 34 y.o., complains of dry skin and oral mucosa. From anamnesis - last month intensively
trained in the gym, and then visited a solarium. During examination: oral mucosa sufficiently
hydrated, salivation is reduced, buccal epithelium is thinned. Red border of lips are dry and flaky.
What is the most likely diagnosis?
Hypovitaminosis E
Hypovitaminosis B2
Hypovitaminosis A
Hypovitaminosis C
Hypovitaminosis PP
Patients, 42 y.o., complains of a burning sensation and pain in the tongue, loss of appetite, fatigue,
loss of memory. In anamnesis – exacerbation of chronic hastritis. OBJECTIVELY: on the skin of the
face and neck there are erythematous patches with peel. In the mucosa of the mouth can be seen areas
of hyperemia and normal mucosa. Tongue is swollen, bright red, shiny, filiform papillae are
atrophied. Deficiency of what vitamin can cause this pathology?
Vitamin B1
Vitamin A
Vitamin B2
VitaminuV12
Vitamin C
Patient, 49 y.o., was diagnosed with pernicious anemia.What glossitis is one of the symptoms of that
anemia?
Henter-Miller glossitis
Desquvamative
Fissured
Rhomboid
Candidiasis of the tongue
Patient complains of burning, dryness and pain in area of the mucous membrane of the mouth, lips,
tongue while eating. These symptoms are combined with weakness, rapid fatigue. OBJECTIVELY:
oral mucosa is pale, slightly moist, tongue is swollen, increased in dimensions, papillaes are
atrophied, especially at the tip of the tongue. Back of the tongue is bright red. Put the diagnosis.
Pernicious anemia, Addison-Biermer
Hypochromic iron-deficiency anemia
Desquamativ glossitis
Rhomboid glossitis
Glossalgia
A 41 year old man complains of pain and gums bleeding (especially during food taking), weakness,
pain in the limbs, chilliness unwarranted. He live in the North during last 15 years. OBJECTIVELY:
skin is dry, with brownish tinge, scaly, with symptom of “goose skin”. The mucous membrane of the
gums is swollen, hyperemic with cyanotic tinge, gingival papillae are hypertrophied, covering the
crowns of the teeth and bleeds after touching. Pathological tooth mobility of 1th degree is present. In
the mucosa of the soft palate numerous petechiae was finded. Deficiency of which vitamin may cause
vitamin
E picture:
this clinical
Vitamin B12
Vitamin A
D.
E.
423.
A.
B.
C.
D.
E.
424.
A.
B.
C.
D.
E.
425.
A.
B.
C.
D. *
E.
426.
A.
B. *
C.
D.
E.
427.
A.
B. *
C.
D.
E.
428.
A.
B.
C.
D.
E. *
Vitamin C
Vitamin B
48 y.o. patient complains of painful rash on the lips, which are burning, itching and aggravated
during meals. His condition was deteriorated in recent months. OBJECTIVELY: on edematous red
lips border and surrounding skin erythematous patches can be seen with erosions, fissures and peels.
Between them there are atrophic scars. What is the most likely diagnosis:
Exfoliative cheilitis, exudative form
Erosive-ulcerative form of leukoplakia
Erosive-ulcerative form of lupus
Abrasive prekantseroznyy cheilitis Manganotti
Erosive-ulcerative form of lichen ruber planus
During 24 y.o. patient examination, the redness in the corners of the mouth was found. A few days
after in these areas painful cracks were revealed. Objectively: lips are red, swollen, with cracks and
erosion, there is glossitis, accompanied by a sharp pain. With a lack of a vitamin such symptoms
occur? Select the one correct answer
B2.
PP.
B1.
S.
A.
25 year woman complains about enlargement, itching and dryness of her lips. These changes she
noticed after using of new lipstick. Objectively: lips are edematous, compact-grained, palpation is
painless. Regional lymph nodes are not enlarged. What can be the diagnosis?
Lip cancer
Exfoliative cheilitis
Actinic cheilitis
Allergic cheilitis
Toxic cheilitis
45 year old patient was diagnosed with “hairy tongue”. What is the main symptom of this disease?
Enlargement of the tongue
Enlargement of the filiform papillae
Enlargement of the fungiform papillae
Brown tongue as a result of smoking
Hair in the middle part of the tongue
54 year old patient was diagnosed with glossophytia. What medicine will be included in the
treatment?
Antibiotics
Antiseptics
Antihistamines
Analgetics
Vitamins
During the examination of 34 year old woman angular cheilitis was diagnosed. What can be the
reason of this cheilitis?
Low immunity
Virus
Fungal infection
Trauma
All answers are right
429.
A. *
B.
C.
D.
E.
430.
A.
B.
C.
D.
E. *
431.
A.
B.
C. *
D.
E.
432.
A. *
B.
C.
D.
E.
433.
A.
B. *
C.
D.
E.
434.
A.
B. *
22 year old boy complains about swelling and eversion of his lower lip. Objectively: enlargement of
the lower lip caused by mucous gland hyperplasia. The “dew” symptom is positive. What can be the
diagnosis?
Cheilitis glandularis
Cheilitis exfoliative
Actinic cheilitis
Allergic cheilitis
Cheilitis granulomatouz
A 42 year old patient applied to a dentist and complained about white caseous deposit on the dorsum
of his tongue and burning sensation. It is known from the patient's anamnesis that he underwent
treatment in an in-patient hospital on account of pneumonia. What is the most probable diagnosis?
Lupus erythematosus
Typical form of leukoplakia
Lichen ruber planus
Scarlet fever
Acute pseudomembranous candidosis
A 52-year-old patient complains about dryness and burning of the lateral surface of her tongue. These
sensations disappear during eating. She noted such sensations three months ago. She has a history of
gastritis with reduced secretory function. Objectively: mucous membrane of tongue and oral cavity
has no peculiarities. The back of tongue has thin white coating. Regional lymph nodes are
unpalpable. Oral cavity is sanitized. What is the most likely diagnosis?
Lingual nerve neuritis
Glossodynia
Candidiasis
Desquamative glossitis
Hunter-Moeller glossitis
A 19-year-old girl complains about having crusts, lip tenderness, especially at lip joining.
Objectively: there are yellow-brown crusts on the lip red border from Klein zone to it's middle, after
their removal bright red smooth surface without erosions appears. Mucous membrane in Klein zone
is slightly hyperemic and edematic. What is the most likely diagnosis?
Exudative form of cheilitis exfoliativa
Epidermolysis bullosa
Exudative form of cheilitis actinica
Meteorological cheilitis
Eczematous cheilitis
A 35-year-old patient complains about itch, burning and edema of lips. He has been suffering from
this for a week. Objectively: reddening of red border and skin, especially in the region of mouth
corners, there are also vesicles, crusts, small cracks along with erythematous affection of red border.
What is the most likely diagnosis?
Acute herpetic cheilitis
Multiform exudative erythema
Acute eczematous cheilitis
Allergic contact cheilitis
Exudative form of exfoliative cheilitis
16 year old female patient complains about enlargement, itching and dryness of her lips. These
changes she has noticed after using of new lipstick. Objectively: lips are edematous,
compact-grained, palpation is painless. Regional lymph nodes are not enlarged. Which ointment can
With
analgineto the patient?
you prescribe
With prednisolone
C.
D.
E.
435.
A.
B. *
C.
D.
E.
436.
A. *
B.
C.
D.
E.
437.
A. *
B.
C.
D.
E.
438.
A. *
B.
C.
D.
E.
439.
A.
B.
C.
D.
E.
440.
A.
With erythromycin
With antibiotics
With steroids
A patient complains about pain in the oral cavity, burning and dryness. Examination revealed
fiery-red dry mucous membrane. The tongue is crimson, dry, glossy, filiform papillae are atrophied.
There is some deposit in tongue folds that is hard to be removed. The patient undergoes treatment for
pneumonia, she takes antibiotics. What is the most likely diagnosis?
Fastened erythema
Acute atrophic candidiasis
Pellagrous glossitis
B2 hypovitaminosis
Benign migratory glossitis
A 41 year old patient applied to a dentist and complained about white caseous deposit on the dorsum
of his tongue and burning sensation. It is known from the patient's anamnesis that he underwent
treatment in an in-patient hospital on account of pneumonia. What was the doctors mistake in
pneumonia treatment?
He should prescribe the eubiotics and probiotics during the antibiotic course
The course of antibiotics was too long
The course of antibiotics was too short
He should prescribe oral antibiotics additionally
He should prescribe antihistamines additionally
26 year old man, builder, complains about itch and burning lip pain during every summer last three
years. In other time of the year this symptoms disappeared. What can be the diagnosis?
Meteorological cheilitis
Exfoliative cheilitis
Cheilitis glandular
Atrophic candidiasis
Meteorological glossitis
Woman complains about white deposits which she has noticed a week ago after pleuritis treatment.
Microscopic examination revealed abundance of yeast fungus. Blood tests are normal. What
medicine can cause this complication?
Antibiotics
Antihistamines
Vitamins
Antifungal
No correct answer
Woman complains about white deposits which she noticed a week ago after pleuritis treatment.
Microscopic examination revealed abundance of yeast fungus. Blood tests are normal. What is the
most probable diagnosis?
Candidiasis
Hairy tongue
Cancer
Leukoplakia
Herpes
60 year old patient complains about mouth dryness, bad breath, sensation of a foreign body on his
tongue. Objectively: 5 mm long brown papillae on the upper surface of tongue. What is the most
likely diagnosis?
Granulomatous cheilitis
B. *
C.
D.
E.
441.
A.
B.
C.
D.
E. *
442.
A.
B.
C. *
D.
E.
443.
A.
B.
C.
D. *
E.
444.
A.
B.
C. *
D.
E.
445.
A.
B. *
C.
D.
E.
446.
A. *
B.
C.
Hairy tongue
Actinic cheilitis
Meteorological cheilitis
Geographic tongue
During the examination of 17 year boy, few areas of desquamation was revealed.This areas (from the
patients words) are present from early childhood and looks like map. Patient feels no pain or other
complains. What is the most likely diagnosis?
Granulomatous cheilitis
Hairy tongue
Actinic cheilitis
Meteorological cheilitis
Geographic tongue
46 year old patient complains about painful lower lip. This pain he noticed 6 month ago. Objectively:
deep fissure in the middle part of lower lip and red border with brown crust and dense borders. What
will be the treatment in that case?
Antibiotics
Antiseptics
Surgical treatment
Corticosteroids
Proteolytic enzymes
24 year old woman was diagnosed with eczematous cheilitis. What will be the treatment?
There is no need to prescribe the medicine, only to avoid the cause.
To avoid the cause and prescribe topical corticosteroids
To prescribe topical corticosteroids
To avoid the cause, to prescribe topical corticosteroids and antihistamines
No correct answer
26 year old patient complains about swelling, redness and dryness of his lips. Objectively: the skin of
the lipsin vermillion area is thinned and red, presense of scaly pathes can be noticed. From
anamnesis: he works as a beach lifeguard. What should be the doctors tactics at that case ?
To take the biopsy first
To prescribe antibiotics
To prescribe topical retinoids
Laser therapy
Cryotherapy
A steeplejack with a long record of service consults a dentist about dryness, burning and insignificant
lip edema. The same symptoms were noted one year ago in autumn. He was diagnosed with
meteorological cheilitis. What recommendation can you give at that case?
To improve his oral hygiene
To use oil liniment at that time
To use antiseptics
To use analgetics
All answers are right
22 year old patient applied to a dentist and complained about white caseous deposit on the dorsum of
his tongue and burning sensation. It is known from the patient's anamnesis that he underwent
treatment in an in-patient hospital on account of pneumonia. What was the doctors mistake in
He
should prescribe
the eubiotics and probiotics during the antibiotic course
pneumonia
treatment?
The course of antibiotics was too long
The course of antibiotics was too short
D.
E.
447.
A.
B. *
C.
D.
E.
448.
A.
B. *
C.
D.
E.
449.
A.
B.
C. *
D.
E.
450.
A. *
B.
C.
D.
E.
451.
A. *
B.
C.
D.
E.
452.
A. *
B.
C.
He should prescribe oral antibiotics additionally
He should prescribe antihistamines additionally
Patient, 23 y.o., female, complains about pain in the oral cavity, burning and dryness. Examination
revealed fiery-red dry mucous membrane. The tongue is crimson, dry, glossy, filiform papillae are
atrophied. There is some deposit in tongue folds that is hard to be removed. The patient undergoes
treatment for pneumonia, she takes antibiotics. What is the most likely diagnosis?
Fastened erythema
Acute atrophic candidiasis
Pellagrous glossitis
B2 hypovitaminosis
Benign migratory glossitis
Woman complains about enlargement, itching and dryness of her lips. This changes have been
noticed after using of new lipstick. Objectively: lips are edematous, compact-grained, palpation is
painless. Regional lymph nodes are not enlarged. Which ointment can you prescribe to the patient?
With analgine
With prednisolone
With erythromycin
With antibiotics
With steroids
The patient complains of existing painful sores in the mouth. From history: deterioration of health,
weight loss, fever up to 37.1, diarrhea that lasts for three weeks. Which test should pass the patient?
general blood test;
test for markers of hepatitis;
test ELISA;
fungal tests;
examination of salivary glands.
The patient complains about the discomfort in the mouth, weight loss, temperature 37.1 that doesn’t
decrease within 2 months. Objectively: there are white hair growths on the tongue edge. What is the
preliminary diagnosis?
hairy leukoplakia;
chronic hyperplastic candidiasis;
acute hyperplastic candidiasis;
hairy tongue;
soft leukoplakia.
At the clinical examination in 50 years old male revealed a chronic mucosal candidiasis, generalized
lymphadenopathy. During the year he concerned regarding chronic recurrent herpes, fever to
37,4-37,5, body weight decreased by 8 pounds in the last month. This clinical picture indicates:
AIDS;
infectious mononucleosis;
acute leukemia;
chronic leukemia;
lymphogranulomatosis.
At the clinical examination of 44 years old patient revealed a chronic mucosal candidiasis,
generalized lymphadenopathy. During the year he concerned regarding chronic recurrent herpes,
fever to 37,4-37,5, body weight decreased by 8 pounds in the last month. This clinical picture
AIDS;
indicates:
infectious mononucleosis;
acute leukemia;
D.
E.
453.
A.
B.
C.
D.
E. *
454.
A. *
B.
C.
D.
E.
455.
A. *
B.
C.
D.
E.
456.
A.
B.
C.
D. *
E.
457.
A.
B.
C.
D.
E.
chronic leukemia;
lymphogranulomatosis.
The patient A., 40 years old, has a disfunction of the gastrointestinal tract, blood forming and nervous
systems. Skin and oral mucous membrane are pale, with yellow shade, sometimes there are dotted
haemorrhage. Make a diagnosis.
leukopenia;
leukemia;
trombocytopenic purpura;
hypo avitaminosis C;
B-12 (folate-) deficiency anemia.
32 y.o. patient was diagnosed with pernicious anemia. What glossitis is one of the symptoms of that
anemia?
Henter-Miller glossitis
desquvamative
fissured
rhomboid
candidiasis of the tongue
The patient complains of burning, dryness and pain in area of the mucous membrane of the mouth,
lips, tongue during eating. These symptoms are combined with weakness, rapid fatigue.
OBJECTIVELY: oral mucosa is pale, slightly moist, tongue is swollen, increased in dimensions,
papillaes are atrophied, especially at the tip of the tongue. Back of the tongue is bright red. Put the
pernicious
diagnosis. anemia, Addison-Biermer;
hypochromic iron-deficiency anemia;
desquamative glossitis;
rhomboid glossitis;
glossalgia.
A 42 year old man complains of pain and gums bleeding (especially during food taking), weakness,
and pain in the limbs, chilliness unwarranted. He lives in the North during last 15 years.
OBJECTIVELY: skin is dry, with brownish tinge, scaly, with symptom of “goose skin”. The mucous
membrane of the gums is swollen, hyperemic with cyanotic tinge; gingival papillae are
hypertrophied, covering the crowns of the teeth and bleeds after touching. Pathological tooth mobility
of 1st degree is present. In the mucosa of the soft palate numerous petechiae was finded. Deficiency
of which vitamin may cause this clinical picture:
vitamin E;
vitamin B12;
vitamin A;
vitamin C;
vitamin B.
During 21 y.o. patient examination, the redness in the corners of the mouth was found. A few days
after in these areas painful cracks was revealed. Objectively: Lips are red, swollen, with cracks and
erosion; there is glossitis, accompanied by a sharp pain. With a lack of a vitamin such symptoms
occur? Select the one correct answer.
B2;
PP;
B1;
C;
A.